185
MECHANICS UNIT I Mechanical Systems : The Mechanical System - Gen- eralized Coordinates - Constraints - Virtual work - Energy and Mo- mentum. Chapter 1 : Sections 1.1 to 1.5. UNIT II Lagrange’s Equations : Derivation of Lagrange’s equa- tions - Examples - Integrals of motion. Chapter 2 : Sections 2.1 to 2.3 (omit sec 2.4) UNIT III Hamilton’s Equations : Hamilton’s Principle - Hamil- ton’s Equation - Other variational Principle. Chapter 4: Sections 4.1 to 4.3 omit Section 4.4. UNIT IV Hamilton - Jacobi Theory : Hamilton Principle func- tion - Hamilton - Jacobi Equation - Separability. Chapter 5 : Sections 5.1 to 5.3 UNIT V Canonical Transformation : Differential forms and gen- erating functions - Special Transformations - Lagrange and Poisson brackets. Chapter 6 : Sections 6.1, 6.2 and 6.3 (omit sections 6.4, 6.5 and 6.6) Contents and Treatment as in : D. Greenwood. classical Dynam- ics. Prentices, Prentice Hall of India, New Delhi, 1985. Reference : 1. H. Goldstein, Classical Mechanics, (2nd Editin) narosa Publish- ing House, New Delhi. 2. N.C. Rane and P.S.C. Joag, Classical Mechanics, tata McGraw Hill, 1991. 3. J.L. Synge and B.A. Griffth, Principles of Mechanics (3rd Edi- tion) MCGraw Hill Book Co New yuk, 1970. 1

MSc Maths Optional Paper V

Embed Size (px)

Citation preview

Page 1: MSc Maths Optional Paper V

MECHANICS

UNIT I Mechanical Systems : The Mechanical System - Gen-

eralized Coordinates - Constraints - Virtual work - Energy and Mo-

mentum. Chapter 1 : Sections 1.1 to 1.5.

UNIT II Lagrange’s Equations : Derivation of Lagrange’s equa-

tions - Examples - Integrals of motion. Chapter 2 : Sections 2.1 to 2.3

(omit sec 2.4)

UNIT III Hamilton’s Equations : Hamilton’s Principle - Hamil-

ton’s Equation - Other variational Principle. Chapter 4: Sections 4.1

to 4.3 omit Section 4.4.

UNIT IV Hamilton - Jacobi Theory : Hamilton Principle func-

tion - Hamilton - Jacobi Equation - Separability. Chapter 5 : Sections

5.1 to 5.3

UNIT V Canonical Transformation : Differential forms and gen-

erating functions - Special Transformations - Lagrange and Poisson

brackets. Chapter 6 : Sections 6.1, 6.2 and 6.3 (omit sections 6.4, 6.5

and 6.6)

Contents and Treatment as in : D. Greenwood. classical Dynam-

ics. Prentices, Prentice Hall of India, New Delhi, 1985.

Reference :

1. H. Goldstein, Classical Mechanics, (2nd Editin) narosa Publish-

ing House, New Delhi.

2. N.C. Rane and P.S.C. Joag, Classical Mechanics, tata McGraw

Hill, 1991.

3. J.L. Synge and B.A. Griffth, Principles of Mechanics (3rd Edi-

tion) MCGraw Hill Book Co New yuk, 1970.

1

Page 2: MSc Maths Optional Paper V

UNIT I

Introduction :

Mechanics forms the basis of physics it contributes to our knowl-

edge of people working of nature and universe classical mechanics deals

with the description of actual or possible motion of points like as well

as extended bodies.

The motion of celestial bodies as well as man made objects such as

space problems. Satellites etc., are fields. The various classical dynam-

ics, Hamilton mechanics, provides most modern research in frontial ar-

eas. Particularly relation between symmetric property or conservation

law.

Application :

Mechanics is the study of motion of physical bodies. The motion

of celestial bodies planets, stars etc., path of an artillery shell or of a

space satellites send from a earth to a planet are among its problems.

Classical mechanics denotes part of Mechanics. Classical dynamics

will also be interpreted to include the type of mechanical assising to

include the type of Mechanical assising out of the special theory of

relativity. Mechanics of a system of particles :

When a Mechanical system consists of two (or) more particles we

must distinguish between the external forces acted upon the particles

of the system forces not belongs to the system. The internal forces

assising on account of interaction between themselves.

Mechanics of a particles :

Let r be a radius of the particle from some given origin and v its

the vector velocity then

v =dr

dt.

The linear momentum p of the particle is defined as the product

of particle mass and its velocity. In consequence of interaction with

external objects and field the particle may experience forces of various

types.

2

Page 3: MSc Maths Optional Paper V

For example, gravitation or electro dynamics.

The Mechanic of particle is contained in Newton second law mo-

tion.

Equation of Motion :

The motion of particle is described by the differential equation

F ∝ a−→F = m−→a

= m

(

d−→vdt

)

=d

dt(m−→v ) =

d

dt(p)

−→F =

dp

dt

F =d

dt(mv)

= md

dt

(

dr

dt

)

= md2r

dt2

∴ F = md2r

dt2

Thus the equation of motion is a differential equation of second

order.

Equation of motion of System :

The equation of motion of a system of N particles is written as

mi

··−→r =−→Fi +

−→Ri (i = 1, 2, 3, · · · , N)

where miis the mass of the ith particle,−→Fi is the applied force

and−→Ri the constraint force.

Generalized coordinates :

To described the configuration of the system we select the smallest

possible number of variables which define configuraiton of the system.

These are called the generalized co-ordinates of the system.

3

Page 4: MSc Maths Optional Paper V

Configuration :

A set of generalized co-ordinates is any set of co-oirdinates which

described the configuration.

Notation for generalized Co-ordinates :

Generalized co-ordinates are q1, q2, · · · , qn(qi, i = 1, 2, · · · , n) the

symbols q1, q2, · · · , qn corresponding to the co-ordinates that we choose

to describe the motion.

1. When the particle move in a plane it described by cartesian co-

ordinates (x, y) or polar co-ordinates and soon

q1 = x

q2 = yor

q1 = r

q2 = θ

2. Some spherical symmetry the sherical co-ordinates is

q1 = r =√

x2 + y2

q2 = θ = cot−1

(

z√

x2 + y2

)

q3 = φ = tan−1

(y

x

)

Definitions :

1. Newton fundamental equation

2. System of particle

3. Degree of freedom

4. Generalized Co-ordinates

5. Configuration space

6. Holonomic constraints

7. Non polynomic constraints

4

Page 5: MSc Maths Optional Paper V

8. Scleronomic constraints

9. Rheonomic constraints

10. Clarnomic system

11. Realnomic system

12. Polyateral constraints

13. Unilateral constraints

Degree of freedom :

The number of degree of freedom is equal to the number of co-

ordinates minus the number of independent equation of constraints.

(or)

If the configuration of a system of a N particles described using de-

gree N cartesian relation these co-ordinates then the degrees of fredom

equals 3N − ℓ.

Example :

Let 3 particles connected by the rigid rods to form a triangular

body with the particles are at the corners. There particles are fixed

by a cartesian co-ordinates each rigid rod is represonted mathemati-

cally by indepenent constraints equation. All together there are three

constraints. Hence degree of freedom equals 9 − 3 = 6.

Generalized co-ordinates :

A set of parameters used to represent the configuration of a system

without ambiguity are called Generalized co-ordinates.

Configuration Space :

Let n be the degree of freedom of a system of N - particles. Let

its configurations be specified by n - generalized co-ordinates.

The n - dimensional space representing these Co-ordinates as a

single point q (q1, q2, · · · , qn) is called a configuation space.

Holonomic constraints :

Let the configuration of a system be specified by n - generalized

Co-ordinates (q1, q2, · · · , qn).

5

Page 6: MSc Maths Optional Paper V

Let us assume that ethere are k independent equation of constraints

of the form

φj (q1, q2, · · · , qn, t) = 0, j = 1, 2, 3, · · · , k.

A constraints which can be expressed in this fashion is known as a

Holonomic constraints.

Holonomic system :

A system is said to be Holonomic if its constraints are always Holo-

momic.

Example :

Consider the motion of 2 particles connected by a rigid rod of length

ℓ in xy - plane constraint equation is given by

(x2 − x1)2 + (y2 − y1)

2 = ℓ2.

Scleronomic constraints :

The constraints is said to be scleronomic if time t does not appear

explicity.

Example:

Consider the motion of two particle in the xy plane they are con-

nectd by a rigid rod of length ℓ. The corresponding constraint equation

is

ℓ2 = (x2 − x1)2 + (y2 − y1)

2.

where x1, x2, y1, y2 are cartesian co-ordinates of 2 particles.

The equation does not contain the time t explicity.

Scleronomic system :

A system is said to be scleronomic if its constraints are scleronomic.

Rhenomic system :

A system is said to be Rhenomic if its constraints are Rhenomic.

Rhenomic constraints :

If the length ℓ have been generalized as an explicity function of

time then the constraint is Rhenomic. OR Constraint relation depend

6

Page 7: MSc Maths Optional Paper V

explicity on time.

Non - Holonomic constraints :

Let a system has m contraints which are non - integrable and dif-

ferential expression is of the form

n∑

i=1

ajidqi + ajtdt = 0, j = 1, 2, 3, · · · ,m

where a′s is a function of the q′s and t.

This type of constraints are known as non- Holom Holkonomic

constraints.

Bilateral constraints :

If any configuration of the system for every small allowable dis-

placement in any fixed time. The negative displacement is also al-

lowable then the constraints are called bilateral constraint. They are

expressed as equalities.

Unilateral constraint :

In any configuration of the system for every small allowable dis-

placement in any fixed time either a positive or negative displacement

is allowable then the constraints are called unilateral constraints. They

are experessed as in equlities.

f (q1, q2, · · · , qn, t) ≤ 0 (unilateralconstraints)

f (q1, q2, · · · , qn, t) = 0 (Bilateral)

Example :

Let a free from particle be contained in a fixed hollowsphere of

radius r if its centre at the origin of a cartesian co-ordinates system.

If (x, y, z) are the co-ordinates of the particle then x2 + y2 + z2 ≤ 0

where r is trhe radius.

Virtural displacement :

Let a system of N particles is given by 3N cartesian co-ordinates

x1, x2, · · · , x3N with respect to some intertial frame adn may be subject

7

Page 8: MSc Maths Optional Paper V

to the contraints. At any given time let us assume that the co-ordinates

have small infinite displacement δx1, δx2, · · · , δx3N .

They are assumed to occure without passing of tiem and donot

necessary conform to the constraints. These small change Sk in the

configuration is known as Virtual displacement.

Virtual work :

Consider a system of N particles. Let the configuration to repre-

sented by the cartesian co-ordinates x1, x2, · · · , x3N .

Let f1, f2, · · · , f3N are the component forces applied at the corre-

sponding

Co-ordinates in a positive sense then the virtual workdone (δW )

of these forces for a virtual displacement δx is given by

δW =3N∑

j=1

−→Fj

−−→δxj

Note :

In a vector form δW =N∑

i=1

−→Fi· δ−→ri where

−→Fi is the applied force of

ith particle −→ri is the positive vector.

The forces are assumed to be constant through out the virtual

displacement.

Workless constraints :

A workless constraints is any bilateral constraint such that the

virtual work done by the corresponding constraint force is zero for any

virtual displacement, which is consistent with the constraint.

Examples :

1. Sliding motion on a frictionless susface.

2. Rolling contact without slipping.

Book work : The virtual displacement is not ingeneral a possible

real displacement.

Proof : In the virtual displacement, the moving constraints are

8

Page 9: MSc Maths Optional Paper V

assumen to be slopped.

Let the system have k holonomic constraints.

φj (q1, q2, · · · , q3N , t) = 0, j = 1, 2, 3, · · · , k. (1)

Let us take the totla differential of φ and obtain

dφj =∂φj

∂q1dq1 +

∂φj

∂q2dq2 + · · · +

∂φj

∂q3N

dq3N +∂φj

∂tdt = 0.

Suppose the system is subject to k holonomic constraints

φj (x1, x2, · · · , x3N , t) = 0, j = 1, 2, 3, · · · , k.

dφj =∂φj

∂x1

dx1 +∂φj

∂x2

dx+ · · · +∂φj

∂x3N

dx3N +∂φj

∂tdt = 0.

Virtual displacement

dxi = δxi, dt = 0 (2)

dφj =3N∑

i=1

∂φj

∂xi

dxi +∂φj

∂tdt = 0, j = 1, 2, 3, · · · , k.

Case (i) :

Suppose te system has non-holonomic constraint. Then

3N∑

i=1

ajidxi + ajtdt = 0, j = 1, 2, 3, · · · , k.

Virtual displacement dt = 0, dxi = δxi

n∑

i=1

ajiδxi = 0 (j = 1, 2, 3, · · · ,m)

Form (1) and (2) shows that any holonomic constraint must also

9

Page 10: MSc Maths Optional Paper V

be scleronomic constraint

i.e.,∂φj

∂t= 0, j = 1, 2, 3, · · · , k

Suppose non-holonomic constraint aj = 0 . These constraint are

not ment in the general case.

Hence a virtual displacement is not possible as a real displacement.

State and prove the principle of virtual work :

Statement:

The necessary and sufficient condition for the static equilibrium of an

initially mottionless sceleronomic which is subject of the system in to

the virtual workdone for an arbitrary virtual displacement satisfying

the constraint is zero.

Proof :

Case (i)

Let us assume that we are given an sclernomic system of N particles.

Let us assume that their system is in static equilibirium. Then for

each particle−→Fi +

−→Ri = 0

where−→Fi is the external force.

−→Ri is the constraint force acting on ith particle is

(−→Fi +

−→Ri

)

δ−→ri = δW, i = 1, 2, 3, · · · , n

where −→ri is the position vector of ith particle.

∴ Virtual workdone by all the force in moving through an arbitrary

vitrual displacement consistant with the constraint is zero.

N∑

i=1

(−→Fi +

−→Ri

)

δ−→ri = 0

N∑

i=1

−→Fiδ

−→ri +N∑

i=1

−→Riδ

−→ri = 0

10

Page 11: MSc Maths Optional Paper V

constraints are workless, thenN∑

i=1

−→Riδ

−→ri = 0

N∑

i=1

−→Fiδ

−→ri = 0

i.e., δW = 0

⇒ workdone = 0, i.e., the virual workdone by the applied force is zero.

Case (ii) :

Let us assume that the same system of particles are initially mo-

tionless. But it is not in static equlibirium.

The motion is s to C we can always choose a virtual displacement

in the direction of actual motion at each particle.

In this case, the virtual work is positive

N∑

i=1

−→Fiδ

−→ri +N∑

i=1

−→Riδ

−→ri > 0

δW =N∑

i=1

−→Fiδ

−→ri > 0,−→Fis 6= 0

δW > 0

If the system is not in equlibrium then the virtual workdone is not

zero.

1. Show that virtual workdowne by the constraint forces

on a system of two particles connected by the rigid massless

rod is zero.

Solution :−→Ri,

−→Ri are the constraint force along the rod on the 2 particles.

They are opoosite in sense with equal magnitude.

−→er is the unit vector along the rod. δ−→r1 , δ−→r2 are the virtual dis-

placement at the ends virtual workdone by a constraint force is given

11

Page 12: MSc Maths Optional Paper V

by

δWC =−→R1δ

−→r1 +−→R2δr2

−→R1 = R1 (−−→er ) = −R−→er

−→R2 = R2 (−→er ) = −R1

virtual displacement along the rod all equal

δ−→r1er = δ−→r2er

δWC = −R1−→er δ

−→r1 +R2−→er δ

−→r2= (R2 −R1)

−→er δr1

δWC = 0 (∵ R2 = R1 equal magnitude) .

Example for bilateral constraints :

2. Given an equation of scleronomic with workless constraints

(bilateral) to which the principle of the virtual work can be

applied (or)

Two frictionless blocks of equal mass m all connected by

the massless rigid rod using x1 and x2 as co-ordinates. Solve

for the force F2. If the system is in static equlibirium.

Solution :

R1 and R2 are the external constraint acting at the wall and floor

repectively.

The applied forces are the gravitational force acting in the blocks

and the external forces F2 system is in static equlibirium.

Virtual workdone by external force (F ) = 0

Workdone by constraint force (R) = 0

We have

mg(δx1) + F2(δx2) = 0. (1)

The virtual displacement along the rod at ends are equal.

∴ δx1 sin θ = δx2 cos θ (2)

12

Page 13: MSc Maths Optional Paper V

Multiply (1) by sin θ,

⇒ mg sin θδx1 + F2 sin θδx2 = 0

mgδx2 cos θ + F2 sin θδx2 = 0

(mg cos θ + F2 sin θ) δx2 = 0.

But displacement δx2 6= 0.

A frictionless system which is constrained to move in the vertical

plane.

mg cos θ + F2 sin θ = 0

F2 sin θ = −mg cos θ

F2 = −mg cot θ

3. Using a suitable example, show how the concept of

virtual work can be applied to system with unilateral con-

straints.

Solution :

Let us assume a cube of mass is placed between two frictionless

mutually perpenticular plane.

Assume that the motion is an vertical plane. Le x1 = x2 = 0 at the

equilibirium position then unilateral constraints are x1 ≥ 0, x2 ≥ 0.

The only applied force is the weight mg. It has components F1, F2 in

x1 and x2 direction.

m

450

450

g

x1

R 1 R2

x 2

When in equilibuim R1 = −F1 and R2 = −F2

δWC =mg√

2(δx1 + δx2) > 0

13

Page 14: MSc Maths Optional Paper V

F1 = F2 = −mg√2

Virtual workdone by applied forces is

δW =−mg√

2(δx1 + δ2) ≤ 0

Thus the virtual δW ≤ 0 for any virtual displacement consistent

with unilateral constraints.

To find workdone by constraint forces

R1 = R2 =mg√

2

Virtual workdone by constraint forces

δWC = R1δx1 +R2δx2 =mg√

2(δx1 + δx2) ≥ 0.

Hence for a system with unilateral constraints

1. Workdone by external force is −ve.

2. Workdone by constraint force is + ve.

D’ Alembert’s Principle :

Let the system has N particles. Consider on ith particle to the

system mi is the mass of the particle.−→ri is the position vector of particle.−→Fi is applied force of particle.

Let−→Ri be constraint force of particle.

Let··−→ri be the acceleration of ith particle relative to an inertial

frame. −→mi

··−→ri is the intertial force acting on the ith particle.

Now we have the equation

−→Fi +

−→Ri −−→mi

··−→ri = 0, i = 1, 2, 3, · · · , N. (1)

(The sum of all the force real and inertial acting on each particle of a

system is zero)

This result is known as D’ Alembert’s principle.

14

Page 15: MSc Maths Optional Paper V

Note :−→Fi and

−→Ri together are called as real forces.

Lagrange’s form of D’ Alembrt’s Principle :

Let us assume that the system has N particles. Let mi be the mass

of the ith particle frame.−→Fi is the applied forced and

−→Ri is the constraint force on the ith

particle.

−−→mi

··−→ri is inertial force acting on the particle. Now we have the

equation

−→Fi +

−→Ri −−→mi

··−→ri = 0 (i = 1, 2, 3, · · · , N) .

Now the virtual workdone by the system is

N∑

i=1

(−→Fi +

−→Ri −−→mi

··−→ri

)

δ−→r i = 0

−→Ri is the workless constraint forces

N∑

i=1

−→Riδ

−→ri = 0

N∑

i=1

(−→Fi −−→mi

··−→ri

)

δ−→ri = 0.

This equation is the Lagrange’s form of D’ Alemberts principle.

1. Obtain the differential equation of the given pendulum

(sphericla pendulum) or A particle of mass m is subspended

by a massless wire of length r = a + b cos ωt(a > b > 0) to form

a sphorical pendulum. Find the equation of motion.

Solution :

Let us assume that the spherical co-ordinates θ and φ.θ is mea-

sured from the upward vertical. φ is the angle between a verticla

plane through a supporting point and a vertical plane containing the

pendulum.

15

Page 16: MSc Maths Optional Paper V

O

r

m

efeq er

g

q

f

The acceleration of a particle whose spherical co-ordinates are

(r, θ, φ) is as follows

··−→r =

(

··

r − r·

θ2

− r·

φ sin2 θ

)

−→er +

(

r··

θ + 2·

θ − r·

φ2 sin θ cos θ

)

−→eθ

+

(

r··

φ sin θ + 2·

φ sin θ + 2r·

θ·

φ cos θ

)

−→eφ

where −→eθ ,−→er ,

−→eφ are unit vectors forming an orthogonal triad.

A virtual displacement constraint with the constraint is,

δ−→r = rδθ−→eθ + r sin δφ−→eφ .

The applied gravitational force is,

−→F = mg cos

(

1800 − θ)−→er +mg sin

(

1800 − θ)−→eθ

−→F = −mg cos θ−→er +mg sin θ−→eθ

we have(−→F −m

··

r)

δ−→r = 0 (By D’ Alemberts principle)

16

Page 17: MSc Maths Optional Paper V

(−mg cos θ−→er +mg sin θ−→eθ ) −m

((

··

r − r·

θ2

− r·

φ sin2 θ

)

−→er

)

+

(

r··

θ + 2·

θ − r·

φ2 sin θ cos θ

)

−→eθ +

((

r··

φ sin θ + 2·

φ sin θ + 2r·

θ·

φ cos θ

)

−→eφ

)

× rδθ−→eθ + r sin θδφ−→eφ = 0

Taking dot product,

mgr sin θ −mr

(

r··

θ + 2·

θ − r·

φ2 sin θ cos θ

)

δθ

−m(

r··

φ sin θ + 2·

φ sin θ + 2r·

θ·

φ cos θ

)

r sin θδφ = 0

δφ and δθ are independent Virtual displacements.

∴ co-efficeient are equal to zero respectively

mr

[

g sin θ −(

r··

θ + 2·

θ − r·

φ sin θ cos θ

)]

= 0 (1)

Since mr sin θ 6= 0

− sin θmr

(

r··

φ sin θ + 2·

φ sin θ + 2r·

θ·

φ cos θ

)

= 0 (2)

r = a+ b cosωt

r = −bω sinωt

(1) ⇒ (a+ b cosωt)··

φ sin θ−2bω sinωt·

θ−(a+ b cosωt)·

φ2

sin θ cos θ−g sin θ = 0

(2) ⇒ (a+ b cosωt)··

φ sin θ−2bω sinωt sin θ·

φ+2 (a+ b cosωt)·

θ·

φ cos θ = 0.

These are differential equation of the motion.

Generalized forces :

Let us assume that a given set of force F1, F2, · · · , , F3N is applied

to system of N particles.

17

Page 18: MSc Maths Optional Paper V

The virtual workdone by these force is

δW =3N∑

j=1

Fjδxj (1)

Let us suppose that the 3N ordinary constraint co-ordinates are

related with the n - generalized co-ordinates q1, q2, · · · , qn.Let these are related by the relations

xj = xj(q1, q2, · · · , qn, t), j = 1, 2, 3, · · · , 3N.

Using displacement is virtual workdone we get,

dxj =∂xj

∂q1dq1 +

∂xj

∂q2dq2 + · · · + ∂xj

∂qndqn +

∂xj

∂tdt

δxj =∂xj

∂q1δq1 +

∂xj

∂q2δq2 + · · · + ∂xj

∂qnδqn

δxj =n∑

i=1

∂xj

∂qiδqi (j = 1, 2, · · · , 3N)

(1) ⇒ δW =3N∑

j=1

Fj

(

n∑

i=1

∂xj

∂qiδqi

)

=n∑

i=1

(

3N∑

j=1

Fj

∂xj

∂qi

)

δqi

3N∑

j=1

Fj

∂xj

∂qiis called the generalized force.

δW =n∑

i=1

Qiδqi.

Principle of work and kinetic evergy :

Statement:

The increase in K.E. of a particle as its moves from one arbitary point

to another point is equal to the workdone by the force acting on the

particle during the given intervals (or) chang in K.E. = workdone.

Proof:

Kinetic Energy is given by T =1

2mv2 v is the velocity of particle and

18

Page 19: MSc Maths Optional Paper V

m is the mass of the particle.

Let W be the workdone by the force−→F in moving particle from A

to B is

W =

B∫

A

−→F · d−→r

=

B∫

A

m··−→r d−→r , ∵

−→F = m

··−→r

= m

B∫

A

··−→r d−→r

W =1

2m

B∫

A

d

dt

(

·−→r·−→r)

dt

=1

2m

B∫

A

d

(

·−→r)2

=m

2

B∫

A

d

(

·−→r)2

=m

2

B∫

A

d(

v2)

=m

2

(

v2)B

A

=m

2

(

v2B − v2

A

)

=1

2mv2

B − 1

2mv2

A

W = TB − TA

Book work : State and prove the principle of conservation of

energy.

Statement: Let the only force acting on the given particle be conser-

vative.

Proof : Let−→F be the conservative force acting on the particle at the

19

Page 20: MSc Maths Optional Paper V

point (x, y, z).

m is the mass of the particle.

T is the K.E. of the particle.

W is the workdone of the particle, from A to B

W = VA − VB = TA − TB

Since the points A and B are arbitrary.

TA +VA = TB +TB = E of the energy is conservative. We conclude

that the total mechanical evergy E remains constant during the motion

of the particle.

This is the principle of conservation of evergy.

Book work :

An equilibrium configuration at a conservative holonomic system

with workless fixed constraints must occur at a position where the P.E.

has a stationary value.

Proof :

consider a system of N particles whose applied forces are conser-

vative and derived from the P.E. function V (x1, x2, · · · , x3N).

The virtual workdone by the forces

δW =3N∑

j=1

Fxjδxj

But Fxj =−∂V∂xj

Workdone δW =3N∑

j=1

(−∂V∂qi

)

δxj = −δV.

This is the first variation of the P.E.

By principle of virtual work the necessary condition for the static

equilibrium of the system is δW = 0, δV = 0.

∂V

∂xj

= 0, j = 1, 2, 3, · · · , 3N.

20

Page 21: MSc Maths Optional Paper V

Let us assume the P.E is interms of generalized co-ordinates

V = V (q1, q2, · · · , qn)

δV =n∑

i=1

∂V

∂xi

(δqi) , q′s are independent

We conclude that an equilibrium configuration of a conservative

holonomic system with workless fixed constraints must occur at a po-

sition.

Where the P.E has a stationary value. Consider the stability of

this system at position of static equilibrium.

∴ δV = 0

⇒ ∂V

∂qi= 0, i = 1, 2, 3, · · · , n.

These condition imply that the P.E. is at a stationary value system.

Expanding the P.E. function V (q1, q2, · · · , qn) about the reference

value V0, we have

V = V0 +

(

∂V

∂q1

)

0

δq1 +

(

∂V

∂q2

)

0

δq2 + · · ·

+1

2

(

∂2V

∂q21

)

(δq1)2 + · · · +

(

∂2V

∂q1∂q2

)

(δq1δq2) + · · ·

∴ ∆V is the change in P.E. from its value of equation position.

∆V = V − V0

=1

2

(

∂2V

∂q21

)

(

δ2q1)

+

(

∂2V

∂q1∂q2

)

(δq1δq2) +1

2

(

∂2V

∂q22

)

(

δ2q2)

+ · · ·

where a zero subscript on a function implices that it is to be evalu-

ated at the reference value of the q’s. The δq’s represent infinitesimal

change from this reference configuration.

(i) If ∆V > 0 ⇒ The equilibrium is stable

(ii) If ∆V < 0 ⇒ The equilibrium is unstable

21

Page 22: MSc Maths Optional Paper V

(iii) If ∆V = 0 ⇒ The equilibrium is nutral.

K..onig’s Theorem :

The total K.E. of a system is equal to the sum of

(i) The K.E. due to a particle having a mass equal to the total mass

of the system and moving with the velocity of the centre of mass

and

(ii) the K.E. due to the motion the system relative to its centre of

mass.

Find the K.E. of the system of particle.

Proof :

Consider a system of N particles.

Let −→ri be the position vector of the ith particle (whose mass is no)

relative to a point O fixed in an inertial frame.

jm

im

yO

irr

im

.c m

Crr

Crr

x

z

Let −→rC be the position vector of the mass centre om.

Let mi be the mass of the system.

Let T be the K.E. of the system.

T =N∑

i=1

1

2mi

·−→r2

·−→r =·−→r C +

·−→ρi

22

Page 23: MSc Maths Optional Paper V

·−→r2

=·−→r

2

C + −→ρi2 + 2

·−→rC

·−→ρi

T =1

2

(

N∑

i=1

mi

)

·−→r2

C +1

2

N∑

i=1

mi

·−→ρ i

2

+N∑

i=1

mi

(

·−→r C

·−→ρ i

)

=1

2

·−→r2

C

N∑

i=1

mi +1

2

N∑

i=1

mi

·−→ρ i

2

+·−→r C

N∑

i=1

mi

·−→ρ i

=1

2

·−→r2

C

N∑

i=1

mi +1

2

N∑

i=1

mi

·−→ρ i

2[

N∑

i=1

mi

·−→ρ i = 0

]

T =1

2m

·−→r2

C +1

2

N∑

i=1

mi

·−→ρ i

2

T = K.E of mass centre + K.E. relative to the centre of mass.

K.E. of a particle of mass m with vector of c.m.(centre of mass)

Book work :

Kinetic Energy of a system of a particle using an arbitrary reference

point.

Proof :

Consider a system of N - particle. Let −→ri be the position vector

of an ith particle (mass m) relative to a point O fixed in an inertial

frame.

Let P be an arbitrary point. −→rC is the position vector of mass

centre c.m.

km

y

O

irr

im .c m

P

Crr

irr

Crr

z

23

Page 24: MSc Maths Optional Paper V

−→ρc is the position vector of mass centre relative to the point P .

Let T be the kinetic energy of the system.

T =N∑

i=1

1

2mi

·−→r2

·−→r =·−→r ρ +

·−→ρi

·−→r2

i =

(

·−→r ρ +·−→ρi

)2

=·−→r

2

ρ +

·

−→ρi2 + 2

·−→r ρ

·−→ρi

T =1

2

N∑

i=1

mi

·−→r2

ρ +1

2

N∑

i=1

mi

·−→ρ i

2

+N∑

i=1

mi

·−→r ρ

·−→ρ i

=1

2

N∑

i=1

mi

·−→ρ i

2

+1

2

·−→r2

ρm+·−→r ρ

N∑

i=1

mi

·−→ρ i

T =1

2m

·−→r2

ρ +1

2

N∑

i=1

mi

·−→ρ i

2

+·−→r ρm

·−→ρ C

[

ρC =1

m

mi

·−→ρ C

]

Hence the total K.E. is the sum of this 3 parts.

1. The K.E. due to the particle having a mass and moving with

reference point P .

2. The K.E. of the system due to its motion relative to P .

3. The sclar product of the velocity of the reference point and the

linear momentum of the system relative to a reference point.

Note :

ρi = ρC + ρj

·

ρi =·

ρC +·

ρj∑

ρi =∑

ρC +∑

ρj

But∑

mρj = 0,∑

ρi =∑

ρC

24

Page 25: MSc Maths Optional Paper V

Kinetic Energy of rigid body :

Take CM as the origin of vectors. Let dV be a small volume element

of a rigid body with density.

Each element of the rigid body is having in general translational

and rotational velocity.

We can assume that the rigid body is having instantaneous axis of

rotaion. The typical volume element dV can be chosen so small.

ie, rotational K.E is negligibe compared with its translational K.E

Each element of a rigid body can be considered as a particle of

infinite small mass.

m = V × ρ

Hence the limiting case of K.E of system of N - particles.

Then the K.E of the system

T =1

2m

·−→r2

C +1

2

N∑

i=1

mi

·−→ρ2

i

=1

2m

·−→r2

C +1

2

V

ρ·−→ρ

2

i dV

−→ρ is the position vector of volume element dV relative to mass

centre c.m.

m is the mass of the body. The first term is called translational

K.E. and second term is called rotational K.E.

Let assume that the body be rotating with angular velocity −→ω .

To find

·−→ρ = −→ω ×−→ρ·−→ρ

2

= (−→ω ×−→ρ )2

= −→ω · −→ρ ×·−→ρ

= (−→ω ×−→ρ ) ··−→ρ = −→ω ·

[

−→ρ(

−→ω ×·−→ρ)]

·−→ρ2

= −→ω · [(−→ρ · −→ρ )−→ω − (−→ρ · −→ω )−→ρ ]

25

Page 26: MSc Maths Optional Paper V

= −→ω ·[−→ρ 2−→ω − (−→ρ · −→ω )−→ρ

]

·−→ρ2

= −→ω ·[−→ρ 2−→ω − (−→ρ · −→ω )−→ρ

]

Let ~ρ = x~i+ y~j + z~k ~ω = ωx~i+ ωy

~j + ωz~k

~ω ·[

~ρ2~ω − (~ρ · ~ω)~ρ]

=∑

[

(

x2 + y2 + z2)

ωx~i− (xωx + yωy + zωz)

·

~xi

]

ω

=∑

[(

y2 + z2)

ωx − xyωy − xzωz

]

=

[(

y2 + z2)

ωx − (xyωy + xzωz)]

i+ [ ]j + [ ]k

ω

T =1

2

V

−→ρ.−→ρ 2dV

=1

2~ω

V

−→ρ([(

y2 + z2)

ωx − (xyωy + xzωz)]

i

+ [ ]j + [ ]k

)

ω

dV

Define m.I and product of inertial as follows. Let

Ixx =

V

(

y2 + z2)−→ρ dV

Iyy =

V

(

z2 + x2)−→ρ dV

Izz =

V

(

x2 + x2)−→ρ dV

Ixy = Iyx = −∫

V

(xy)−→ρ dV

Iyz = Izy = −∫

V

(yz)−→ρ dV

26

Page 27: MSc Maths Optional Paper V

Izx = Ixz = −∫

V

(zx)−→ρ dV

Trot =1

2Ixxω

2x +

1

2Iyyω

2y +

1

2Izzω

2z + Ixyωxωy + Iyyωyωz + Izxωzωx

=3∑

i=1

3∑

j=1

1

2Iijωiωj

=1

2−→ω T I−→ω

=1

2

ωx

ωy

ωz

T

Ixx Ixy Ixz

Iyx Iyy Iyz

Izx Izy Izz

ωx

ωy

ωz

=1

2(−→ω )

T −→I (−→ω )

Kinetic Enery of rigid body = 1

2m

·−→rc + −→ω T I−→ωAngular Momentum of a system :

The angular momentum of a system of N particles and the total

mass ‘M ’ about a fixed point O is equal to the angular momentum

about θ of a single particle of mass m which is moving with centre of

mass plus the angular momentum of the system about the centre of

mass m.

Find the angular momentum of a system of particles about a fixed

point θ.

moment of momentum = Angular momentum.

Proof :

Let us consider a system of N - particles Let mi is the mass of the

ith particle whole position vectors given by −→ri .

The angular momentum of the ithparticle = r × ρ

= r ×mv

27

Page 28: MSc Maths Optional Paper V

= m·−→r × r

AM = −→ri ×m·−→r

Angular momentum of the system H =N∑

i=1

−→ri ×m·−→r .

ButN∑

i=1

mi−→ρi = 0 ⇒

N∑

i=1

i−→ρi = 0

Angular momentum of the system

=

(

×·−→rC

)

m+ 0 + 0 +N∑

i=1

−→ρi ×m·−→ρ

= −→rC ×m·−→rC +

N∑

i=1

−→ρi ×m·−→ρ

H =

(

−→rC ×m·−→rC

)

+HC

wher HC is the angular momentum of the system relative to a

centre of mass.

Note :

Tra =1

2−→ω∫

η−→ρ × (−→ω ×−→ρ ) dV

=1

2−→ω−→HC

Angular momenum of a rigid body :

Le dV be small volume element of a rigid body having density η.

Each element of the rigid body is having translation and relation.

We can assume that the rigid body is having instantaneous axis of

rotation.

The typical volume element dV can be chosen so small.

28

Page 29: MSc Maths Optional Paper V

Its rotational K.E is negligible compared with its translational K.E

each element of rigid body can be considered as particle of infinite

decimal mass.

The angular momentum of a systme of particle−→H = −→rC ×m

·−→rC +N∑

i=1

−→ρi ×m·−→ρ C .

In the case of rigid body

−→H = −→rC ×m

·−→rC +

V

η

(

−→ρ ×·−→ρ)

dV

But·−→ρ = −→ω ×−→ρ , where −→ω is the angular velocity of the rigid body.

∴−→ρ ×

·−→ρ = −→ρ × (−→ω ×−→ρ ) = −→ρ 2−→ω − (−→ρ · −→ω )−→ρ

Let us consider a cartesian system with its origin at the centre of mass.

−→ρ i = x−→i + y

−→j + z

−→k , −→ω = ωx

−→i + ωy

−→j + ωz

−→k

~ρ2~ω· (~ω)~ρ =∑

(

x2 + y2 + z2)

ωx − (xωx + yωy + zωz)−→xI

=∑

(

y2 + z2)

ωx− (xyωy + xzωz)−→i

=[

(

y2 + z2)

ωx − (xyωy + xzωz)]

i

Ixx =

V

(

y2 + z2)

ηdV

Iyy =

V

(

z2 + x2)

ηdV

Izz =

V

(

x2 + y2)

ηdV

Ixy = Iyx = −∫

V

(xy) ηdV

Iyz = Izy = −∫

V

(yz) ηdV

Izx = Ixz = −∫

V

(zx) ηdV

29

Page 30: MSc Maths Optional Paper V

V

η

(

−→ρ ×··−→ρ)

dV

=

V

η[−→ρ 2−→ω − (−→ρ · −→ω )−→ρ dV

]

=

V

[(

ηy2 + z2)

ωx − ηxyωy − ηxzωz

]−→i

+[

η(

z2 + x2)

ωy−ηyzωx − ηyxωx

]−→j

+[

η(

x2 + y2)

ωx−ηzxωy − ηzyωy

]−→k

dV

= Ixxωx

−→i + Iyyωy

−→j + Izzωz

−→k + Ixyωy

−→i

+Iyzωz

−→j + Izxωx

−→k + Ixzωz

−→i + Iyxωx

−→j + Izyωy

−→k

−→HC = (Ixxωx + Ixzωz + Ixyωy)

−→i + (Iyyωy + Iyzωz + Iyxωx)

−→j

+ (Izzωz + Izxωx + Izyωy)−→k

−→HC = I−→ω where I =

Ixx Ixy Ixz

Iyx Iyy Iyz

Izx Izy Izz

−→H = −→rC ×m

·−→rC +−→HC where

−→HC = I−→ω .

Problem:

Prove that Trot =1

2−→ω · −→HC .

Proof:

The rotational K.E of a rigid body M given by

T =1

2

V

η·−→ρ dV (1)

η is the density of the rigid body, −→ρ is the position of the rigid

body is the position vector of elementary volume dv with respect to

centre of mass.

Let −→ω be the angular velocity of the body. Let −→rC be the position

30

Page 31: MSc Maths Optional Paper V

vector of centre of mass w.r. to origin ‘O’. Then−→H =

(

−→rC ×m·−→rC

)

+

−→HC

where−→HC =

V

η (−→ρ × (−→ω ×−→ρ )) dV (2)

But

·−→ρ2

=·−→ρ

·

·−→ρ =·−→ρ · (−→ω ×−→ρ )

= (−→ω ×−→ρ ) ··−→ρ

= −→ω ·(

−→ρ ×·−→ρ)

= −→ω ·[

−→ρ ×(

·−→ρ ×−→ω)]

Trot =1

2

V

·−→ρ2

ηdV

=1

2

V

η−→ω [−→ρ × (−→ω ×−→ρ )] dV

=1

2−→ω · −→HC ( using (2))

Angular momentum with respect to an arbitrary point :

Let P be arbitrary point, −→rP is the position vector of P w.r. to O,

mi is the position of the ith particle with position vector −→ri , c.m. is

the centre of mass of the system, −→ρi is the position vector of the ith

particle w.r. to ‘P ’.

Angular momentum of the ith particle w.r.to p = ρimi

·−→ρi .

Angular moment of the system about the position P is

−→HP =

N∑

i=1

−→ρi ×mi

·−→ρi

−→ρi = −→ri −−→rP

31

Page 32: MSc Maths Optional Paper V

But

−→rP = −→rC−−→ρ

C

−→ρi = −→ri −(−→r

C−−→ρC

)

= −→ri −−→rC

+ −→ρC

·−→ρi =·−→ri −

·−→rC

+·−→ρC

−→HP =

N∑

i=1

(−→ri −−→rC

+ −→ρC

)

×mi

(

·−→ri −·−→rC

+·−→ρC

)

=N∑

i=1

mi

(

−→ri ×·−→ri −−→ri ×

·−→rC

+ −→ri ×·−→ρC−−→r

·−→ri +−→rC ×

·−→rC −−→rC ×·−→ρC

+ −→ρC×

·−→ri −−→ρC×

·−→rC

+ −→ρC×

·−→ρC

)

AM of the system about the arbitrary point

=N∑

i=1

−→ri ×mi

·−→ri −N∑

i=1

mi−→ri ×

·−→rC

+N∑

i=1

mi−→ri ×

.−→ρC

−−→rC×

N∑

i=1

mi

·−→ri + −→rC ×m·−→rC −−→rC ×m

·−→ρC

+−→ρC×

N∑

i=1

mi

·−→ρi −−→ρC×m

·−→rC

+ −→ρC×m

·−→ρC

But

−→r C =

N∑

i=1

mi−→ri

N∑

i=1

mi

⇒N∑

i=1

mi−→r C =

N∑

i=1

mi−→ri

·−→r C =N∑

i=1

mi

·−→ri

32

Page 33: MSc Maths Optional Paper V

AM of the system about the arbitrary point

−→HP =

−→H ·m−→r C ×

·−→r C +m−→r C ×·−→ρ C −−→r C ×m

·−→r C + −→r C ×m·−→r C

−−→r C ×m·−→ρ C + −→ρ C ×m

·−→r C −−→ρ C ×m·−→r C +

·−→ρ C ×m·−→ρ C

=−→H −−→r C ×m

·−→r C + −→ρ C ×m·−→ρ C

=N∑

i=1

−→r i ×mi

·−→r i −−→r C ×m·−→r C + −→ρ C ×m

·−→ρ C

1. Considering the K.E of a system of N particles show with usual

notation that

T =1

2m

·−→r 2C +

1

2

N∑

i=1

mi

·−→ρ 2i

2. Prove with usual notation that

Trot =1

2−→ω−→HC

3. Obtain the expression for the rational K.E. (Rigid body on sys-

tem of particle)

Generalized Momentum :

Consider a system with n - generalized co-ordinates q1, q2, · · · , qn.Let T be K.E of the system. V is the potential energy of the

system. Let us defined the Lagrangian function L(

q,·

q, t)

as follows

L = T − V .

The generalized momentum pi associated with qi is defined as

pi =∂L

∂·

qi

=∂T

∂·

qi

[

[

∂V

∂·

qi

]

= 0

]

.

Note : i) L is almost a function of quadratic in q′s. Hence pi is a

linear function of q′s. If V = V (q, t), then∂L

∂qi=∂T

∂qi− ∂V

∂qi.

ii) Consider a particle of mass in with cortesian co-ordinates (x, y, z).

33

Page 34: MSc Maths Optional Paper V

The K.E of the particle is given by

T =1

2m(

·

x2

y2

z2)

px =∂T

∂·

x=

1

2m2

·

x = m·

x

Similarly py = m·

y and

pz = m·

z

px is the x component of the linear momentum. If the position of the

particle is given by spherical polar co-ordinates (r, θ, φ) ,then

T =1

2m

(

·

r2

+ r2·

θ2

+ r2·

φ2

sin2 θ

)

pr =∂T

∂·

r=

1

2m2

·

r = m·

r

pθ =∂T

∂·

θ=

1

2mr22

·

θ = mr2·

θ

pφ =∂T

∂·

φ=

1

2mr22

·

φ sin2 θ = mr2 sin2 θ·

φ

pris called linerar momentum component radial direction.

pθ is called horizandal component of hte angular momentum.

pφ is called vertical component of angular momentum.

1. Three particles are conneeted by 2 rigid rods having a joint be-

tween them to form a system verticla force F and a momentum

M are applied as shown in figure. The configuration of the sys-

tem is given by the ordinary co-ordinates (x1, x2, x3) or by the

generalized co-ordinates (q1, q2, q3) where x1 = q1 +q2 + 1

2q3;x2 =

q1 − q3;x3 = q1 − q2 + 1

2q3. Find the expression for K.E and gen-

eralized momentum.

Solution :

Let T be the K.E. of the system.

∴ T =1

2m(

·

x2

1 +·

x2

2 +·

x2

3

)

34

Page 35: MSc Maths Optional Paper V

x1 = q1 + q2 +1

2q3

x2 = q1 − q3

x3 = q1 − q2 +1

2q3

·

x1 =·

q1 +·

q2 +1

2

·

q3

·

x2 =·

q1 −·

q2

·

x3 =·

q1 −·

q2 +1

2

·

q3

∴ T =1

2m

(

q2

1 + 2·

q2

2 +3

2

·

q2

3

)

p1 =∂T

∂·

q1

=1

2m3(

q1

)

= 3m·

q1

p2 =∂T

∂·

q2

=1

2m4

·

q2 = 2m·

q2

p3 =∂T

∂·

q3

=1

2m

3

22·

q3 =3m

2

·

q3

p1, p2, p3 are called the generalized moment equal are inertial co-efficeient.

2 Three particles are connected by two rigid rods having a joint

between them to form the system shown in the figure. A virtual

force F and momentum m are applied to the system as shown the

configuration of the system is given by the ordinary co-ordinates

(x1, x

2, x

3) or by the gneralized co-ordinates (q

1, q

2, q

3), where

x1 = q1 + q2 + 1

2q3;x2 = q1 − q3;x3 = q1 − q2 + 1

2q3. Find the

generalized force Q1 ,Q2,Q

3

l l

mm

m M3

4

l

2x 3

x1

x

Solution :

Assume small motions the applied force on the system F1 =3F

4, F2 =

F

4− m

ℓ, F3 =

m

ℓ.

35

Page 36: MSc Maths Optional Paper V

Let Q1, Q2, Q3 be the corresponding generalized forces on the sys-

tem.

Qi =3N∑

j=1

Fj

∂xj

∂qi⇒

3N∑

j=1

Fj

∂xj

∂qi

Q1 = F1

∂x1

∂q1+ F2

∂x2

∂q1+ F2

∂x3

∂q1

=3F

4(1) +

(

F

4− m

)

+m

=3F

4+F

4= F

Q2 = F1

∂x1

∂q2+ F2

∂x2

∂q2+ F2

∂x3

∂q2

=3F

4(1) +

(

F

4− m

)

(0) +m

ℓ(−1)

=3F

4− m

Q1 = F1

∂x1

∂q1+ F2

∂x2

∂q1+ F2

∂x3

∂q1

=3F

4

(

1

2

)

+

(

F

4− m

)

(−1) +m

(

1

2

)

=3F

8− F

4+m

ℓ+m

2ℓ

=F

8+

3m

2ℓ

Thus Q1 = F,Q2 = 3F4− m

ℓand Q3 = F

8+ 3m

2ℓ.

Energy of momentum conservative systems :

Let (x, y, z) be the position of a particle. Let−→F be the total force

acting on the partical with components

Fx = −∂V∂x

, Fy = −∂V∂y

, Fz = −∂V∂τ

where V (x, y, z) is the potential energy function depending on position

only. In this case F is called conservative force.

Book work : Prove that for any constervative force

∮ −→F × d−→r = 0

Proof : Let−→F be the conservative force acting on the particele at

36

Page 37: MSc Maths Optional Paper V

(x, y, z). Let V (x, y, z) be the potential energy of the particle.

Fx = −∂V∂x

, Fy = −∂V∂y

, Fz = −∂V∂y

Workdone =(

Fx

−→i + Fy

−→j + Fz

−→k)

· d(

x−→i + y

−→j + z

−→k)

W =

Fdr

dW =−→F · dr = Fxdx+ Fydy + Fzdz

=

(

−∂V∂x

)

dx+

(

−∂V∂y

)

dy +

(

−∂V∂z

)

dz

= −(

∂V

∂xdx+

∂V

∂ydy +

∂V

∂zdz

)

dW = −dV (x, y, z)

Thus dW is an exact differential equation

−→F · d−→r = −dV

Let W be the workdone by the force−→F is the moving particle from

A to B.

Total workdone = W =

B∫

A

− dV = −B∫

A

dV = − [V ]BA = VA − VB

∴ Workdone on the particle is depending on the initial and final

positions.

Workdone is moving around a closed curve is

∮ −→F d−→r = 0

[

√A =

√B]

37

Page 38: MSc Maths Optional Paper V

UNIT II

Lagrange’s Equation :

Let us consider a system of N particles whose positions relative to

an inertial frame are given by the cartesian co-ordinates x1, x2, · · · , x3N .

Kinetic energy of the system

T =1

2

3N∑

k=1

mk

·

x2

k (1)

where m1 = m2 = m3,m4 = m5 = m6 is the mass of the first particle,

second particle respectively.

Let q1, q2, · · · , qn be the generalized co-ordinates of the system.xk =

xk (q, t) , k = 1, 2, 3, · · · , 3N where we assume that tehse functions are

twice differentiable with respect to the q′s and t. We find that

·

xk

(

q,·

q, t)

=n∑

i=1

∂xk

∂qi

·

qi +∂xk

∂t

·

xk =∂xk

∂q1

·

q1 +∂xk

∂q2

·

q2 + · · · + ∂xk

∂qn

·

qn +∂xk

∂t

·

xk =n∑

i=1

∂xk

∂qi

·

qi +∂xk

∂t

Put in (1), we get

∴ T =1

2

3N∑

k=1

mk

[

n∑

i=1

∂xk

∂qi

·

qi +∂xk

∂t

]2

=1

2

3N∑

k=1

mk

[

n∑

i=1

n∑

j=1

∂xk

∂qi

∂xk

∂qj

·

qi

·

qj + 2n∑

i=1

∂xk

∂qi

·

qi

∂xk

∂t+

(

∂xk

∂t

)2]

=1

2

n∑

i=1

n∑

j=1

[

3N∑

k=1

mk

∂xk

∂qi

∂xk

∂qj

]

·

qi

·

qj +n∑

i=1

[

3N∑

k=1

mk

∂xk

∂qi

∂xk

∂t

]

·

qi

+1

2

3N∑

k=1

mk

(

∂xk

∂t

)2

.

38

Page 39: MSc Maths Optional Paper V

Let3N∑

k=1

mk

∂xk

∂qi

∂xk

∂qj= mij and

3N∑

k=1

mk

∂xk

∂qi

∂xk

∂t= ai

T =1

2

n∑

i=1

n∑

j=1

mij

·

qi

·

qj +n∑

i=1

ai

·

qi +1

2

3N∑

k=1

mk

(

∂xk

∂t

)2

T(

q,·

q, t)

= T2 + T1 + T0.

where T2 is a homogeneous quadratic in q′s, T1 is homogeneous linear

function in·

qi, T0 is a function of q′s and t′s in general.

Note:

(i) T is a functin of(

q,·

q, t)

.

∴ T = T(

q,·

q, t)

(ii) For a sceleronomic system T0 = 0

T1 = 0

since∂xk

∂t= 0

In this case T = T2.

2. Obtain Lagrange’s equatin from D’ Alembert’s principle interms

of generalized co-ordinates.

Proof :

Let us assume that the system has N particles with 3N cartesian

co-ordinates from D’ alembert’s principle, we have

3N∑

k=1

(

Fk −mk

··

xk

)

δxk = 0 (1)

where Fk is the applied force component associated with xk.

39

Page 40: MSc Maths Optional Paper V

Let us assume that the system is determined by n-generalized co-

ordinates q1, q2, · · · , qn.

xk = xk (q, t)

xk = xk(q1, q2, · · · , qn, t)

δxk =n∑

i=1

∂xk

∂qiδqi +

∂xk

∂tδt

(1) ⇒3N∑

k=1

(

Fk −mk

··

xk

)

n∑

i=1

∂xk

∂qiδqi = 0

n∑

i=1

3N∑

k=1

(

Fk −mk

··

xk

) ∂xk

∂qiδqi = 0 (2)

·

xk =n∑

i=1

(

∂xk

∂qi

·

qi

)

Differentiating w.r.to·

qi

∂·

xk

∂·

qi

=∂xk

∂qi(3)

d

dt

(

∂·

xk

∂·

qi

)

=d

dt

(

∂xk

∂qi

)

=∂

·

xk

∂qi

T =1

2

3N∑

k=1

mk

·

x2

k

pi =∂T

∂·

qi

=1

2

3N∑

k=1

mk 2·

xk

∂·

xk

∂qi

=3N∑

k=1

mk

·

xk

∂·

xk

∂qi

d

dt

(

∂T

∂·

qi

)

=3N∑

k=1

[

mk

··

xk

∂·

xk

∂qi+mk

·

xk

d

dt

(

∂·

xk

∂qi

)]

40

Page 41: MSc Maths Optional Paper V

=3N∑

k=1

mk

∂xk

∂qi

··

xk +3N∑

k=1

mk

·

xk

∂·

xk

∂qi(4)

∂T

∂qi=

1

2

3N∑

k=1

mk 2·

xk

∂·

xk

∂qi(5)

d

dt

(

∂T

∂qi

)

− ∂T

∂qi=

3N∑

k=1

mk

··

xk

∂xk

∂qi(6)

Let us define the generalized force Qi asq

Qi =3N∑

k=1

Fk

∂xk

∂qi

By (2) we have

n∑

i=1

[

3N∑

k=1

Fk

∂xk

∂qi−

3N∑

k=1

mk

··

xk

∂xk

∂qi

]

δqi = 0

n∑

i=1

[

Qi −d

dt

(

∂T

∂·

qi

)

+∂T

∂qi

]

δqi = 0 (7)

This is the Lagrange’s form of D’ Alemberts principle interms of gen-

eralized co-ordinates.

The above discussion the system is under instantaneous constraints.

Let us assume the system be holonomic and described by n- inde-

pendet generalized co-ordinates q1, q2, · · · , qn.The δq′is are independent.

Hence the co-efficeient of δq’s in equation (7) must be zero. q’s are

independent.

∴ co-efficient of δq = 0

d

dt

(

∂T

∂·

qi

)

− ∂T

∂qi= Qi, i = 1, 2, 3, · · · , n.

These equations are Lagrange equations from holonomic system.

Book work : Obtain the standard form of Lagrange’s equation for a

41

Page 42: MSc Maths Optional Paper V

holonomic system.

Proof : Let us assume that all the generalized forces are derivable form

a potential function V (q, t) as let the system is holonomic conservative.

Qi = −dVdqi

=d

dt

(

∂T

∂·

qi

)

− ∂T

∂qi= −∂V

∂qi

d

dt

(

∂T

∂·

qi

)

− ∂T

∂qi+∂V

∂qi= 0

d

dt

(

∂T

∂·

qi

)

− ∂

∂qi(T − V ) = 0

Let L = T − V (Lagarangian function) T = L+ V.

∂T

∂·

qi

=∂L

∂·

qi

[

∵ L(

q,·

q, t)]

d

dt

(

∂T

∂·

qi

)

− ∂L

∂·

qi

= 0, i = 1, 2, 3, · · · , N.

Note :

1. The standard form of Lagrange’s equations

d

dt

(

∂T

∂·

qi

)

− ∂L

∂qi= 0, i = 1, 2, 3, · · · , N.

since pi =∂L

∂·

qi

d

dt(Pi) −

∂L

∂qi= 0

where pi =∂L

∂qi, i = 1, 2, 3, · · · , n.

2. Let us assume the generalized forces Qi is not completly deriva-

42

Page 43: MSc Maths Optional Paper V

tive from a potential function

Qi = −∂V∂qi

+Q′

i

we haved

dt

(

∂T

∂·

qi

)

− ∂T

∂qi= Qi

d

dt

(

∂T

∂·

qi

)

− ∂T

∂qi= −∂V

∂·

qi

+Q′

i

d

dt

(

∂T

∂·

qi

)

− ∂L

∂·

qi

+∂T

∂qi= Q′

i

Let L = T − V.

d

dt

(

∂T

∂qi

)

− ∂L

∂·

qi

= Q′

i, i = 1, 2, 3, · · · , n.

Book work : Obtain the Lagrange’s equation for a non-holonomic

system.

Proof : Let us assume that system has ‘m’ non holonomic constraint

equationn∑

i=1

ajidqi + ajidt = 0 (j = 1, 2, 3, · · · ,m) .

Let us assume a virtual displacement consistent with the con-

straints.

Now we have

n∑

i=1

aijδqi = 0, j = 1, 2, 3, · · · ,m (1)

The constraints are assumed to be workless. Hence we have,

n∑

i=1

ciδqi = 0 (2)

43

Page 44: MSc Maths Optional Paper V

where ci is the generalized constraints force

Multiply equation (1) by Lagrange’s multiply λj

λj

n∑

i=1

ajiδqi = 0, j = 1, 2, 3, · · · ,m (3)

Adding these m equations and subtracting from (2) we get

(2) − (3) ⇒n∑

i=1

(

ci −m∑

j=1

λjaji

)

δqi = 0

Choose the λ′s arbitrary such that

ci =m∑

j=1

λjaji, i = 1, 2, 3, · · · ,m

of the generalized forces are not wholly derivable form a potential

function.

QQi = −∂V∂qi

+Q′

i;

we haved

dt

(

∂L

∂ξi

)

− ∂L

∂ξi

= Q′

i, i = 1, 2, 3, · · · , n

In our discussion Q′

i nothing but the ci

d

dt

(

∂L

∂·

qi

)

− ∂L

∂qi=

m∑

j=1

λjaji, i = 1, 2, 3, · · · , n.

This is the standard form of Lagrange’s equation for the non-holonomic

systyem.

1. Find the differential equation of motion for a spherical pendulum

of length ℓ (using Lagrange’s equation).

Solution : Let use the spherical co-ordinates θ and φ. θ is mea-

sured from the upward vertical, φ is the angle between a vertical plane

through supporting point ‘O’ and a vertical plane containing the pen-

44

Page 45: MSc Maths Optional Paper V

dulum.

The spherical co-ordinates are (r, θ, φ) for this problem.

r = ℓ (constant) ⇒ r′ = 0

Let T be the K.E. and V is the P.E.

T =1

2m

(

ℓ2·

θ2

+ ℓ2·

φ2

sin2 θ

)

V = −mg (−ℓ cos θ) = mgℓ cos θ

L = T − V

=1

2m

(

ℓ2·

θ2

+ ℓ2·

φ2

sin2 θ

)

−mgℓ cos θ

The Lagrange’s equation is given byd

dt

(

∂L

∂·

qi

)

− ∂L

∂qi= 0. For our

problem, qi = ℓ, q2 = 0, q3 = φ, we have

d

dt

(

∂L

∂·

θ

)

− ∂L

∂θ= 0

d

dt

(

1

2mℓ22

·

θ

)

−[

1

2mℓ2

·

φ22 sin θ cos θ +mgℓ sin θ

]

= 0

mℓ2··

θ −mℓ2·

φ2 sin θ cos θ −mgℓ sin θ = 0

Dividing by mℓ

ℓ··

θ − ℓ·

φ2 sin θ cos θ − g sin θ = 0 (1)

q

f

mg

sinl q

cosl ql

O

180 q-

45

Page 46: MSc Maths Optional Paper V

Also

d

dt

(

∂L

∂·

φ

)

− ∂L

∂φ= 0

d

dt

(

1

2mℓ22

·

φ sin2 θ

)

− 0 = 0

d

dt

(

mℓ2·

φ sin2 θ

)

= 0

Differentiating w.r.to t

mℓ2 sin2 θ··

φ+ 2m·

φℓ2 sin θ cos θ·

θ = 0

Dividing by mℓ

cosh l q=cosh

lq=

g

h

m

sin2 θ··

φ+ 2 sin θ cos θ·

φ·

θ = 0 (2)

sin θ··

φ+ 2 cos θ·

φ·

θ = 0

we have

∂L

∂φ= 0

d

dt

(

∂L

∂·

φ

)

= 0

46

Page 47: MSc Maths Optional Paper V

∂L

∂φ= pφ = constant

i.e., mℓ2 sin2 θ.·

φ = pφ = constant

where pφ is the angular momentum above a vertical axis flow the sup-

porting point.

Inertial Co-efficeient :

The co-efficient of the q is the expression for the generalized mo-

ments are known an inertial co-efficeient.

In the Book work 2, 3n, 2m,3m

2are inertial co-efficeient.

2 A doubel pendulum consists of particles supported by mass less

rods assuming that all motions take place in a vertical plane.

Find the differential equation of the motion linearise these equa-

tions assuming small motions.

Solution :

Let T be the K.E of the system and V is the potential energy of

the system.

Let L = T − V. Let V be the velocity of the particle.

V = The velocity of upper particle + velocity of the lower particle.

l

f&l

f&l

q&l

f q-

f q-q

O

q

m B

T =1

2mv2

1 +1

2mv2

2

v1 = ℓ·

θ

v2 =

ℓ2·

φ2 + ℓ2·

θ2

+ 2ℓ·

φℓ·

θ cos (φ− θ)

T =1

2mℓ2

·

θ2 +1

2m

[

ℓ2·

φ2 + ℓ2·

θ2

+ 2ℓ·

φℓ·

θ cos θ

]

47

Page 48: MSc Maths Optional Paper V

=1

2mℓ2

[

θ2 +·

φ2 + 2·

φ·

θ cos (φ− θ)

]

V = −mg [ℓ cos θ + ℓ cosφ+ ℓ cos θ]

= −mg (2 cos θℓ+ ℓ cosφ)

L = T − V

=1

2mℓ2

[

θ2 +·

φ2 + 2·

φ·

θ cos (φ− θ)

]

+mg (2 cos θ + ℓ cosφ)

l cosql

BA

C

std level

cosfl

cosql

( )1p mg

f

f

mg

∴The equation

d

dt

(

∂L

∂·

θ

)

− ∂L

∂θ= 0

d

dt

[

1

2mℓ22.2

·

θ +1

2mℓ2.2

·

φ cos (φ− θ)

]

−1

2mℓ22

·

θ·

φ − sin θ (φ− θ) (−1) + (−2mgℓ sin θ) = 0

48

Page 49: MSc Maths Optional Paper V

l

O

q

f

g

lm

mℓ2d

dt

[

θ +·

φ cos (φ− θ)

]

−mℓ2·

θ·

φ sin (φ− θ) + 2mgℓ sin θ = 0

mℓ2[

2··

θ +··

φ cos (φ− θ) +·

φ (− sin (φ− θ))

(

·

φ−·

θ

)

−mℓ2·

θ·

φ sin (φ− θ) + 2mgℓ sin θ

]

= 0

mℓ2[

2··

θ +··

φ cos (φ− θ) −·

φ2 sin (φ− θ)

]

+ 2mgℓ sin θ = 0 (1)

Now the equation

d

dt

(

∂L

∂φ

)

− ∂L

∂φ= 0

d

dt

1

2mℓ2

(

2φ+ 2θ cos (φ− θ))

− 1

2mℓ22θφ(− sin (φ− θ)) −mgℓ sinφ

= 0

mℓ2d

dt

[

φ+ θ cos (φ− θ)]

+mℓ2θφ sin(

φ− θ)

+mgℓ sinφ = 0

mℓ2[

φ+ θ cos (φ− θ) + θ2sin (φ− θ)

]

+mgℓ sinφ = 0 (2)

49

Page 50: MSc Maths Optional Paper V

(1) and (2) are the differential equation of motion linearise the

equation. Let us assume the system having small oscillations (θ, φ)

are small.

∴ sin θ = θ, sinφ = φ

cos (φ− θ) = 1, sin (φ− θ) = φ− θ

Neglecting higher order terms equation (1) becomes

mℓ2(

2θ + φ)

+ 2mgℓθ = 0

and equation (2) becomes

mℓ2(

θ + φ)

+mgℓφ = 0

3 A block of mass m2 can slide on another block of mass m1 which

in turn, slides on a horizontal surface, as shown in the figure.

Using x1 and x2 as co-ordinates, obtain the differential equations

of motion. Solve for the accelerations of the two blocks as they

move under the influence of gravity, assuming that all surfaces

are frictionless. Find the fore of interaction between the blocks.

Solution : Let x1 be the displacement of m1, let x2 be the dis-

placement of m2 relative to m1. Let v1 be the velocity of m1 and v2

be the velocity of m2 relative to m1.

T =1

2m1v

21 +

1

2m2v

22

v1 = x1

v2 =√

x21 + x2

2 − 2x1x2 cos 450

=

x21 + x2

2 −√

2x1x2

∴ T =1

2m1x

21 +

1

2m2

(

x1 + x2 −√

2x1x2

)

.

50

Page 51: MSc Maths Optional Paper V

2m

2x

1x

045

Let V be the P.E. of the system

V −m2gx2 sin 450 = − 1√2m2gx2

∴ L = T − V

=1

2m1x

21 +

1

2m2

(

x21 + x2

2 −√

2x1x2

)

+1√2m2gx2

x1− Lagrange’s equation is

d

dt

(

∂L

∂x1

)

−(

∂L

∂x1

)

= 0

d

dt

(

1

2m12x1 +

1

2m22x1

)

− 1√2m2x2 = 0

m1x1 +m2x1 −1√2m2x2 = 0

(m1 +m2) x1 −1√2m2x2 = 0 (1)

[width=6cm]m12

x2− Lagrange’s equation is

d

dt

(

∂L

∂x2

)

−(

∂L

∂x2

)

= 0

d

dt

(

1

2m12x2 −

1√2m2x1

)

− 1√2m2g = 0

m2x2 −1√2m2x1 =

1√2m2g (2)

51

Page 52: MSc Maths Optional Paper V

(1) + (2)1√2m2x2 −

1√2m2x1 =

1√2m2g

(1) ⇒ (m1 +m2) x1 =1√2

[

1√2m2x1 +m2g

]

(

m1 +m2 −m2

2

)

x1 =1√2

m2g√2

x1 =m2g

2m1 +m2

(1) ⇒ (m1 +m2)m2g

2m1 +m2

=1√2m2x2

x2 =√

2(m1 +m2) g

2m1 +m2

4 A particle of mass m can side without friction on the inside of a

small tube which is bent in the form of a circle of radius r, the

tube rotates about a vertical diameter with a constant angular

velocity ω, as shown in figure write the differential equation of

motion.

Solution :

Let ‘O’ be the centre of the tube and r be the radius of the tube.

Let T be the K.E of the particle of mass m,V is the P.E. of the

particle.

T =1

2m(

r2θ2+ r2ω2 sin2 θ

)

=1

2mr2

(

θ2+ ω2 sin2 θ

)

V = mgh = mgr cos θ

L = T − V

=1

2mr2

(

θ2+ ω2 sin2 θ

)

−mgr cos θ, r is constant

52

Page 53: MSc Maths Optional Paper V

The Lagrange’s equation is

d

dt

(

∂L

∂θ

)

− ∂L

∂θ= 0

d

dt

(

1

2mr22θ

)

−[

1

2mr2ω22 sin θ cos θ +mgr sin θ

]

= 0

d

dt

(

mr2θ)

−[

1

2mr2ω22 sin θ cos θ +mgr sin θ

]

= 0

mr2θ −mrω2 sin θ cos θ −mgr sin θ = 0

5 Mass spring system : A particle of mass m is connected by a

massless spring of stiffness K and unstressed length r0 to a point

P which is moving along a circubr path of radius a at a uniform

angular rate ω. Assuming that the particle moves without friction

on a horizontal plane, find the differential equations of motion.

Solution :

Resolving r and rθ along aω and rθ .

In this probem, we have dealing with a single particle of mass ‘m’.

Let O taken as origin of refeence length ℓ.

~rP is the position vector of the point P w.r.to the point O.·

~ρ is the velocity of the particle w.r.to P.

Let T be the K.E. of the particle, V is the P.E of th+e particle.

T =1

2m

·

~r·

~rP +1

2m

·

~ρ2

~rPm·

~ρO

·

~ρO is the position vector of the system w.r.to ρ.

In the above case, ρC

= ρ

T =1

2m

·

~r·

~rP +1

2m

·

~ρ2

~rPm·

~ρC

·

~rP = aω·

~ρ2

= r2 + r2θ2

·

~rPm·

~ρ = maω[

r sin (θ − ωt) + rθ cos (θ − ωt)]

53

Page 54: MSc Maths Optional Paper V

T =1

2ma2ω2 +

1

2m(

r2 + r2θ2)

+maω[

r sin (θ − ωt) + rθ cos (θ − ωt)]

V =1

2K (r − r0)

2

L = T − V

=1

2ma2ω2 +

1

2m(

r2 + r2θ2)

+maω[

r sin (θ − ωt) + rθ cos (θ − ωt)]

− 1

2K (r − r0)

2

The Lagrange’s r equation is

d

dt

(

∂L

∂r

)

− ∂L

∂r= 0

d

dt

[

1

2m2r +maω sin (θ − ωt)

]

1

2m2rθ

2+maω

(

θ cos (θ − ωt))

− 1

2K2 (r − r0)

= 0

mr +maω cos (θ − ωt)(

θ − ω)

−mrθ2 −

maωθ cos (θ − ωt) +K (r − r0) = 0

mr −maω2 cos (θ − ωt) −mrθ2+K (r − r0) = 0

The Lagrange’s θ equation is given by

d

dt

(

∂L

∂θ

)

− ∂L

∂θ= 0

d

dt

[

1

2m2r2θ +maω cos (θ − ωt)

]

maωr cos (θ − ωt)(

−maωrθ sin (θ − ωt))

= 0

mr2θ + 2mrθ +maωr cos (θ − ωt) −maωrθ sin (θ − ωt) +maωr2 sin (θ − ωt) −maωr cos (θ − ωt) +maωr sin (θ − ωt) = 0

mr + 2mrθ +maω2r cos (θ − ωt) = 0

54

Page 55: MSc Maths Optional Paper V

Knife edge problem :

Example of a non-holonomic rheonomic system.

Two particles are connected by a rigid massless rod of length ℓ

which rotates in a horizontal plane with a constant angular vlocity

ω. Knife - edge supports at the two particles prevent either particle

from having a velocity component along the rod, but the particles can

slide without friction in a direction perpendicular to the rod. Find the

differential equations of motion. Solve for x, y and the constraint force

as functions of time if the center of mass is initially at the origin and

has a velocity v0 in the positive y direction.

Solution :

Let (x, y) be the co-ordinates of the CM at time the system has

translational and rotational K.E

Velocity component along the rod = 0

∴ (x1 + x2) cosωt+ (y1 + y2) sinωt = 0 (1)

T =1

2m(

x21 + y2

1

)

+1

2m(

x22 + y2

2

)

=1

2m(

x21 + x2

2 + y21 + y2

2

)

=1

2m

(

2x2 + 2y2 + ℓ2ω2

2

)

= m(

x2 + y2)

+ +mℓ2ω2

4

∴ V = 0

Lagrange’s equation of for non-haolonomic system is

d

dt

(

∂L

∂qi

)

− ∂L

∂qi=

m∑

i=1

λjaij = λ1a11.

The velocity of centre of mass exsited only in a direction perpen-

dicular to the rod. Hence it has no velocity along the rod.

55

Page 56: MSc Maths Optional Paper V

x1 = x− ℓ

2cosωt

y1 = y − ℓ

2sinωt

x2 = x+ℓ

2cosωt

y2 = y +ℓ

2sinωt

x1 = x+ℓω

2sinωt

y1 = y − ℓω

2cosωt

(2)

x2 = x− ℓω

2sinωt

y 2 = y +ℓω

2cosωt

From (1) and (2)

x cosωt+ y sinωt = 0

x cosωt+ y cos(

900 − ωt)

= 0

x cosωt+ y sinωt = 0 (A)

[

x+ℓω

2sinωt+ x− ℓω

2sinωt

]

cosωt+

sinωt

[

y − ℓω

2cosωt+ y +

ℓω

2cosωt

]

= 0

2x cosωt+ 2y sinωt = 0

cosωtdx+ sinωtdy = 0n∑

i=1

ajidqi + ajtdt = 0

a11x+ a12y = 0, q1 = x1, q2 = y2

56

Page 57: MSc Maths Optional Paper V

a11 = cosωt

a12 = sinωt

n∑

i=1

ajiaij = 1

d

dt

(

∂L

∂x

)

− ∂L

∂x= λa11

L = T − V

= m(

x2 + y2)

+m

4ℓ2ω2

∂L

∂x= 2mx

d

dt(2mx) − 0 =

n∑

i=1

λjaji = λ1 cosωt

2mx = λ1 cosωt (B)

d

dt

(

∂L

∂y

)

− ∂L

∂y= λa12

d

dt(2my) = λ1a12

2my = λ1 sinωt (C)

(C)

(B)⇒ tanωt =

y

x

x cosωt+ y sinωt = 0

− xy

= tanωt

y

x= − x

y2xx+ 2yy = 0

2xd (x) + 2yd (y) = 0

d

dt

(

x2 + y2)

= 0

d

dt

(

2x2 + 2y2)

= 0

2x2 + 2y2 = constant

57

Page 58: MSc Maths Optional Paper V

Initially x = 0, y = 0,

0 + v20 = 0,

x2 + y2 = v20

x cosωt+ y sinωt = 0

Hence the centre of mass moves with constant velocity, satisfying those

equation, we have

x = −v0 sinωt

y = v0 cosωtdx

dt= −v0 sinωt

dx = −v0 sinωtdt∫

dx = −∫

v0 sinωtdt

x =v0

ωcosωt+ C1

x = 0, t = 0

0 =v0

ω+ C1

C1 = −v0

ω

x =v0

ωcosωt− v0

ω

=v0

ω(cosωt− 1)

y0 = 0; t = 0

C2 = 0

∴ y =v0

ωsinωt

Eliminating t by ω, x and y we have circular path. The generalized

58

Page 59: MSc Maths Optional Paper V

forces are

Ci =m∑

i=1

λjaji, j = 1, 2, 3, · · · ,m

C1 = λ1a11

C1 = cosωt2mx

cosωt

[

∵ λ1 =2mx

cosωt

]

x = −v0 sinωt

x = −v0ω cosωt

∴ C1 = −2m (v0 cosωt)

C2 = λ2a12

=2my

cosωtsinωt

=−2mv0 sinωt cosωt

cosωt= −2mv0 sinωt

Note:

In the above case,

x =v0

ω(cosωt− 1)

x+v0

ω=

v0

ωcosωt

(

x+v0

ω

)2

= y +v2

0

ω2

Hence the system moves in a circular path with radius v0/ω.

2.3 Integrals of the motion:

Ignorable co-ordinates:

Let the holnomic system be described by n standard Lagrange’s

equationd

dt

(

∂L

∂qi

)

− ∂L

∂qi= 0, i = 1, 2, 3, · · · , n

Let (q, q, t) containing all q1, q2, · · · , qn, but some of the q′s, say q1, q2, · · · , qnare missing from the Lagrangian. These k− co-ordinaties are called ig-

norable co-ordinates. Since∂L

∂qiis zero for each ignorable co-ordinate,

59

Page 60: MSc Maths Optional Paper V

it follows that

d

dt

(

∂L

∂qi

)

= 0, (i = 1, 2, 3, · · · , k) ,

or

pi =∂L

∂qi= βi, i = 1, 2, 3, · · · , k

where the β′s are constants evaluated from the initial conditions. Hence

we find that the generalized momentum corresponding to each ignor-

able co-ordinate is constant, that is, it is an integral of the motion.

Example :

Let us consider the Kepler’s problem, that is, the problem of the

motion of a particle of unit mass which is attracted by an inverse square

gravitational force to a fixed point O. Using polar co-ordinates, the

kinetic and potential evergies are

T =1

2

(

r2 + r2θ2)

V = −µr

where µ is a positive constant known as the gravitational coefficeient.

The Lagrangian function is

L = T − V =1

2

(

r2 + r2θ2)

r

L is a independent co-ordinates

Lagrangian ‘r’ equation is

d

dt

(

∂L

∂r

)

− ∂L

∂r= 0

d

dt(r) − 1

2

(

2rθ2)

r2= 0

r − rθ2+µ

r2= 0. (1)

Since θ does not appear explicity in the Lagrangian function, it is an

60

Page 61: MSc Maths Optional Paper V

ignorable co-ordinate. The θ equation of motion is

d

dt

(

r2θ)

= 0

or∂L

∂θ= constant

1

2r22θ = β ⇒ θ =

β

r2

r2 =β

θ(2)

where β is constant and is equal to the angular momentum of the

particle about the attracting centre O.

r − rβ2

r4+µ

r2= 0 ( using (2) in (1))

r − β

r3+µ

r2= 0

The Routhian function :

Consider a standard holonmic system with q1, q2, · · · ., qk are ignor-

able co-ordinates

L = L (qk+1, · · · , qn, q1, q2, · · · , qn, t) .

Now let us define a Routhian function

R (qk+1, · · · , qn, qk+1, · · · , qn, β1, β2, · · · , βk, t)

as follows,

R = L−k∑

i=1

βiqi

where βi =∂L

∂qi, i = 1, 2, 3, · · · , k.

Obtain the Lagrange’s equation using Routhian funcion.

Solution :

61

Page 62: MSc Maths Optional Paper V

Consider the holonomic system with ‘n’ independent generalized

co-ordinates

q1, q2, · · · .., .qn. Letq1, q2, · · · .., qk be the ignorable co-ordinates. The

Legrangian function is

L = L (qk+1, · · · , qn, q1, q2, · · · , qn, t) .

Now∂L

∂qi= βi, i = 1, 2, 3, · · · , k (1)

Now the Routhian funciton is given by

R = L−k∑

i=1

βiqi (2)

Subsitute the values for q1, q2, · · · , qk

Using (1) we have

R = R (qk+1, · · · , qn, qk+1, · · · , qn, β1, β2, · · · , βk, t) .

Differentiating (2)

δR = δL− δk∑

i=1

βiqi (3)

δL =n∑

i=k+1

∂L

∂qiδqi +

k∑

i=1

∂L

∂qiδqi +

n∑

i=k+1

∂L

∂qiδqi +

∂L

∂tδt

δR =n∑

i=k+1

∂R

∂qiδqi +

n∑

i=k+1

∂R

∂qiδqi +

n∑

i=1

∂L

∂βi

δβi +∂R

∂tδt (4)

δ

k∑

i=1

βiqi =k∑

i=1

∂L

∂qiδqi +

k∑

i=1

qiδβi

62

Page 63: MSc Maths Optional Paper V

(3)⇒

δ

(

L−k∑

i=1

βiqi

)

=n∑

i=k+1

∂L

∂qiδqi +

n∑

i=k+1

∂L

∂qiδqi−

k∑

i=1

qiδβi +∂L

∂tδt (5)

From (4) and (5)

The corresponding coefficients must be equal.

∂L

∂qi=∂R

∂qi, i = k + 1, k + 2, k + 3, · · · , n

∂L

∂qi=∂R

∂qi, i = k + 1, k + 2, k + 3, · · · , n

(6)

and

qi = − ∂R

∂βi

, i = 1, 2, 3, · · · , k

∂L

∂t=

∂R

∂t.

Now let us substitute from equation (6) in to Lagrangian equations

and obtain

d

dt

(

∂R

∂qi

)

− ∂R

∂qi= 0, i = k + 1, k + 2, k + 3, · · · , n

∂R

∂βi

= −qi, i = 1, 2, 3, · · · , k

Illustrate Kepler’s problem using Routhian method.

Solution :

Let us assume that the particle has unit mass. Let (r, θ) be the

polar co-ordinates of the particle.

Let T be the kinetic energy of the particle. V is the potential evegy

of the particle.

T =1

2

(

r2 + r2θ2)

V = −µr

L = T − V

63

Page 64: MSc Maths Optional Paper V

=1

2

(

r2 + r2θ2)

r∂L

∂θ= 0

∴ θ is ignorable solution

∂L

∂θ= β ⇒ r2θ = β

Now the Routhian function is given by

R = L− βθ

=1

2

(

r2 + r2θ2)

r− βθ

=1

2r2 +

1

2r2β

2

r4+µ

r− β2

r2

=1

2r2 +

µ

r− 1

2

β2

r2

Routhian equation is given by

d

dt

(

∂R

∂qi

)

−(

∂R

∂qi

)

= 0

d

dt

(

∂R

∂r

)

−(

∂R

∂r

)

= 0 (1)

Here there are two co-ordinates (r, θ)

∂L

∂θ= β ( constant)

∴The Routhian equation is only at r

d

dt(r) −

[

(

− µ

r2

)

− 1

2β2(

−2r−3)

]

= 0

r +µ

r− β2

r3= 0

or

r − β2

r3+µ

r= 0

64

Page 65: MSc Maths Optional Paper V

Conservative system :

Write down the 3 condition defining a conservative system

and show that these conditions are sufficeient to ensure the

existence of an energy integral or Jacobi integral.

Proof :

1. The standard form of Lagrange’s equation (holonoic or nonholo-

nomic) applies.

2. The Legrangian function L is not an explicit function of time.

3. Any constraint equatins can be expressed in the differential form

n∑

i=1

ajidqi = 0, j = 1, 2, 3, · · · ,m

ie, all the coefficients are equal to zero.

Consider the standard nonholonomic form of Lagrange’s equation

d

dt

(

∂L

∂qi

)

− ∂L

∂qi=

n∑

i=1

λjaji, i = 1, 2, · · · , n (1)

where L (q, q) is not an explicit function of time.

Now let us consider the total derivative

αjt =∂φ

∂t

dL

dt=

n∑

i=1

∂L

∂qiq +

n∑

i=1

∂L

∂qiqi (2)

(1) ⇒ ∂L

∂qi=

d

dt

(

∂L

∂qi

)

−m∑

j=1

λjaji

(2) ⇒ dL

dt=

n∑

i=1

[

d

dt

(

∂L

∂qi

)

−m∑

j=1

λjaji

]

qi +n∑

i=1

∂L

∂qiqi

65

Page 66: MSc Maths Optional Paper V

=n∑

i=1

[

d

dt

(

∂L

∂qi

)

qi +∂L

∂qiqi

]

−n∑

i=1

m∑

j=1

λjajiqi

=d

dt

[

n∑

i=1

(

∂L

∂qi

)

qi

]

−m∑

j=1

(

n∑

i=1

ajiqi

)

λj

dL

dt=

d

dt

[

n∑

i=1

(

∂L

∂qi

)

qi

]

− 0

d

dt

[

L−n∑

i=1

(

∂L

∂qi

)

qi

]

= 0

d

dt

[

n∑

i=1

(

∂L

∂qi

)

qi − L

]

= 0

Integrating w.r.to t

n∑

i=1

(

∂L

∂qi

)

qi − L = h ( constant)

∴ h =n∑

i=1

(

∂L

∂qi

)

qi − L

L =n∑

i=1

(

∂L

∂qi

)

qi − h

h = T2 − T0 + V

=n∑

i=1

(

∂L

∂qi

)

qi − L

This integral is called Jacobi integral or energy integral. This en-

ergy integral exists for all conservative system.

Book work :

Define a natural system. Prove that for a conservative

system, Jacobi integral is not equal to the total evergy.

Proof :

For every conservative system

h = T2 − T0 + V

66

Page 67: MSc Maths Optional Paper V

For the conservative system energy integral

h =n∑

i=1

(

∂L

∂qi

)

qi − L (1)

L = T − V

= T2 + T1 + T0 − V

We know that

T2 =1

2

n∑

i=1

m∑

j=1

mjiqiqj

T1 =n∑

i=1

aiqi

T0 =1

2

3N∑

k=1

mk

(

∂xk

∂t

)2

n∑

i=1

∂L

∂qiqi =

n∑

i=1

∂T

∂qiqi

=n∑

i=1

(

∂T2

∂qi+∂T1

∂qi+∂T0

∂qi

)

qi

= 2T2 + T1 + 0

(1) ⇒ h = 2T2 + T1 − T1 − (T − V )

= 2T2 + T1 − T2 − T0 + V

= T2 − T0 + V = T ′ + V ′

∴ The energy T ′ + V ′ is constant for any conservative system but

it is not equal to total energy.

Natual system :

A natual system is a consevative system.

Proof :

A system is said to be natural system if

67

Page 68: MSc Maths Optional Paper V

(i) it is described by standard form of holonomic Lagrangian equa-

tion.

(ii) the Lagrangian function L is not an explicit function of time.

(iii) any constraint equation can be expressed in the differential form

n∑

i=1

ajidqi = 0

(iv) the K.E is expressed as homogeneous quadratic functio of q’s.

To prove Jacobi integral = Total energy for the natural system.

T =1

2

n∑

i=1

m∑

j=1

mijqiqj = T2

T0 = 0;T1 = 0

We know that

L = T − V, L = T2 − V∂L

∂qi=

∂T2

∂qin∑

i=1

∂L

∂qiqi =

n∑

i=1

∂T2

∂qiqi = 2T2 [ by the above Book work]

h =n∑

i=1

∂L

∂qiqi − L

= 2T2 − (T2 − V )

= T2 + V

Jacobi integral = Total evergy.

Example : 1 Suppose a mass - spring system is attached to a

frame which is translating with a uniform velocity v0 as shown

in figure. Let ℓ0 be the unstressed spring length and use the

elongation x as the generalized co-ordinate. Find the Jacobi

integral for the system.

68

Page 69: MSc Maths Optional Paper V

Solution : Let m be the mass of the body. Let x be the displacement

of the body from the relating postion at time t .

0v

km

0x+l

Velocity of the body = x

Velocity of the system = v0 + x

The K.E of the system =1

2m (v0 + x)2

T =1

2m(

v20 + x2 + 2v0x

)

T2 =1

2mx2

T1 =1

2mv0x

T0 =1

2mv2

0

Let V be the potntial Energy of the system.

∴ V =1

2kx2

L = T − V

=1

2mv2

0 +1

2mx2 +mv0x =

1

2kx2

L is independent of ‘t’

Hence the system is conservative.

Jacobian integral h = T2 − T0 + V

h =∂L

∂xx− L

= (mx+mv0) x−

1

2mv2

0 +1

2mx2 +mv0x

2 − 1

2kx2

69

Page 70: MSc Maths Optional Paper V

= mx2 +mv0x−1

2mv2

0 −1

2mx2 −mv0x

2 +1

2kx2

=1

2mx2 − 1

2mv2

0 +1

2kx2

= T2 − T0 + V

Example 2 : A small tabe, bent in the form of a circle of

radius r, rotates about a vertical diameteer with a constant

angular velocity ω. A particle of mass m can slide without

friction inside the tube. At any given time, the configuration

of the system is specified by the angle θ which is measured

from the upward vertical to the line connecting the center O

and the particle. Find the Jacobi integral.

Solution :

w

g

O

q

r

m

Let O be the center of the tube. The transformation equations

relating the generalized co-ordinate θ and the position (x, y, z) of the

particle.

x = r sin θ cosωt

y = r sin θ sinωt

z = r cos θ

70

Page 71: MSc Maths Optional Paper V

AP

OP= sin θ ⇒ AP = r sin θ

OA

OP= cos θ ⇒ OA = r cos θ

where r is the radius of the tube.

Let T be the K.E of the particle of mass m, V is the P.E of the

particle

T =1

2m(

r2θ2+ r2ω2 sin2 θ

)

=1

2mr2

(

θ2+ ω2 sin2 θ

)

V = mgr cos θ, T2 =1

2mr2θ

2, T1 = 0, T0 =

1

2mr2ω2 sin2 θ

L = T − V =1

2mr2

(

θ2+ ω2 sin2 θ

)

−mgr cos θ

L is independent of t.

Hence the system is conservative Jacobi method.

h =n∑

i=1

∂L

∂qiqi − L

=∂L

∂θθ − L

=(

mr2θ)

θ −(

1

2mr2θ

2+

1

2r2ω2 sin2 θ −mgℓ cos θ

)

=1

2mr2θ

2 − 1

2r2ω2 sin2 θ +mgℓ cos θ

= T2 − T0 + V.

Example 3 :

Two particles, each of mass m, are connected by a rigid mass-

less rod of length ℓ. The particles are supported by knife

edges placed perpendicular of the rod. Assuming that all

moion is confined to the horizontal xy plane, find the Jacobi

integral.

Solution: Let (x1, y1), (x2, y2) be the co-ordinate of the two ends of

71

Page 72: MSc Maths Optional Paper V

the rod at time ‘t’.

Let (x, y) be the co-ordinates of the cm and we have holonomic con-

straint

(x1 − x2)2 + (y1 − y2)

2 = ℓ2 (1)

We know that

tanωt =y2 − y1

x2 − x1

y2 − y1 = (x2 − x1) tanωt (2)

x1 = x− ℓ

2cosωt

x2 = x+ℓ

2cosωt

y1 = y − ℓ

2sinωt

y2 = y +ℓ

2sinωt

x1 + x2 = 2x

y1 + y2 = 2y

Let T be the K.E. of the system. Let V be the P.E. of the system.

V = 0

T =1

2m(

x21 + y2

1

)

+1

2m(

x22 + y2

2

)

x1 = x+ℓ

2ω sinωt

x2 = x− ℓ

2ω sinωt

y1 = y − ℓ

2ω cosωt

y2 = y +λ

2ω cosωt

x21 + x2

2 + y21 + y2

2 = 2(

x2 + y2)

+ℓ2

2ω cos

ω

2

= 2(

x2 + y2)

+ 2ℓ2ω2

4

72

Page 73: MSc Maths Optional Paper V

T = m(

x2 + y2)

+mℓ2ω2

4V0 = 0, T2 = mx2 +my2

T0 =mℓ2ω2

4,

h = T2 − T0 + V

L = T − V = T − 0 = T

L is independent of t.

cosωtx+ sinωty = 0

ajit = 0

The system is conservative.

Jacobi integral h =n∑

i=1

∂L

∂qiqi − L

=∂L

∂xx+

∂L

∂yy − L

= (2mx) x+ (2my) y −(

mx2 +my2 +mℓ2ω2

4

)

= mx2 +my2 − 1

4mℓ2ω2.

These result show that the toal mass is M.I about the cente of mℓ2/2.

Orthogonal System :

Orthogonal system is an natural system. A natural system is said

to be orthogonal if

i) It is described by standard holonomic Lagrangian equation.

ii) The Lagrangian function L is not an explicit functio of time.

iii) The K.E contain’s only q2i and no cross products in the q′s.

Liouville’s system :

A system is said to be Liouville’s system if

73

Page 74: MSc Maths Optional Paper V

i) It is described by standard holomic Lagrange’s equation.

ii) The Lagrange’s function is not an explicit function of time.

iii) Any constraint equation can be expressed in the differential form

n∑

i=1

ajidqi = 0, j = 1, 2, 3, · · · ,m

iv) The K.E. Tand P.E. V are given by

K.E.T =1

2f

n∑

i=1

mi (qi) (qi)2 where mi (qi) > 0

P.E.V =1

f

n∑

i=1

vi (qi)

f =n∑

i=1

fi (qi > 0)

Liouville’s system special case of orthogonal system.

Book Work : Liouville’s system special case of orthogonal system

(OR)

With usual notation show that the following system is separable (or)

Reduce the orthogonal system to quadrature. (Or)

Explain one of the Quadratic system to be orthogonal system.

T =1

2f

n∑

i=1

q2i

V =1

f

n∑

i=1

vi (qi)

f =n∑

i=1

fi (qi > 0)

Proof : Lagrange’s equation is in the form

d

dt

(

∂L

∂qi

)

− ∂L

∂qi= 0

74

Page 75: MSc Maths Optional Paper V

f = f1 (q1) + f2 (q2) + · · · + fi (qi) + · · · + fn (qn)

L = T − Vd

dt

(

∂ (T − V )

∂qi

)

− ∂ (T − V )

∂qi= 0

d

dt

(

∂T

∂qi

)

− ∂T

∂qi+∂V

∂qi= 0

d

dt(f qi) −

1

2

∂fi

∂qi

n∑

i=1

q2i +

[

1

f

∂vi

∂qi− 1

f 2

∂fi

∂qi

n∑

i=1

vi (qi)

]

= 0

d

dt(f qi) −

1

2

∂fi

∂qi

n∑

j=1

q2j +

1

f

∂vi

∂qi− v

f

∂fi

∂qi= 0 (1)

This is a natural system.

∴ T + V = h; qi = qj

1

2f

n∑

j=1

q2j + V = h

1

2

n∑

j=1

q2j =

h− V

f

(1) ⇒ d

dt(f qi) −

h− V

f

(

∂fi

∂qi

)

+1

f

(

∂vi

∂qi

)

− V

f

(

∂vi

∂qi

)

= 0

d

dt(f qi) −

h

f

(

∂fi

∂qi

)

+1

f

(

∂vi

∂qi

)

= 0

Multiply 2f qi on both sides

2f qid

dt(f qi) − 2f qi

(

h

f

)(

∂fi

∂qi

)

+2f qif

(

∂vi

∂qi

)

= 0

2f qi

[

d

dt(f qi)

]

− 2hfqi

(

∂fi

∂qi

)

+ 2qi

(

∂vi

∂qi

)

= 0

d

dt

(

f 2q2i

)

− 2h

(

dqidt

)(

∂fi

∂qi

)

+ 2qi

(

∂vi

∂qi

)

= 0

d

dt(f qi)

2 − 2h

(

dqidt

)(

∂fi

∂qi

)

+ 2qi

(

∂vi

∂qi

)

= 0

75

Page 76: MSc Maths Optional Paper V

d

dt(f qi)

2 − 2h

(

dqidt

)(

∂fi

∂qi

)

+ 2

(

∂vi

∂qi

)(

dqidt

)

= 0 (2)

d

dt(f qi)

2 − 2h

(

∂fi

∂t

)

+ 2

(

∂vi

∂t

)

= 0

d

dt(f qi)

2 = 2h

(

∂fi

∂t

)

− 2

(

∂vi

∂t

)

= 2d

dt(hfi − vi)

T + V =1

2f

n∑

i=1

q2i + V = h

1

2f

n∑

i=1

q2i + 2

n∑

i=1

vi (qi) = h

Multifly 2f on both sides,

f 2

n∑

i=1

q2i + 2

n∑

i=1

vi (qi) − 2fh = 0 (3)

(2) ⇒

d

dt

(

f 2i q

2i

)

= 2

[

d

dt(hfi − vi)

]

d

dt

(

f 2q2i

)

= 2

∫ [

d

dt(hfi − vi)

]

f 2qi = 2 (hfi − vi) + 2ci

q2i =

2

f 2(hfi + ci − vi)

Let 2ci = f 2q2i − 2fhi + 2vi.

2n∑

i=1

ci = 0 ⇒n∑

i=1

ci = 0 [by (3)]

Here c′is and h together comprise n - independent constant of mo-

tion.

76

Page 77: MSc Maths Optional Paper V

q2i =

2

f 2i

[(hfi − vi) + ci]

qi =

2 [(hfi − vi) + ci]

f

dqidt

=

2 [(hfi − vi) + ci]

fdqi

2 [(hfi − vi) + ci]=

dt

f

dq1√

2 [(hf1 − v1) + c1]=

dq2√

2 [(hf2 − v2) + c2]= · · ·

· · · =dqn

2 [(hfn − vn) + cn]=dt

f= dτ (4)

Hence required system reduced to quadratures.

Note:

Replacing dqi by√

mi (qi)dqi

T =1

2f

n∑

i=1

mi (qi) q2i where mi (qi) > 0

V =1

f

n∑

i=1

vi (qi)i ; f =n∑

i=1

fi (qi) > 0

A natural system of this type is Liouville’s system.

The equation (4) ⇒

dq1√

Q1 (q1)=

dq2√

Q2 (q2)= · · · =

dqn√

Qn (qn)=dt

f= dτ

Qi (qi) =2

mi

(hfi − vi + ci) , i = 1, 2, 3, · · · , n

f1dq1√

Q1 (q1)=

f2dq2√

Q2 (q2)= · · · =

fndqn√

Qn (qn)=f · dtf

77

Page 78: MSc Maths Optional Paper V

f1dq1√

Q1 (q1)+

f2dq2√

Q2 (q2)+ · · · + fndqn

Qn (qn)=

(f1 + f2 + · · · + fn)

fdt

=f

fdt = dt

∴ dt =n∑

i=1

fidqi√

Q1 (qi)= t+ β1,whereβ1is constant.

Show that spherical pendularn problem can be solved com-

pletly by quadratures.

Solution :

T =1

2m(

ℓ2θ2+ ℓ2φ

2sin2 θ

)

V = mgℓ cos θ

where m is the mass of the particle.

h = T + V, L = T − V

The system is conservative and natural.

h =1

2m(

ℓ2θ2+ ℓ2φ

2sin2 θ

)

+mgℓ cos θ

L =1

2m(

ℓ2θ2+ ℓ2φ

2sin2 θ

)

−mgℓ cos θ

L is an independent on φ,∴ φ is ignorable.

∂L

∂φ= a constant = αφ

1

2mℓ22φ sin2 θ = αφ

φ =2φ

mℓ2 sin2 θ

(1) ⇒ h =1

2mℓ2θ

2+

1

2

mℓ2αφ2 sin2 θ

m2ℓ4 sin4 θ+mgℓ cos θ

=1

2mℓ2θ

2+

αφ2

2mℓ2 sin2 θ+mgℓ cos θ

78

Page 79: MSc Maths Optional Paper V

Multiply by 2mℓ2 sin2 θ,

m2ℓ4 sin2 θ θ2+ 2φ2 = 2mℓ2 sin2 θ (h−mgℓ cos θ)

θ =

2mℓ2 sin2 θ (h−mgℓ cos θ) − 2φ2

mℓ2 sin θ

θ =dθ

dt.

This values apply dφ

θ∫

θ0

mℓ2 sin θ√

2mℓ2 sin2 θ (h−mgℓ cos θ) − 2φ2

dθ =

t∫

t0

dt = t− t0

And

αφ

mℓ2 sin2 θ= φ

dt=

αφ

mℓ2 sin2 θ

dφ =αφ

mℓ2 sin2 θdt

(1) ⇒ αφ

mℓ2 sin2 θdt =

mℓ2 sin θdθ

2mℓ2 sin2 θ×

αφ

mℓ2 sin θ(h−mgℓ cos θ) − αφ2

=αφdθ

sin θ√

2mℓ2 sin2 θ (h−mgℓ cos θ) − αφ2

φ∫

φ0

dφ =

θ∫

θ0

αφdθ

sin θ√

2mℓ2 sin2 θ (h−mgℓ cos θ) − αφ2

φ− φ0 =

θ∫

θ0

αφdθ

sin θ√

2mℓ2 sin2 θ (h−mgℓ cos θ) − αφ2

where φ0 = φ (t0) .

Now we have obtained the required four constraints of motion

79

Page 80: MSc Maths Optional Paper V

namely αφ, h, t0, φ0.

Discuss probem of the spherical pendulum considering it as

a Lioville’s system.

Solution : We know that

T =1

2m(

ℓ2θ2+ ℓ2φ

2sin2 θ

)

V = mgℓ cos θ

For Lioville’s system

T =1

2f

(

n∑

i=1

mi (qi) q2i

)

V =1

f

n∑

i=1

vi (qi)i

1

2(fθ + fφ) = mθθ

2+mφφ

2

=1

2

[

mℓ2θ2+mℓ2 sin2 θφ

2]

(fθ + fφ)Mθ = mℓ2

(fθ + fφ)Mφ = mℓ2 sin2 θ

=1

sin2 θ,Mφ = 1,Mθ =

1

sin2 θ

(fθ + fφ) v = v1 (θ) + v (φ)(

Mℓ2 sin2 θ)

×mgℓ cos θ = v1 (θ)

vθ = m2gℓ3 sin2 θ cos θ

v1 (θ) = m2gℓ3 sin2 θ cos θ

fθ + fφ = mℓ2 sin2 θ

fθ = mℓ2 sin2 θ [∵ fφ = 0]

We know that

φi (qi) =2

Mi

(hfi − vi + ci) (i = 1, 2, · · · , n)

φθ =2

(hfθ − vθ + cθ)

80

Page 81: MSc Maths Optional Paper V

=2

[

hmℓ2 sin2 θ −m2gℓ3 sin2 θ cos θ]

= 2 sin2 θ[

Mℓ2 sin2 θ (h−mgℓ cos θ) + cθ]

φφ =2

[hfφ − vφ + cθ]

= 2 [h (0) − 0 + cφ]

= 2cφ

Using

f 2φ2

i = 2 [hfi (gi) − vi (qi) + ci]n∑

i=1

ci = 0

= 2 [hfφ − vφ + cφ]

∂L

∂φ= mℓ2φ sin2 θ = αφ

(fθ + fφ)2 φ

2= 2 [h (0) − 0 + cφ]

mℓ2φ sin2 θ = 2cφ

2cθ = −2cφ = −2φ2

2cφ = −2cθ =(

mℓ2φ sin2 θ)2

= αφ2

n∑

i=1

fidqi√

φi (qi)= t+ β1

θ∫

θ0

fθdθ√

φθ (θ)= t− t0

θ∫

θ0

mℓ2 sin2 θ√

φθ (θ)dθ = t− t0

We have

dq1√

φ1 (q1)−∫

dqi√

φj (qj)= βj, j = 1, 2, 3, · · · , n

81

Page 82: MSc Maths Optional Paper V

θ∫

θ0

dθ√

φ0 (θ)=

φ∫

φ0

dφ√

2cφ

[

∵ φφ = 2cφ]

θ∫

θ0

αφdθ√

φ0 (θ)= φ− φ0

Now we have obtained four required constraint cθ, cφ, φℓ, φ0.

Book work :Small Oscillations

Obtain the equation of the natural system by given a small

motion about its equilibrium position. OR

Obtain the equation of natural system in matrices form as

mq + kq = 0

Proof :

Let the natural system be given by ‘n’ independent generalized co-

ordinates q1, q2, q3, · · · · · · ., qn . Let us assume all the q′s all measurable

from a position of equilibrium.

Consider small motion about this equilibrium position.

Let v0 be the reference point is zero. The P.E. can be written in

the form

∴ V =1

2

n∑

i=1

n∑

j=1

(

∂2v

∂qi∂qj

)

0

qiqj = 0.

Neglecting terms of higher p order terms the second in the q′s we obtain

V =1

2

n∑

i=1

n∑

j=1

(

∂2v

∂qi∂qj

)

0

qiqj

V =1

2

n∑

i=1

n∑

j=1

kijqiqj where kij =∂2v

∂qi∂qj

is the stiffness co-efficient.

V is the homogenous as quoctratic of the q′s for small motion near.

Small motion near a positon of equilibrium.

Let us assume the system consists N - particles with caresian Co-

82

Page 83: MSc Maths Optional Paper V

ordinates x1, x2, · · · .., x3N then the K.E of the system is given by

T =1

2

n∑

i=1

n∑

j=1

mij qiqj

where mij = mji =3N∑

k=1

mk

(

∂xk

∂qi

)(

∂xk

∂qj

)

L = T − V

=1

2

n∑

i=1

n∑

j=1

mij qiqj −1

2

n∑

i=1

n∑

j=1

kijqiqj (1)

The standard Lagrange’s equation for a holomic system is

d

dt

(

∂L

∂qi

)

− ∂L

∂qi= 0, i = 1, 2, 3, · · · , n

(1) ⇒n∑

j=1

mij qj +n∑

j=1

kijqj = 0

mj qj + kijqj = 0

∴ mq + kq = 0

This is the equation of natural system

Memory point : i) For a spherical pendulum

L = T − V

T =1

2m(

ℓ2θ2+ ℓ2φ

2sin2 θ

)

V = mgℓ sin θ

ii) For double pendulum

v1 = ℓθ, v2 =

ℓ2θ2+ ℓ2φ

2+ 2ℓθφ cos (φ− θ)

T =1

2m(

v21 + v2

2

)

83

Page 84: MSc Maths Optional Paper V

=1

2mℓ2θ

2+

1

2m[

ℓ2θ2+ ℓ2φ

2+ 2ℓθφ cos (φ− θ)

]

V = −mgℓ (2 cos θ + cosφ)

iii) For a small tube bent is

T =1

2m(

r2θ2+ r2ω2 sin2 θ

)

L = mgr cos θ

iv) For mass storing system

T =1

2mr2ρ2 +

1

2m

·

~ρ+·

~rPm··

~ρC

=1

2m

·

~r2

P +1

2m

·

~ρ2

~rP

·

~rP = aω,·

~ρ = r2 + r2θ2

·

~rPm··

~ρ = maω[

r sin (θ − ωt) + rθ cos (θ − ωt)]

T =1

2ma2ω2 +

1

2m(

r2 + r2θ2)

+maω[

r sin (θ − ωt) + rθ cos (θ − ωt)]

V =1

2k (r − r0)

2

v) For Kepler’s Probem

T =1

2

(

r2 + r2θ2)

V = −µr, L = T − V

R = L− βθ

vi) Two particles each of masses m find Jjacobi integral.

(x1 − x2)2 + (y1 − y2)

2 = ℓ2

tanωt =y2 − y1

x2 − x1

(y2 − y1) = (x2 − x1) tanωt

84

Page 85: MSc Maths Optional Paper V

UNIT III

Hamilton’s Equations

Stationary Value :

To find the stationary values of the function f 6= z subject to

constraints

φ1 = x2 + y2 + z2 − 4 = 0

φ2 = xy − 1 = 0

combined force F = f + λ1φ1 + λ2φ2.

Solution:

Let augmented function be

F = z + λ1φ1 + λ2φ2

= z + λ1

(

x2 + y2 + z2 − 4)

+ λ2 (xy − 1)

∂F

∂x= 2xλ1 + λ2y = 0 (1)

∂F

∂y= 2yλ1 + λ2x = 0 (2)

∂F

∂z= 1 + 2zλ1 = 0 (3)

∂F

∂λ1

= x2 + y2 + z2 − 4 = 0 (4)

∂F

∂λ2

= xy − 1 = 0 (5)

(2)

(1)⇒ 2y

2x=

−x−y

⇒ x2 = y2 ⇒ x = ±y ⇒ y = ±x

(5) ⇒ xx− 1 = 0

x2 − 1 = 0

x2 = 1

85

Page 86: MSc Maths Optional Paper V

x = ±1

If x = 1, y = 1 and if x = −1, y = −1

(4) ⇒ x = 1, y = 1 ⇒ 1 + 1 + z2 − 4 = 0

z2 = 2 ⇒ z = ±√

2

Here solving the equation, we have stationary at(

1, 1,√

2)

,(

1, 1,−√

2)

,(

−1,−1,√

2)

and(

−1,−1,−√

2)

.

(3) ⇒ 1 + 2zλ1 = 0 ⇒ λ1 =−1

2√

2if z =

√2

If z = −√

2, λ1 =1

2√

2, λ1 = ± 1

2√

2

(2) ⇒ 2 (1)1

2√

2+ λ2 = 0

λ2 = − 1√2, λ2 =

1√2

λ2 = ± 1√2

It is seen that(

1, 1,√

2)

,(

−1,−1,√

2)

are the constraints of maxi-

mum oints and the other two points are minimum points.

Euler Lagrage equation or stationary values a definite inte-

gral.

Solution : Suppose we wish to find the stationary of

I =

x1∫

x0

f [y (x) , y′ (x) , x] dx

where y′ (x) =dy

dxand the limits are fixed.

Let us assume that derivatives f (y, y′, x) has two continuous deriva-

tives in each of its arguments. We have to find a function y′ (x) which

86

Page 87: MSc Maths Optional Paper V

gives a stationary value for I we’ve

y (x) = y′ (x) + δy (x)

where δy (x) is small variation in y. But small variation

δy = αη (x)

∴ δy (x) = αη (x)

∴ y (x) = y∗ (x) + 2η (x)

where η (x) is a arbitary function having the required smoothness and

α is an parameter which does not independent of x.

Hence for any given η (x) we can consider the varied curve y to be

function of α and x.

i.e., y (α, x) = y∗ (x) + αη (x) .

Let us assume the variation δy = 0 at the end points

2η (x0) = 0 and αη (x1) = 0

∴ y (x0) , y (x1) are fixed.

We see that the integral I is a function of α only for any given

η (x) .

I having a stationary value y∗ (x)

δI =

(

dI

)

α=0

= 0

∴ x0, x1 are independent on α

y (x, α) = y∗ (x) + αη (x)

∂y

∂α= η (x) ;

∂y′

∂α= η′ (x)

87

Page 88: MSc Maths Optional Paper V

I =

x∫

x0

f (y, y′, x) dx

dI

dx=

x∫

x0

(

∂f

∂y

∂y

∂α+∂f

∂y′∂y′

∂x

)

dx = 0

x∫

x0

∂f

∂yη (x) dx+

x∫

x0

∂f

∂y′η′ (x) dx = 0

Since η (x0) = 0, η (x1) = 0, we have

x∫

x0

∂f

∂yη (x) dx+

[

η (x)∂f

∂y′

]x

x0

−x∫

x0

η (x)d

dx

(

∂f

∂y′

)

dx = 0

x∫

x0

∂f

∂yη (x) dx+

[

η (x)∂f

∂y′

]x

x0

−x∫

x0

η (x)d

dx

(

∂f

∂y′

)

dx = 0

x∫

x0

∂f

∂yη (x) dx−

x∫

x0

η (x)d

dx

(

∂f

∂y′

)

dx = 0

Since η (x) is arbitary we have

∂f

∂y− d

dx

(

∂f

∂y′

)

= 0

⇒ d

dx

(

∂f

∂y′

)

− ∂f

∂y= 0

This equation is known as Euler- Lagrage’s equation for any curve

y = y∗(x) which result in a stationary value of I.

Branchistochrone Probem :

It is a find a curve y (x) between origin ‘O’ and the point (x1, y1)

such that a particle starting from rest at ‘O’ and sliding down the

curve without friction under the influence of uniform gravity field in a

minimum time to reach the end of the curve.

88

Page 89: MSc Maths Optional Paper V

Solution :

The vertical distance described by the particle to corresponding to

the point (x1, y1) is x with velocity.

Let P be the position of the particle at time with velocity v.

From principle of conservation of energy we obtain

mgx =1

2mv2

v = ±√

2gx

Let t be the time taken to reach the point (x1, y1) is found by noting

first that an infinitesimal path element ds is given by from O

ds = ±√

1 + y′2dx

t =

s∫

0

ds

v=

x1∫

0

1 + y′2√2gx

dx.

Let f (y, y′, x) =

1 + y′2

2gx. The Euler Lagrang equation is

∂F

∂y− d

dx

(

∂F

∂y′

)

= 0 (1)

∂F

∂y= 0

0 − d

dx

(

∂F

∂y′

)

= 0

∂F

∂y′= C where C is constant

∂F

∂y′=

∂y′

1 + y′2

2gx

= C

89

Page 90: MSc Maths Optional Paper V

⇒ 1

2

1 + y′2

2gx

· 2y′

2gx= C

y′√

2gx√

1 + y′2= C

y′2 = C2[

2gx(

1 + y′2)]

= 2C2gx+ 2gxC2y′2

=(

1 − 2gxC2 − 2gxC2)

y′2 − 2gxC2y′2 = 2C2gx⇒ y′2(

1 − 2gxC2)

= 2C2gx

y′2 =2C2gx

(1 − 2gxC2)

y′ =

2C2gx

(1 − 2gxC2)

Let x = a (1 − cos θ) where a =1

4gC2.

y′ =

2gC2 · 1

4gC2

(1 − cos θ)

1 − 2gC21

4gC2(1 − cos θ)

1

2(1 − cos θ)

1 − 1

2(1 − cos θ)

=

1

2(1 − cos θ)

1 − 2gC21

4gC2(1 − cos θ)

1 − cos θ

2 − cos θ=

1 − cos θ

1 + cos θ=

2 sin2θ

2

2 cos2θ

2

dy

dx=

sinθ

2

cosθ

2

⇒ dy =sin

θ

2

cosθ

2

dx

dy =sin

θ

2

cosθ

2

a sin θdθ

90

Page 91: MSc Maths Optional Paper V

=sin

θ

2

cosθ

2

a2 sinθ

2cos

θ

2dθ

= 2a sin2 θ

2dθ

dy =

2a sin2 θ

2dθ = a

(1 − cos θ) dθ

y = −a (θ − sin θ) + c

At x = y = 0, θ = 0 ⇒ c = 0.

Path is y = a(θ − sin θ), x = a (1 − cos θ) which is cycloid. The

constant ‘a’ is choosen δt the poth always through the point (x1, y1).

By comparing other paths through ‘O’ and (x1, y1) we can prove only

the cycloid path has minimum time.

Note:

y′2 − 1 − y′2√α− y

1 + y′2= c

−1√α− y

1 + y′2= c

−√

1 + y′2√α− y =

1

c

Taking square on both sides

(

1 + y′2)

(α− y) =1

c2= c1

y′2 =c1

α− y− 1 =

c1 − α+ y

α− y

y′ = ±√

c1 − α+ y

α− ytake y′ = −

c1 − α+ y

α− y

dy

dx= −

c1 − α+ y

α− y√α− y√

c1 − α+ ydy = dx (1)

91

Page 92: MSc Maths Optional Paper V

Take α− y = c1 sin2θ

2, dy = −c12 sin

θ

2cos

θ

2dθ

2Geodesic Problem :

Definition :

The Problem of finding the shortest path between two points in a

given space is called geoclesic.

The shortest path between two points is given space is the path

which forms a greaest circle which is the geodesic.

To prove this let us consider the equation of the problem.

Find the geodesic for the spherical surface.

(or)

Find the path of minimum length between two given points on the

two dimensional surface of a sphere of radius ‘r’

Let the spherical co-ordinate, (θ, φ) of the point of spherical surface.

Proof:

Let us use the spherical co-ordinate (θ, φ) as variable since r is a

constant.

We know that

x = r sin θ cosφ

y = r sin θ sinφ

z = r cos θ

The differential elements of length ds is given by

ds2 = r2dθ2 + r2 sin2 θdφ2

= r2dθ2

[

1 + sin2

(

)2]

ds = r

1 + sin2

(

)2

s = r

θ′∫

θ0

1 + φ2

1 sin2 θdθ where φ′ =dφ

dθ.

92

Page 93: MSc Maths Optional Paper V

Let f (φ, φ′, θ) =√

1 + φ′2 sin2 θ.We want to find the shortest dis-

tance of the path P and Q.

∴we have to find the stationary value of Euler - Lagrangian equa-

tion is∂f

∂y− d

dx

(

∂f

∂y′

)

= 0.

But here f is a function of (φ, φ′, θ) .

The new Euler’s Lagrangian equation for thus problem is

∂f

∂φ− d

(

∂f

∂φ′

)

= 0.

But f =√

1 + φ′2 sin2 θ.

∂F

∂φ= 0,

∂f

∂φ′=

1

2√

1 + φ′2 sin2 θ2φ′ sin2 θ

∴ Using (1) Euler’s Lagrangian equation

d

(

∂f

∂φ′

)

= 0

∂f

∂φ′= c

∂F

∂φ′=

φ′ sin2 θ√

1 + φ′2 sin2 θ= c

φ′ sin2 θ = c

1 + φ′2 sin2 θ

Squaring and rearranging we get

sin4 θφ′2 = c2(

1 + φ′2 sin2 θ)

sin2 θφ′2(

sin2 θ − c2)

= c2

φ′2 =c2

sin2 θ(

sin2 θ − c2)

φ′ =c

sin θ√

sin2 θ − c2

φ =

c

sin θ√

sin2 θ − c2dθ

93

Page 94: MSc Maths Optional Paper V

=

c

sin2 θ

1 − c2

sin2 θ

φ =

c

sin2 θ√

1 − c2 cosec2 θdθ

=

c cosec2 θ√1 − c2 cosec2 θ

= c

cosec2 θ√

1 − c2 (1 + cot2 θ)dθ

= −c∫

d (cot θ)√1 − c2 − c2 cot2 θ

= −c∫

d (cot θ)

c

(

1 − c2

c2

)

− cot2 θ

φ = cos−1

cot θ√

(

1 − c2

c2

)

+ φ0

= cos−1

cot θ√

(

1 − c2

c2

)

= cos−1

(

c cot θ√1 − c2

)

cos (φ− φ0) =c√

1 − c2cot θ

=c cot θ√1 − c2

cosφ cosφ0 − sinφ sinφ0 =c√

1 − c2cos θ

sin θ

Multiply by r sin θ,

r sin θ cosφ cosφ0 + r sin θ sinφ sinφ0 =c√

1 − c2cos θ

sin θr sin θ

=c√

1 − c2r cos θ

94

Page 95: MSc Maths Optional Paper V

The above equation becomes

x cosφ0 + y sinφ0 =c√

1 − c2z

Then the equation of the form ax + by + cz = 0 . This represents

the plane through the origin. This plane intersects the sphere along a

great circle which is geodesic.

Here φ0 and c are two constants which are adjustable such that the

greats circle will pass through P and Q.

General Euler’s Lagrangian equation :

Let

I =

x1∫

x0

f (y1, y2, · · · , yn, y′

1, y′

2, · · · , y′n, x) .

We have found a necessary condition for a stationary value of I.x0, r

are fixed limits.

State and prove Hamilton’s principle :

Variational principle or integration principle[Integrated form

of D’Alemberts principle]

Statement :

The actual path in configuration space followed by a holonomic

dynomical system during the fixed interval t0 to t1 is such that the

integral I =

t1∫

t0

Ldt is the stationary value with respect to path vari-

ation which vanish at the end points δI = 0. Here L is a function

L (q, q′, t) .q is a n dimensional point q1, q2, · · · , qn.

Proof :

Let ~r1, ~r2, · · · , ~rN be the position vectors of the N particles.

By the form of D’ Alembert’s principle

~r

N∑

i=1

(

~Fi −mi

··

~ri

)

δ~ri = 0

95

Page 96: MSc Maths Optional Paper V

where ~Fi is the applied force acting on the ith particle.

N∑

i=1

~Fiδri −N∑

i=1

mi

··

~riδ~ri = 0 (1)

Virtual work of A.F.workdone = δW =N∑

i=1

~Fiδ~ri.

Now variation in K.E.

δT = δ

[

1

2

N∑

i=1

mi

·

~ri

]

=1

2

N∑

i=1

mi2·

~riδ·

~ri

δT =N∑

i=1

mi

·

~riδ·

~ri (2)

Butd

dt

[

N∑

i=1

mi

·

~riδ~ri

]

=N∑

i=1

mi

··

~riδ~ri +N∑

i=1

mi

·

~riδ·

~ri (3)

whered

dt(δ~ri) = δ

·

~ri = δW + δT

From (2) and (1)

d

dt

[

N∑

i=1

mi

·

~riδ~ri

]

=N∑

i=1

mi

··

~riδ~ri +N∑

i=1

miδ·

~ri

·

~ri

= δT +N∑

i=1

~Fiδ~ri

⇒ δT + δW =d

dt

[

N∑

i=1

mi

·

~riδ~ri

]

(∵ equation (3))

⇒ (δT + δW ) dt = d

[

N∑

i=1

mi

·

~riδ~ri

]

96

Page 97: MSc Maths Optional Paper V

t1∫

t0

(δT + δW ) dt =

t1∫

t0

d

[

N∑

i=1

mi

·

~riδ~ri

]

=

[

N∑

i=1

mi

·

~riδ~ri

]t1

t0

= 0

Now let us assume that the configuration of the system is fixed at the

time t0 to t1 implying the variation δ~ri are zero at these times.

⇒N∑

i=1

[

mi

·

~riδ~ri

]t1

t0

= 0

⇒t1∫

t0

(δT + δW ) dt = 0 (4)

For a given virtual displacement and a time the value of δT and

δW are independent of these Co-ordinates system.

This case of generalized forces equation (4) can be written in the

formt1∫

t0

(

δT +N∑

i=1

Qiδqi

)

dt = 0 (5)

Version of Hamilton’s principle :

Let us assume all the applied forces and derivable from a potentaial

function V (q, t) .

∴ δW = −δV

(4) ⇒t1∫

t0

(δT + δW ) dt =

t1∫

t0

(δT − δV ) dt = 0

=

t1∫

t0

δ (T − V ) dt = 0

=

t1∫

t0

δLdt = 0

97

Page 98: MSc Maths Optional Paper V

= δ

t1∫

t0

δLdt

δI = 0 ⇒ δ

t1∫

t0

Ldt = 0 is Hamilton’s principle.

Hamilton principle and lagrangian equation are equiva-

lent.

(Or)

Derive the lagrangian equation from Hamilton principle.

Solution :

By Hamilton’s principle

δI = 0

where I =

t1∫

t0

L (q, q, t) dt.

Hence L (q, q, t) corresponds to f(y, y, x). Assuming δq′s are inde-

pendent and using Euler Lagrangian equation takes the form

∂L

∂qi− d

dt

(

∂L

∂qi

)

= 0

d

dt

(

∂L

∂qi

)

− ∂L

∂qi= 0 for all i = 1 to n.

Hamilton’s equation or Hamilton’s cononical equation of

motion :

Consider a holonomic system which can be described by the stan-

dard form by of lagrangian equation, namely.

d

dt

(

∂L

∂qi

)

− ∂L

∂qi= 0 , i = 1 to n. (1)

The generalized momentum conjugate to qi is given by

pi =∂L

∂qi(2)

98

Page 99: MSc Maths Optional Paper V

(1) ⇒ d

dt(pi) −

∂L

∂qi= 0

d

dt(pi) =

∂L

∂qi∂L

∂qi= pi (3)

Now, let us define the Hamilton’s function H (q, p, t) for the system

as

H (q, p, t) =n∑

i=1

piqi − L (q, q, t) (4)

H is an explict function of q′s and p′s and t

H (q, p, t) =N∑

i=1

piqi − L (q, q, t)

δH =n∑

i=1

∂H

∂qiδqi +

n∑

i=1

∂H

∂pi

δpi +∂H

∂tδt

=n∑

i=1

piδqi +n∑

i=1

∂H

∂pi

δpi −n∑

i=1

∂L

∂qiδpi −

n∑

i=1

∂L

∂qiδqi −

∂L

∂tδt

n∑

i=1

∂H

∂qiδqi+

n∑

i=1

∂H

∂pi

δpi+∂H

∂tδt =

n∑

i=1

piδqi+n∑

i=1

qiδpi−n∑

i=1

piδqi−∂L

∂tδt

Equating co-efficients on both sides

δpi ⇒ ∂H

∂pi

= qi

δqi ⇒ ∂H

∂qi= −pi;

∂L

∂t= −∂H

∂t

∂H

∂pi

= qi;∂H

∂qi= −p (5)

∂L

∂t= −∂H

∂t

The 2n first order Lagrangian equations is given the set of equation

(5) is called Hamilton equationn of motion.

Hamiton’s funciton is generalized quadratic in the p’s or

99

Page 100: MSc Maths Optional Paper V

the form of Hamiltons’s function.

Proof :

We have

∂T

∂qi= pi =

n∑

j=1

mij (q, t) qj + ai (q, t)

=n∑

j=1

mij qj + ai

n∑

i=1

piqi =n∑

i=1

n∑

j=1

mijqiqj +n∑

i=1

qiqi = 2T2 + T1

We know that

H =n∑

i=1

piqi − L (q, q, t)

=n∑

i=1

piqi − (T − V )

= 2T2 + T1 − T1 − T2 − T0 + V

= T2 − T0 + V

T2 =1

2

n∑

i=1

n∑

j=1

mij qiqj.

This can be put in the matrix form as follows

T2 =1

2mT qq.

But q = b (p− a) bm−1. Both b,m are symmetric matrix.

∴ bT = b,m = mT

∴ T 2 =1

2bT (p− a)T bb−1 (p− a)

=1

2b (p− a) (p− a)T

100

Page 101: MSc Maths Optional Paper V

Expanding this

T2 =1

2

n∑

i=1

n∑

j=1

bijpipj −n∑

i=1

n∑

j=1

bijaipi +1

2

n∑

i=1

n∑

j=1

bijaiaj

T0, V are function of q′s and t adding we have

H (q, p, t) =1

2

n∑

i=1

n∑

j=1

bijpipj−n∑

i=1

n∑

j=1

bijaipi+1

2

n∑

i=1

n∑

j=1

bijaiaj−T0+V

Grouping term by their degree

H = H2 +H1 +H0

H2 =1

2

n∑

i=1

n∑

j=1

bijpipj

H1 = −n∑

i=1

n∑

j=1

bijpjai

H0 =1

2

n∑

i=1

n∑

j=1

bijaiaj − T0 + V

Hence the Hamilton’s function is in general quadratic in p’s.

The Hamilton function of scleronomic system is equal to

toal energy.

Let us consider a system of cartesian co-ordinates do not certain

time t explicity.

It follows that a’s all zero and T = T2.

In sceleronomic system T0 = T1 = 0.

H2 = T2

∴ T1 = T2 and H1 = 0;H0 = V

i.e., H = T + V

101

Page 102: MSc Maths Optional Paper V

∴The Hamilton’s function of sceleronomic system is equal to T.K.

∴ H (q, p, t) =1

2

n∑

i=1

n∑

j=1

bijpipj + V (q, t)

Non - holonomic system has a constant Hamilton’s func-

tion.

Solution :

Let us have a conservative holonomic system we haveH = H (q, p, t) .

dH

dt= H =

n∑

i=1

([

∂H

∂qi

∂qi∂t

]

+

[

∂H

∂pi

∂pi

∂t

])

+

[

∂H

∂t

∂t

∂t

]

H =n∑

i=1

([

∂H

∂qiqi

]

+

[

∂H

∂pi

pi

])

+∂H

∂t

Using Hamilton’s canonical equation

∂H

∂pi

= qi,∂H

∂qi= −pi (A)

and∂H

∂t= −∂L

∂t

∴ H =n∑

i=1

(−piqi + piqi) −∂L

∂t= −∂L

∂t.

If the system is conservative. L does not contain t explicity.

∂L

∂t= 0 ⇒ H = 0

∴ H = C ( constant)

Hence the proof.

Phase space :

Hamilton’s canonical eqn of motion consisting of a set of 2n first

order equation giving q, p as function q’s, p’s are after considered as

components of a single vector X.

Thus the equation of motion for a standard holonomic system can

102

Page 103: MSc Maths Optional Paper V

be written as x = X (x, t) where X is a 2n dimensional vector consist-

ing of nq’s and np’s.

Consider the q’s as x1, x2, · · · , xn and p’s as xn+1, xn+2, · · · , x2n.This

2n dimensional x space is called phase space.

Extended phase space :

In a non-holonomic conservative system, let us consider the variable

qn+1 as choose a parameter q as the new independent variable. This

system consist of (n+ 1)q’s and corresponding (n+ 1)p’s.

This space is called extened phase space.

Autonomous - Non autonomous :

In a conservative system H is not explicit function of time and

configuration of motion of form x = X (x) .

A holonomic system described by equation of this form is called

autonomous where as x = X (x) is called non autonomous.

Liouville’s theorem :

The phase fluid incompressibel.

Proof :

Let a holonomic system be described by n independents q’s.

Let us consider a gropp of phase p’s described fraectorcies in a 2n

dimensional phase space.

Considering the moving points in a small elementry volume dV =

dq1, dq2, · · · , dqn; dp1, dp2, · · · , dpn.

Consisting the moving points of a fluid being called phase fluid.

The phase velocity v of a fluid. Particle is given by the 2n component

(qi, pi) these can be expresed as function of q’s and p’s by canonical

equation

qi =∂H

∂pi

, pi = −∂H∂qi

As a given volume elements of the phase fluid moves it will ingeneral

change in it shape. But the neighbouring particles will remain close to

each other. We can show that volume of each phase fluid is constant

103

Page 104: MSc Maths Optional Paper V

during the motion.

∇~v =n∑

i=1

[

∂qiqi +

∂pi

pi

]

=n∑

i=1

[

∂qi

(

∂H

∂pi

)

+∂

∂pi

(

−∂H∂qi

)]

=n∑

i=1

[

∂2H

∂qi∂pi

− ∂2H

∂pi∂qi

]

= 0

∇~v = 0

A geometric interpretation of this results that the phase fluid is

incompressible.

Kepler’s problem :

Use the Jacobi form of the principle of least action. Obtaing the

orbit for the Kepler’s problem.

Solution :

Let a particle of mass m be attracted to a fixed point O by an

inverse square force.

V = −µmr, Fr = −µm

r2

T =1

2mv2 =

1

2m(

r2 + r2θ2)

h = T + V

=1

2m(

r2 + r2θ2)

− µm

r(1)

By Jacobi form of principle of least action

δ

2 (h− v)ds = 0

⇒ δ

2(

h+µm

r

)

ds = 0 (2)

104

Page 105: MSc Maths Optional Paper V

we have ds2 = m(

dr2 + r2dθ2)

(

ds

)2

= m

[

(

dr

)2

+ r2

]

= m(

r′2 + r2)

ds

dθ=

m (r′2 + r2)

ds =√

m (r′2 + r2)dθ

⇒ δ

∫√

2(

h+µm

r

)

m (r′2 + r2)dθ = 0

⇒ δ

2m(

h+µm

r

)

(r′2 + r2)dθ = 0

where the end points this is the form of

δ

θ∫

θ0

f (r, r′) dθ = 0

where f (r, r′) =

2m(

h+µm

r

)

(r′2 + r2).

Applying the Euler’s Lagrange’s equation for f (r, r′) we have

d

(

∂f

∂r′

)

− ∂f

∂r= 0.

The corresponding energy integral

∑ ∂L

∂qiqi − L = h

∂f

∂r′r′ − f = C ( constant)

f (r, r′) =

2m (r′2 + r2)(

h+µm

r

)

105

Page 106: MSc Maths Optional Paper V

∂f

∂r′=

2m(

h+µm

r

)

2r′

2

2m (r′2 + r2)(

h+µm

r

)

=2m(

h+µm

r

)

r′

2m (r′2 + r2)(

h+µm

r

)

=

2m(

h+µm

r

)

(r′2 + r2)r′

r′∂f

∂r′− f = C

2m(

h+µm

r

)

(r′2 + r2)r′ · r′ −

2m (r′2 + r2)(

h+µm

r

)

= C

2m(

h+µm

r

)

[

r′2√

(r′2 + r2)−√

(r′2 + r2)

]

= C

2m(

h+µm

r

)

[

r′2 − r′2 − r2

(r′2 + r2)

]

= C

2m(

h+µm

r

)

(r′2 + r2)r2 = C (3)

Now

h = T + V

=1

2m(

r2 + r2θ2)

− µm

r

m(

r2 + r2θ2)

= 2(

h+µm

r

)

⇒ m2

(

r2 + r2θ2)

= 2m(

h+µm

r

)

(4)

r =dr

dt=dr

dt

106

Page 107: MSc Maths Optional Paper V

r =dr

dt= r′θ

θ. =r

r′

r = r′θ

(4) ⇒ m2

(

r′2θ2+ r2θ

2)

= 2m(

h+µm

r

)

m2θ2 (

r′2 + r2)

= 2m(

h+µm

r

)

In (3) use this

2m(

h+µm

r

)

(r′2 + r2)r2 = C

m2θ2(r′2 + r2)

(r′2 + r2)r2 = C

−√

m2θ2r2 = C

−mθr2 = C

θ = − C

mr2

Angular momentum is constant

θ2

=C2

m2r4

To find the equation of orbit

(3) ⇒ −

2m(

h+µm

r

)

(r′2 + r2)r2 = C

Squaring and rearranging we get

2m(

h+µm

r

)

r4 = C2(

r′2 + r2)

C2r′2 = 2m(

h+µm

r

)

r4 − C2r2

107

Page 108: MSc Maths Optional Paper V

r′2 =2mr4

Cdr2

(

h+µm

r

)

− r2

=2mr2

C2

[

hr2 + µmr − C2

2m

]

(

dr

)2

=2mr2

C2

[

hr2 + µmr − C2

2m

]

dr

dθ=

2mr2

C2

[

hr2 + µmr − C2

2m

]

dr√

2mr2

C2

[

hr2 + µmr − C2

2m

]

= dθ

dθ =C√2m

dr√

hr4 + µmr2 − C2r2

2m

θ =C√2m

r∫

r0

dr

r2

h+µm

r− C2

2m

=

r∫

r0

d

(

µm2

C2− 1

r

)

µ2m4

C4+

2mh

C2−(

µm2

C2− 1

r

)

θ = sin−1

(

µm2

C2− 1

r

)

µ2m4

C4+

2mh

C2

r

r0

= sin−1

(

µm2

C2− 1

r

)

µ2m4

C4+

2mh

C2

− π

2

sin(

θ +π

2

)

=

µm2

C2− 1

r√

µ2m4

C4+

2mh

C2

108

Page 109: MSc Maths Optional Paper V

cos θ

µ2m4

C4+

2mh

C2=

µm2

C2− 1

r

1

r=

µm2

C2−√

µ2m4

C4+

2mh

C2cos θ

Multiplying byC2

µm2we get

C

µm2/r= 1 −

1 +2hc2

µ2m3cos θ

This is a conic with eccentricity

i.e.,

1 +2hc2

µ2m3

(

r= 1 + e cos θ

)

To find h :

h =1

2m(

r2 + r2θ2)

− µm

r

At r = r0 = rmin, θ = θ0 = 0

m

2

[

0 + r2θ2

0

]

= h+µm

r0

Also

mr20θ0 − C ⇒ θ0 =

−Cmr2

0

(1) ⇒ m

2

[

r20

C2

m2r40

]

= h+µm

r0C2

2mr20

= h+µm

r0

h =C2

2mr20

− µm

r0

Example Given a holonomic system with Lagrangian form L =1

2m (x2

1 + x22 + x2

3)−mgx0 and a constraint x1 − x2 + x3 = 0. We argumented Lagrangian

function to obtain the differential equation of motion for x1.

109

Page 110: MSc Maths Optional Paper V

Solution :

L =1

2m(

x21 + x2

2 + x23

)

−mgx0

constraint:

x1 − x2 + x3 = 0

λ = L+ µg

=1

2m(

x21 + x2

2 + x23

)

−mgx3 + µ (x1 − x2 + x3)

d

dt

(

∂λ

∂x1

)

− ∂λ

∂x1

= 0

we have

d

dt(mx1 + µ) = 0

mx1 +dµ

dt= 0 (1)

d

dt

(

∂λ

∂x2

)

− ∂λ

∂x2

= 0

d

dt(mx2 − µ) = 0

mx2 −dµ

dt= 0 (2)

d

dt

(

∂λ

∂x3

)

− ∂λ

∂x3

= 0

d

dt(mx3 + µ) +mg = 0

mx3 +mg +dµ

dt= 0 (3)

(1), (2) and (3) are the differential equation of motion. Itreating µ

are of the we haved

dt

(

∂λ

∂µ

)

− ∂λ

∂µ= 0

x1 − x2 + x3 = 0 (4)

110

Page 111: MSc Maths Optional Paper V

Adding (1) and (2), we get

x1 + x2 = 0 ⇒ x1 = −x2

Adding (2) and (3) we get

x2 + x3 = 0 ⇒ x2 = −x1 (*)

(∗) ⇒ x1 − x3 = g

x3 = −g − x2

= −g + x1

x1 + x1 + x1 − g = 0

3x1 = g

x1 =g

3

(4) ⇒ x1 + x3 = x2

x3 = x2 − x1

= −x1 − x1

= −2x1 = −2g

3

Definition : Action In mechanics action is deficed as

A =

t1∫

t0

n∑

i=1

piqidt.

State and prove Least action or Principle of least action

The actual path of a conservative holonomic system is such that

the action is stationary with respect to varied pathes having the same

energy integral and the same end points in a ‘q’ space.

Proof :

In this case we are considering a more general type of variation. In

111

Page 112: MSc Maths Optional Paper V

this variation the point (q + δq, t+ δt) corresponds to (q, t).

Let ‘d’ indicates differential change along individual path.

δ indicates variations in going from actual path to varied path

considering the small quadirlateral in the above figure.

qdt+ δq + dδq = δq + (q + δq) (dt+ dδt)

qdt+ dδq = qdt+ qdδt+ δqdt+ δqdδt

Dividing by dt

dδq = qdδt+ δqdδt+ δqdtd

dt(δqi) = qi

d

dt(δt) + δqi + δqi

d

dt(δt)

Omitting δqid

dt(δt)

d

dt(δqi) = qi

d

dt(δt) + δqi

δqi =d

dt(δqi) − qi

d

dt(δt) (1)

Let I =

t1∫

t0

L (q, q, t) dt.

δI = δ

t1∫

t0

Ldt =

t1∫

t0

δLdt+

t1∫

t0

Ld (δt)

=

t1∫

t0

[

n∑

i=1

∂L

∂qiδqi +

∂L

∂qiδqi

+∂L

∂tδt+ L

d

dt(δt)

]

dt

=

t1∫

t0

[

n∑

i=1

∂L

∂qiδqi +

∂L

∂qi

[

d

dt(δqi) − qi

d

dt(δt)

]

+∂L

∂tδt+ L

d

dt(δt)

]

dt

112

Page 113: MSc Maths Optional Paper V

=

t1∫

t0

[

n∑

i=1

∂L

∂qiδqi +

[

∂L

∂qi

d

dt(δqi)

]

− ∂L

∂qiqid

dt(δt)

+∂L

∂tδt+ L

d

dt(δt)

]

dt

Consider

d

dt

[

n∑

i=1

∂L

∂qi

d

dt(δqi)

]

=n∑

i=1

d

dt

(

∂L

∂qi

)

d

dt(δqi)

n∑

i=1

(

∂L

∂qi

)

d

dt(δqi) =

d

dt

[

n∑

i=1

(

∂L

∂qi

)

d

dt(δqi)

]

−n∑

i=1

d

dt

(

∂L

∂qi

)

δqi

Using this value in above equation we get,

δI =

t1∫

t0

[

n∑

i=1

∂L

∂qiδqi +

d

dt

[

n∑

i=1

(

∂L

∂qi

)

δqi

]

−n∑

i=1

d

dt

(

∂L

∂qi

)

δqi

−n∑

i=1

∂L

∂qiqid

dt(δt) +

∂L

∂tδt+ L

d

dt(δt)

]

dt

=

t1∫

t0

d

dt

(

n∑

i=1

∂L

∂qiδqi

)

dt−t1∫

t0

n∑

i=1

[

d

dt

(

∂L

∂qi

)

− ∂L

∂qi

]

δqidt

−t1∫

t0

[

n∑

i=1

∂L

∂qiqi − L

]

d

dt(δt) dt+

t1∫

t0

∂L

∂tδtdt (*)

= I1 − I2 − I3 + I4

where

I1 =

t1∫

t0

d

dt

(

n∑

i=1

∂L

∂qiδqi

)

dt

⇒[

n∑

i=1

∂L

∂qiδqi

]t1

t0

= 0

113

Page 114: MSc Maths Optional Paper V

I2 =

t1∫

t0

n∑

i=1

[

d

dt

(

∂L

∂qi

)

− ∂L

∂qi

]

δqidt = 0

I4 =

t1∫

t0

∂L

∂tδtdt = 0

I3 = −t1∫

t0

[

n∑

i=1

∂L

∂qiqi − L

]

d (δt) = δI

Every varied path having the same evergy integral h.

n∑

i=1

∂L

∂qiqi − L = h

δI = −t1∫

t0

hd (δt) = −h (δt)t1t0

δI = −h (δt1 − δt0) (a)

Let us define the action

A =

t1∫

t0

n∑

i=1

piqidt

=

t1∫

t0

n∑

i=1

∂L

∂qiqidt

=

t1∫

t0

(L+ h) dt

δA = δ

t1∫

t0

Ldt+ δ

t1∫

t0

hdt

= δI +

t1∫

t0

δhdt+

t1∫

t0

hd (δt)

= δI + δh (t1 − t0) + h (δt1 − δt0)

= δI + δh (t1 − t0) − δI using (a)

114

Page 115: MSc Maths Optional Paper V

h is same as in the actual path and varied path δh = 0

∴ δA = 0

∴ The P.L.A. δA = δ

t1∫

t0

n∑

i=1

piqidt = 0

Hence the theorem.

Derive Hamilton principle form of least action

Solution :

Write down upto form the above.

Let us assume that the variation be contemperaneous. In this case

δt = 0.

∴ Second integral vanished.

Let us assume all the applied force are derivable from a potential

function.

Hence the least integral vanishes.

Let us assume all the varied paths have fixed end points in a q

space.

Hence the I, integral vanishes.

∴ δI = 0

This is Hamilton principle.

Obtain Jacobis form of principle of least action

Solution :

Write the bookwork Least action completely let us assume that the

system be natural.

Now

n∑

i=1

piqi − L = H

n∑

i=1

piqi = L+H

= T2 − T0 + V + T − V

115

Page 116: MSc Maths Optional Paper V

= T2 − T0 + T

= T2 − T0 + T2 + T0 + T1

= 2T2 + T1 [∵ T + v = h]

= 2T + 0

n∑

i=1

piqi = 2T

For the natural system T2 = T, T1 = 0, T0 = 0.

Least action = δA = δ

t1∫

t0

n∑

i=1

piqidt = 0

A =

t1∫

t0

2Tdt = 0

δA = δ

t1∫

t0

2Tdt = 0

Define ds as follows

ds2 =∑

i

j

mij qiqjdt2

= 2Tdt2

ds =√

2Tdt

δA = δ

t1∫

t0

√2T

√2Tdt

= δ

t1∫

t0

√2Tds

= δ

t1∫

t0

2 (h− v)ds = 0

This is the principle of least action in Jacobi’s theorem.

116

Page 117: MSc Maths Optional Paper V

Problem :

Given a mass spring system consisting of a mass m and a linear

spring of stiffness of k as shown in gigure. Find the equation of motion

using the Hamilton’s procedure. Assume that the displacement x is

measure from the unstressed position of the spring.

Solution :

First we find the K.E and P.E in the usual form for mass spring

system we obtain

T =1

2mx2

V =1

2x2k

where k is the stiffness.

But we know that

L = T − V =1

2mx2 − 1

2x2k

Angular momentum P =∂L

∂x=

1

2m2x

= mx [ here x is only the co-ordinates q = x]

P = mx

x =P

m

T =1

2mx2 =

P 2

2m

The Hamilton function is

H (x, p) = px− L

= p (pm) −[

1

2m (pm)2 − 1

2kx2

]

=p2

m− p2

2m+

1

2kx2

117

Page 118: MSc Maths Optional Paper V

=p2

m+

1

2kx2 (1)

H (q, p, t) = H (x, p)

Using Hamilton canonical equation

qi =∂H

∂p; x =

∂H

∂p=

2

2mp =

p

m

⇒ p = −∂H∂x

= −(

1

2k · 2x

)

= −kx

Using (1)

∴ p = −kx (2)

But

p = mx

p = mx (3)

From (2) and (3)

mx = −kxmx+ kx = 0

This is equation of the mass spring system.

Kepler’s problem using Hamilton canonical function.

A particle of mass m is attached to a fixed point ‘O’ by an inverse

square force that is Fr = −µmr2

where µ is the gravitational co-efficient

using the plane co-ordinates (r, θ) to describe the position of the par-

ticle.

Find the equation of motion

qi =∂H

∂pi

; p i = −∂H∂qi

.

118

Page 119: MSc Maths Optional Paper V

Solution:

T =1

2m(

r2 + r2θ2)

V =

Frdr =

−µmr2dr

The Lagrange’s function

L = T − V

H = T + V

=1

2m(

r2 + r2θ2)

− µm

r

=1

2m

(

p2r

m2+ r2 p2

θ

m2r4

)

− µm

r

=p2

r

2m+

p2θ

2mr2− µm

r

Using the Hamilton equation we have

r =∂H

∂pr

=2pr

2m=pr

m(1)

pr = −∂H∂r

= −[

1

2mp2

θ (−2) r3 +µm

r2

]

=p2

θ

mr3− µm

r2(2)

From (1)

r =pr

m⇒ r =

pr

m

pθ = −∂H∂θ

⇒ r − 1

m

[

p2θ

mr3

]

+1

m

[µm

r2

]

= 0

⇒ mr − p2θ

mr3+µm

r2= 0

119

Page 120: MSc Maths Optional Paper V

pθ is constant say β

⇒ mr − β2

mr3+µm

r2= 0

pr = mr

pr = mr (3)

From equation (1) and (2)

mr =β2

mr3− µm

r2

mr − β2

mr3+µm

r2= 0

This is the equation of motion.

Problem

Using the Hamilton equation discuss the motion for a charged par-

ticle in an electro magnetic field.

Solution :

The Lagrangian function is

T =1

2mv2

V = ρ(

φ− v · A)

L = T − V

=1

2mv2 − ρ

(

φ− v · A)

Momentum

p = mv + ρA

mv = p− ρA

v =p− ρA

m

Assume ρ is the charge our the particle Hamilton H is given by

H = pv − L

120

Page 121: MSc Maths Optional Paper V

= pv − 1

2mv2 + ρ

(

φ− vA)

=(

mv + ρA)

v − 1

2mv2 + ρφ− ρvA

= mv2 + ρAv − 1

2mv2 + ρφ− ρvA

=1

2mv2 + ρφ

=1

2m

(

p− ρA

m

)2

+ ρφ

=1

2m(p− ρA)2 + ρφ

=1

2m

[

(px − ρAx)2 + (py − ρAy)

2 + (pz − ρAz)2]

+ ρφ

x =∂H

∂px

=1

2m[2 (px − ρAx)]

=1

m(px − ρAx)

Similarly

y =1

m(py − ρAy)

z =1

m(pz − ρAz)

∴ ~v =1

m

(

~p− ρ ~A)

The second canonical equation

x =∂H

∂px

px = −∂H∂x

= − 1

2m

[

2 (px − ρAx)

(

−ρ∂Ax

∂x

)

+ 2 (py − ρAy)

(

−ρ∂Ay

∂x

)

+2 (pz − ρAz)

(

−ρ∂Az

∂x

)]

+ ρ∂φ

∂x

121

Page 122: MSc Maths Optional Paper V

= −ρ∂φ∂x

m

[

(px − ρAx)∂Ax

∂x+ (py − ρAy)

∂Ay

∂x

+ (pz − ρAz)∂Az

∂x

]

(1)

Similarly

py = −ρ∂φ∂y

m

[

(px − ρAx)∂Ax

∂y+ (py − ρAy)

∂Ay

∂y+ (pz − ρAz)

∂Az

∂y

]

pz = −ρ∂φ∂z

m

[

(px − ρAx)∂Ax

∂z+ (py − ρAy)

∂Ay

∂z+ (pz − ρAz)

∂Az

∂z

]

But

∇(

~v · A)

= ∇ [(xi + yj + zk) · (Axi + Ayj + Azk)]

= ∇ (xAx + yAy + zAz)

=∑

i

[

x∂

∂xAx + y

∂yAy + z

∂zAz

]

Substitute the values of x, y , z, we get

∇(

~v · A)

=∑ 1

mi

[

(px − ρAx)∂Ax

∂x+ (py − ρAy)

∂Ay

∂x

+ (pz − ρAz)∂Az

∂x

]

=∑

i

[

px

ρ+∂φ

∂x

]

=

(

px

ρ+∂φ

∂x

)

i+

(

py

ρ+∂φ

∂y

)

j +

(

pz

ρ+∂φ

∂z

)

k

=p

ρ+ ∇φ

p = ρ∇(

~v · ~A)

− ρ∇φ

Hence the proof.

Lagender’s Transformation :

122

Page 123: MSc Maths Optional Paper V

Derive Hamilton’s cononical equation using Legender transforma-

tion and Lagrange’s equation.

Solution : Suppose the funcion F (u1, u2, · · · , un, ω1, ω2, · · · , ωm, t)

is associated with the given system.

Hence u’s are active variables and ω’s and t are passive variable.

Let

vi =∂F

∂ui

(i = 1, 2, · · · , n) . (1)

Let n×m Heissian determinant for the transformation be non-zero.

Then∣

∂2f

∂ui∂uj

=

∂uj

(

∂F

∂ui

)∣

=

∂vi

∂uj

6= 0.

Now let us define a new function G (v1, v2, · · · ., vn, ω1, ω2, · · · , ωn, t)

according to the equation

G =n∑

i=1

uivi − f (u, ω, t) . (2)

Consider a variation ∂G associated with arbitrary variation of the

active variable.So

δG =n∑

i=1

∂G

∂vi

δvi

=n∑

i=1

[

uiδvi + viδui −∂F

∂ui

δui

]

orn∑

i=1

∂G

∂vi

δvi =n∑

i=1

[

uiδvi +

(

viδui −∂F

∂ui

)

δui

]

where δu’s and δv’s are assumed to the arbitrary equation .

Equating the co-efficient on both sides

ui =∂G

∂vi

. (3)

For a holonomic system

123

Page 124: MSc Maths Optional Paper V

The Lagrange’s equation is

pi =∂L

∂qi, i = 1, 2, 3, · · · , n (4)

where pi =∂L

∂qi, i = 1, 2, 3, · · · , n.

Let us choose L (q, q, t) corresponding to the function F (u, ω, t) .

Also we have∣

∂2L

∂qi∂qj

= |mij| 6= 0.

Since the inertia matrix is positive definite matrix.

Let G (u, ω, t) corresponding to the Hamilton’s funciton

H (p, q, t) =n∑

i=1

piqi − L

where qi =∂H

∂pi

.

∂G

∂vi

= ui

∂H

∂qi(q, p, t) = −∂L

∂qi(q, q, t) (5)

From (4) and (5)∂H

∂qi= −pi ⇒ pi = −∂H

∂qi

Problem:

Obtain Hamiton cononical eqn from modified Hamilton’s principle.

Solution :

Consider a holonomic system having n independent q’s.

By Hamilton’s principle, δI = 0 where I =

t1∫

t0

Ldt.

δI = δ

t1∫

t0

Ldt

124

Page 125: MSc Maths Optional Paper V

we have

h = T + V, L = T − V

H =∑

piqi − L

L =∑

piqi −H (p, q, t)

δ

t1∫

t0

Ldt = δ

t1∫

t0

[

piqi −H (p, q, t)]

dt

0 =

t1∫

t0

[

n∑

i=1

(piδqi + qiδpi) −n∑

i=1

(

∂H

∂pi

δpi +∂H

∂qiδqi −

∂H

∂tδt

)

]

dt

125

Page 126: MSc Maths Optional Paper V

UNIT IV

Hamilton -Jacobi theorem

Integrals of Motion :

In a holonomic system consisting of n generalized co-ordinates.

We have n second order non-linear differential equation with time as

independent variables. Any general analytic solution of this differential

equation contains one method of expression the general solution.

ie, To obtain ∂R independent funciton of the form

fi(q, q, t) = γj, j = 1, 2, 3, · · · , 2n

We have the γj’s are arbitrary constant.

These two functions are called the integral.

The value of each γ’s are depending on the initial condition. In

principle these 2n equations can be solved for the q’s as function of γ

and t.

It is possible to find q’s and q’s as a function of γ and t.

∴It is possible to find

qi = qi (γ1, γ2, · · · , γ2n, t)

qi = qi (γ1, γ2, · · · , γ2n, t)

1. Corresponding Hamilton’s Principle function. Solve the Hamil-

ton poblem giving the motion in phase space as function of time.

Solution :

Let

I =

t1∫

t0

Ldt.

Consider the cononical integral which is associating with Hamil-

ton’s principle.

126

Page 127: MSc Maths Optional Paper V

We want to evaluate this integral over actual dynamical path of a

holonomic system that obeys standard Lagrange’s equation or Hamil-

ton’s equation. If 2n independent initial conditions as specified at the

time t0 any final time t1.

Let us consider the solution of equations

qi1 = qi1 (q0, q0, t0, t1) , i = 1, 2, 3, · · · , n.

Let us assume Jacobian

∂ (q11, q12, · · · , q1n)

∂ (q10, q20, · · · , qn0)

is non zero. Hence we get

qi0 = ηi (q0, q1, t0, t1) , i = 1, 2, 3, · · · , n

We can evaluate the integral

t1∫

t0

Ldt as a funciton of (q0, q0, t0, t1) .

Then subsititution for qi0 from the above equation we have

s = s (q0, q1, t0, t1) =

t1∫

t0

Ldt.

The function s (q0, q1, t0, t1) obtain from the canonical equation.

Required form to be obtained.

Let us assume twice differentiation in all its arguments. This is

called Hamilton’s principle function.

Let us consider a non contemporaneous system general variation

of the canonical integral with the reference in the actual solution.

We have,

H −H (q1, p, t)

dH

dt=

n∑

i=1

[

∂H

∂qi

dqidt

+∂H

∂pi

dpi

dt

]

+∂H

∂tdt

127

Page 128: MSc Maths Optional Paper V

=n∑

i=1

[

∂H

∂qiqi +

∂H

∂pi

pi

]

+∂H

∂t(1)

From Hamilton’s canonical equation we have

qi =∂H

∂pi

, pi = −∂H∂qi

, i = 1, 2, · · · , n

∂L

∂t= −∂H

∂t

(1) ⇒ H =∂H

∂t= −∂L

∂t

We know that from the principle of least action

δI =

t1∫

t0

n∑

i=1

d

dt

(

∂L

∂qi

)

δqidt+

t1∫

t0

∂L

∂tδt−

(

n∑

i=1

∂L

∂qiδqi − L

)

d

dt

dt

−t1∫

t0

[

n∑

i=1

d

dt

(

∂L

∂qi

)

− ∂L

∂qi

]

δqidt

=

t1∫

t0

n∑

i=1

d

dt(piδqi) dt+

t1∫

t0

(

−∂H∂t

δt−Hd

dt(δt)

)

dt

=

t1∫

t0

n∑

i=1

d

dt(piδqi) dt −

t1∫

t0

d

dt(Hδt) dt

As our assumtion δS = δI.

∴ δS =

[

n∑

i=1

piδqi −Hδt

]t1

t0

dS =n∑

i=1

piδqi −t1∫

t0

d

dt(Hδt) dt.

This is difference of two forces.

(By writting the above quation in differential form)

s = s (q0, q1, t0, t1)

128

Page 129: MSc Maths Optional Paper V

ds =n∑

i=1

(

∂s

∂qi1δqi1 +

∂s

∂qi0δqi0

)

+∂s

∂t1δt1 +

∂s

∂t0δt0 (2)

The (2n+2) arguments of the principle funcitons are independent.

Hence the corresponding co-efficients in (1) and (2) are equal.

pi1 =∂s

∂qi1, pi0 = − ∂s

∂qi0, i = 1, 2, 3, · · · ., n

H1 = − ∂s

∂t1;H0 =

∂s

∂t0

Now, we get pi0 as function f (qi0, qi1, t0, t1) .

Assuning∣

∂2s

∂qi0∂qj1

6= 0

We can solve for each qi1as a funciton of the initial condition

i.e., qi1 = qi1 (q0, p0, t0, t1) , i = 1, 2, 3, · · · , n.

If this is substituted in pi1 =∂s

∂qi1, we have

pi1 = pi1 (q0, p0, t0, t1) .

This completes the solution of the Hamilton’s problem. Note :

(i)

t1∫

t0

Ldt is a fucntion (qi1, qi1, t0, t1) but qi1, qi1 are the funciton of

q0, q0, t0, t1,

(ii) In the whole discussion first finding the principle funcion without

knowing of solution.

Pfaffian Differential forms :

i) A pfaffian form gΩ is the m variables x1, x2, · · · , xm can be writ-

ten as

Ω = X1 (x) dx1 + · · · +Xm (x) dxm.

129

Page 130: MSc Maths Optional Paper V

Let us define Cij =∂Xi

∂xj

− ∂Xj

∂xi

ii) If the pfaffian form is an exact differentiation then all the ds are

zero

we have

ds =n∑

i=1

pi1dqi1 −n∑

i=1

pi0dqi0 −H1dt1 −H0dt0.

The R.H.S consists of twon variables are p’s and q’s each expression

can be written as

p1q1 + p2q2 + · · · + pnqn + 0 · dp1 + · · · + 0 · dpn − 1 + h (q, pt) dt

Now, we have m = 2n+ 1 (odd) .

By pfaffican system these equation are of the form

n∑

i=1

Cijdxi = 0, j = 1, 2, 3, · · · ,m.

Applying this equation to the differential equaiton,we have

dqj −∂H

∂pj

dt = 0 (1)

−dpj −∂H

∂qjdt = 0 (2)

(1) and (2) can be written as

(1) ⇒ qj =∂H

∂pj

(2) ⇒ pj = −∂H∂qj

, j = 1, 2, 3, · · · , n

H = H (q, p, t)

∂H

∂t=

n∑

i=1

[

∂H

∂qiqi +

∂H

∂pi

pi

]

+∂H

∂t

These are the Hamilton’s canonical equation.

130

Page 131: MSc Maths Optional Paper V

Also using (2), we have

H =∂H

∂t.

Now let us generalized the differential form

ds =n∑

i=1

pi1dqi1 −n∑

i=1

pi0dqi0 −H1dt1 +H0dt0.

Using the 2n parameter (γ1, γ2, · · · , γ2n) to specify the intial con-

dition in place the q0’s and p0’s in otherwise.

We assume a transformation

qi0 = qi0 (γ11, γ21, · · · , γ2n)

pi0 = pi0 (γ11, γ21, · · · , γ2n)

(3)

such that∂ (q10, q20, · · · , qn0, p10, p20, · · · , pn0)

∂ (γ11, γ21, · · · , γ2n)6= 0

Then using (3) we have

n∑

i=1

pi0dqi0 =2n∑

i=1

pi0

∂qi0∂γj

(4)

By the theorem that the 2n can be replaced by nα’s and nβ’s where

the function

αi = αi (γ11, γ21, · · · , γ2n)

βi = βi (γ11, γ21, · · · , γ2n) , i = 1, 2, 3, · · · , n.

choosen such that

n∑

i=1

βidαi =2n∑

i=1

Γ (γ, dγ)

131

Page 132: MSc Maths Optional Paper V

in the same manner as (4) we have

Γj (γ) =n∑

i=1

βi

∂αi

∂γj

, j = 1, 2, 3, · · · , n.

It can be show that α’s are not unique

n∑

i=1

pi0dqi0 =n∑

i=1

βjdαi

where α’s and β’s are another representation shows that (q0, p0) and

(α, β) are connected by a homogeneouss cannonical transformation at

a time t0.

Hamilton - Jacobi equation :

Consider a holonomic system giving 2n independent initial condi-

tions at time to as q0 6= p0.

Now we have differential equation

Assume that

ds =n∑

i=1

pi1dqi1 −n∑

i=1

pi0dqi0 −H1dt1 +H0dt0 (1)

where s is the Hamilton’s principle funtion. It is associated with

cononical transformation relating the initial and final point of a path

in a phase space.

Let the initial condition be specified by

αi = αi (q10, q20, · · · , qn0, p10, p20, · · · , pn0)

βi = βi (q10, q20, · · · , qn0, p10, p20, · · · , pn0)

which satisfied.n∑

i=1

pi0dqi0 =n∑

i=1

βidαi

∴ ds =n∑

i=1

pi1dqi1 −n∑

i=1

βidαi −H1dt1 +H0dt0 (2)

132

Page 133: MSc Maths Optional Paper V

Now we consider s number can be associated as a function of

s = s (qi1, αi, t1, t0)

ds =n∑

i=1

ds

dqi1dqi1 +

n∑

i=1

∂s

∂αi

dαi +∂s

∂t1dt1 +

∂s

∂t0dt0 (3)

Hence q’s and α’s are independent equality variable.

∴ Equating the corresponding coefficient in ds we have

−βi =∂s

∂αi

, i = 1, 2, 3, · · · , n.

pi1 =∂s

∂qi1, H1 = − ∂s

∂t1, H0 =

∂s

∂t0.

Let us assume initial time t0 = 0 , take the final time at t.

i.e., t1 = t⇒ dt1 = dt

Hence we can drop the subsorip q’s . Hence (1) becomes

ds =n∑

i=1

pidqi −n∑

i=1

βidαi −Hdt. (I)

From this we see that s is a funciton of (q, α, t)

(p is a function of q, q, r, t )

ds =n∑

i=1

∂s

∂qidqi +

n∑

i=1

∂s

∂αi

dαi +∂s

∂tdt (II)

Once again assuming

∂2s

∂qi∂αj

6= 0.

∴ Equating the coefficient in I and II, we have

−βi =∂s

∂αi

, i = 1, 2, 3, · · · , n (4)

pi =∂s

∂qi, i = 1, 2, 3, · · · , n (5)

133

Page 134: MSc Maths Optional Paper V

∂s

∂t= −H (6)

Form (4) we can get q’s as function (α, β, t) using this in equation (5)

we can find p as function of (α, β, t) .

Hence we have the solution for Hamilton’s rpinciple.

H is usually as a function of (q, p, t) . Using (5) and (6) we have

∂s

∂t+H (q, p, t) = 0

∂s

∂t+H

(

q,∂s

∂q, t

)

= 0

This is called Hamilton - Jacobi equaiton.

State and prove Jacobi theorem.

Statement :

If s (q, α, t) is any complete solution of the Hamilton Jacobi equa-

tion

∂s

∂t+H

(

q,∂s

∂q, t

)

= 0

−βi =∂s

∂αi

, i = 1, 2, 3, · · · , n (1)

pi =∂s

∂qi, i = 1, 2, 3, · · · , n

where β’s are arbitrary constant are used to solve for qi (α, β, t) and

pi (α, β, t) .

Then these expression provide the general solution of the canonical

equation associated with Hamilton H (q, p, t) .

OR

Prove that any complete solution of the Hamilton Jacobi

equation leads to a solution of this Hamilton problem.

Proof :∂s

∂t+H

(

q,∂s

∂q, t

)

= 0 (2)

∂s

∂qi= pi, i = 1, 2, 3, · · · , n (3)

134

Page 135: MSc Maths Optional Paper V

which is a function of (q, α, t) . Differentiating w.r.to αi

d2s

dαi∂t+

n∑

j=1

∂H

∂pj

∂pj

∂αi

= 0 (4)

where pj is considered as a function of (q, α, t) in (3).

[p (q, x, t) are independent to each otherds

dqi= pi]

∂s

∂αi

is a function of (q, α, t) and α’s and β’s are constants.

Now,∂s

∂αi

= −βi, i = 1, 2, 3, · · · , n

Taking the total time derivation of this w.r.to t

∂2s

∂t∂αi

+n∑

j=1

∂2s

∂qj∂αi

qj = 0 (5)

∂2s

∂t∂αi

+n∑

j=1

∂qj

(

∂qj∂t

)(

∂s

∂αi

)

dt = 0

(5) and (4) ⇒ Using (1),(2) and (3), we have

n∑

j=1

(

qj −∂H

∂qjαi

)

∂2s

∂qj∂αi

= 0, i = 1, 2, 3, · · · , n (6)

But

∂2s

∂qi∂αj

6= 0.

qj =∂H

∂pj

, j = 1, 2, 3, · · · , n (7)

Now, ∂s∂t

+H (q, p, t) = 0.pi is a function of (q, α, t), differentiating (1)

partially w.r.to qj, we have

∂2s

∂qj∂t+

n∑

i=1

∂H

∂pi

∂pi

∂qj+∂H

∂qj= 0 (8)

135

Page 136: MSc Maths Optional Paper V

(3) ⇒ pj =∂s (q, α, t)

∂qj, j = 1, 2, 3, ..., n.

Taking the total time derivation of this equation we have to differen-

tiating w.r.to t

pj −∂2s

∂t∂qj−

n∑

i=1

∂2s

∂qj∂qiqi = 0 (9)

(6) + (7) ⇒

pj +∂H

∂qj+

n∑

i=1

∂H

∂pi

∂pi

∂qj−

n∑

i=1

∂2s

∂qj∂qiqi = 0

⇒ pj +∂H

∂qj+

n∑

i=1

qi∂pi

∂qj−

n∑

i=1

∂pi

∂qjqi = 0

⇒ pj +∂H

∂qj= 0, pi = − ∂s

∂qi, j = 1, 2, 3, ..., n

pj = −∂H∂qj

Equations (7) and (10) together give the cononical equation of Hamil-

ton. Thus any complete solution of the Hamilton Jacobi equation leads

to a solution of the Hamilton equation.

Modified Hamilton Jacobi equation :

Consider a conservative holonomic system with n independent for

this system H is not an explicit function of time t for this system

H (q, p) = αn = h where h is the value of the Hamilton Jacobi integral.

∂s

∂t+H

(

q,∂s

∂q, t

)

= 0

∂s

∂t+H (q, p) = 0

∂s

∂t= −H = −αn (1)

136

Page 137: MSc Maths Optional Paper V

∴ s can be taken as linera funciton of time t is

s (q, α, t) = −αnt+ ω (q, α) .

Then we have omitted arbitrary additive constant.

The funciton

ω (q, α) = ω (q1, q2, ..., qn, α1, α2, ..., αn) .

does not contain time explicitly and is known as a characteristic func-

tion.

Now,

∂s

∂αi

=∂ω

∂αi

, i = 1, 2, 3, ..., (n− 1) (2)

∂s

∂αn

=∂ω

∂αn

− t (3)

∂s

∂qi=

∂ω

∂qi, i = 1, 2, 3, ..., n (4)

Using (3) and (4), Hamilton jacobi equaiton Induces to

This is the modified Hamilton Jacobi equaiton

Note:

We have

βi =∂s

∂αi

, pi =∂s

∂qi∂s

∂αi

=∂ω

∂αi

, i = 1, 2, 3, ..., (n− 1)

∂s

∂αn

=∂ω

∂αn

− t

i∂s

∂qi=

∂ω

∂qi, i = 1, 2, 3, ..., n

Comparing the equation we have

βi =∂ω

∂qi, i = 1, 2, 3, ..., n

137

Page 138: MSc Maths Optional Paper V

where βn is the intial time to t0.

Ignorable co-ordinates :

Consider a holonomic system described by the standard form

d

dt

(

∂L

∂qi

)

− ∂L

∂qi= 0,

qi is said to be an ignorable co-ordinate if∂L

∂qi= 0.

Also pi =∂L

∂qi= βi only for the ignorable co-ordinate.

Case (i)

Consider a system with ignorable co-ordinates q1, q2, ..., qn .Let us

assume the system is not consecutive. We know that

pi = αi, i = 1, 2, 3, ..., k.

We can assume principle solution is of the form

s (q, α, t) =k∑

i=1

αiqi + s′ (qk+1, ..., qn, α1, α2, ..., αn, t)

Hamilton Jacobi equation leads to

∂s

∂t+H

(

q,∂s

∂q, t

)

= 0

∂s′

∂t+H

(

qk+1, ..., qn, α1, α2, ..., αk,∂s′

∂q′k+1

, ...,∂s′

∂q′n, t

)

= 0.

The complete solution of these equation involve (n − k) non additive

constants exclusive of the known solution of this equation is obtained

form.

−βi = qi +∂s′

∂αi

, i = 1, 2, 3, ..., k

pi = αi, i = 1, 2, 3, ..., k

pi =∂s′

∂qi, i = k + 1, k + 2, k + 3, ..., n

pi = −qi0

138

Page 139: MSc Maths Optional Paper V

−βi =∂s′

∂αi

, i = k + 1, k + 2, k + 3, ..., n

Case (ii)

Consider a system with ignorable co-ordinate q1, q2, ..., , qk and con-

servative with the help of the previous result we have

s (q, α, t) =k∑

i=1

αiqi − αnt+ ω′ (qk+1, ..., qn, α1, α2, αn, t)

The modified Hamilton Jacobi equation becomes

H

(

qk+1, ..., qn, α1, α2, ..., αk,∂ω′

∂q′k+1

, ...,∂ω′

∂q′n

)

= qn

The complete solution for ω′ in this case involve (n− k − 1) non addi-

tive constants αk+1, αk+2, ..., αn−1 the energy constent αn and constant

moments α1, α2, ..., αk.

The motion of the system is given by

−βi = qi +∂ω′

∂αi

, i = 1, 2, 3, ..., k

−βi =∂ω′

∂αi

, i = 1, 2, 3, ..., (k − 1)

t− βn =∂ω′

∂αn

pi = αi (i = 1, 2, 3, ..., k) ⇒ pi =∂ω′

∂qi, (i = k + 1, k + 2, ..., n)

Ordinary Conservation of linear momentum

This follows from Newton’s second law of motion

d

dt(mv) =

dp

dt= F.

If the total force is zero, thendp

dt= 0 at the linear momentum is

conserved.

Note :

The word conservation applied in the of connectedness.

139

Page 140: MSc Maths Optional Paper V

Conservation of angular momentum :

Angular momentum is the angular of linear momentum in the case

of relational motion.

Consider a particle at mass m and the linear momentum p at a

positin vector ~r relative to the origin O at an inertial frame.

Angular momentum l at the particle with respect to the origin O

is L = r × ρ

Moment of a force about a origin is N = r × F then F =dp

dt.

N = r × F ⇒ N = r × dp

dt(1)

⇒ N =d

dt(r × ρi) −

dr

dt× ρ

⇒ N =d

dt(r × ρ) − v ×mv (2)

⇒ N =d

dt(r × ρ) − 0 (3)

∴ N =d

dt(r × ρ) (4)

∴The second term is zero both vectors are parallel now.

N =d

dt(r × ρ)

=dL

dt.

Thus the rate of change of angular momentum of a particle is equal

to the force acting on it is the total force N = 0.

ThendL

dt= 0,angular momentum is considered in the absence of

an external force.

Principle of conservation of evergy :

Let the only force acting on the given particle be conservation is

of the force are derivable from scalar potential evergy function.

F = −∇V, then the T.E.=K.E.+P.E.

Suppose under the action of such force F particles moves from the

140

Page 141: MSc Maths Optional Paper V

position 1 to 2. Then the workdone

ω

2=

2∫

1

Fds

ω =

Fds =

2∫

1

Fds =

2∫

1

dF

dtds

The particle moves ds distance in the time dt with velocity r, so

p = mv = mr.

dr

dt= r, dr = rdt

ω

2=

2∫

1

d

dt(mr) rdt

=

2∫

1

d

dt

(

mr2)

dt

=

2∫

1

2d

dt

(

1

2mr2

)

dt

= T2 − T1

T2, T1 denote the K.E of the particle of the K.E of the position 2 and

1 respectively.

F = −∇V

ω12 =

2∫

1

−∇V dr =

2∫

1

− dV

dxdr

−2∫

1

dV = V1 − V2

141

Page 142: MSc Maths Optional Paper V

From (1) and (2),

T2 − T1 = V1 − V2

T2 + V2 = V1 + T1 = constant

∴ T + V = constant

⇒ T − E of a particle is conserved (constant)

Problem :

For a simple spring system using Hamilton Jacobi method to solve

it.

The K.E and P.E are given by

T =1

2mx2, V =

1

2kx2

P =∂T

∂x=

1

2m2x = mx

x =ρ

m⇒ x2 =

P 2

m2

H = T + V

=P 2

2m+kx2

2

We are considering a conservative systm.

∴ H = T + V = α

The modified homilton Jacobi equation is

1

2m

(

∂ω

∂x

)2

+1

2kx2 = α

(

∂ω

∂x

)2

= 2m

(

α− 1

2kx2

)

142

Page 143: MSc Maths Optional Paper V

where α is energy constant.

∂ω

∂x=

2m

(

α− 1

2kx2

)

=

2m

(

2α− kx2

2

)

=√mk

(

k− x2

)

Taking

a =

mω2and ω =

k

m

∂ω

∂x=

x∫

x0

√mxdx

x∫

x0

dt√

a2 − ξ2

ω (x, α) = mω

x∫

x0

a2 − ξ2dt, ω =

x

m

∂ω

∂α= mω

1

2

2a√a2 − ω2

1

2

ω2m

2

mω2dω

= t− β

=1

ω

x∫

x0

dt√a2 − t2

= − 1

ω

x∫

x0

dt√t2 − a2

= − 1

ω

[

cos−1 t

a

]x1

x0

=1

ω

[

cos−1 x0

a− cos−1 x1

a

]

ρ is initial time t0. i.e., β = t0.

t− t0 =1

ω

[

φ− cos−1

(x

a

)]

ω (t− t0) = φ− cos−1

(x

a

)

143

Page 144: MSc Maths Optional Paper V

φ− ω (t− t0) = cos−1

(x

a

)

− [ω (t− t0) − φ] = cos−1

(x

a

)

x = a cos [ω (t− t0) − φ]

=

mω2cos [ω (t− t0) − φ] .

Let x (t0) = x0. Then x (t0) = v0. We know that

α = T + V

=1

2mx2 +

1

2kx2

=1

2mv2

0 +1

2kω2x2

0

=mω2

2

[

v20

ω2+ x2

0

]

(1)

sinφ =√

1 − cos2 φ

=

a2 − x20

a2

=1

a

a2 − x20

=1

a

mω2− x2

0 using (1)

=1

a

v20

ω2+ x2

0 − x20

=v0

Now,

x = a cos [ω (t− t0) − φ]

= a [cosω (t− t0) cosφ+ sinω (t− t0) sinφ]

=x0

cosφ[cosω (t− t0) cosφ+ sinω (t− t0) sinφ]

= x0 cosω (t− t0) +x0

cosφsinω (t− t0)

v0

= x0 cosω (t− t0) + a sinω (t− t0)v0

144

Page 145: MSc Maths Optional Paper V

= x0 cosω (t− t0) +v0

ωsinω (t− t0)

Differentiating,

x = 0

Amplitute oscillation is given by

a =

v20

ω2+ x2

0

Kepler’s Problem :

Use Hamilton Jacoi equation to analysis the keplers problem or

modified Jacobi method.

Solution :

Suppose a particle of unit mass attacted by an inverse square grav-

itational force at a fixed point ‘O’. The position of a given problem

is given interms of the polar co-ordinates (r, θ) in the plane of the

orbits.

The K.E and P.E. are

T =1

2m(

r2 + r2θ2)

V = −µr

where µ is gravitation coefficient.

Lagrangian function is

L = T − V

=1

2m(

r2 + r2θ2)

r

We find that the generalized momentum are given by

pr =∂L

∂r; θ =

r2

αθ = pθ =∂L

∂θ= r2θ

145

Page 146: MSc Maths Optional Paper V

θ =αθ

r2

The system is natural.

H = T + V

H =1

2

(

p2r +

r2

)

− µ

r= αt (1)

where αt represents constants values of the total energy (1) does not

appear in H.L.

∴ It is ignorable.

∂L

∂θ= 0 ⇒ ∂L

∂θ= p0 = constant = α0

The modified Hamilton Jacobi equation is

1

2

(

∂ω′

∂r

)2

+α2

θ

2r2− µ

r= αt

r is ignorable.

1

2

(

∂ω′

∂r

)2

= αt +µ

r− α2

θ

2r2

(

∂ω′

∂r

)2

= 2

(

αt +µ

r− α2

θ

2r2

)

∂ω′

∂r=

2αt + 2µ

r− α2

θ

r2

ω′ =

r∫

r0

2αt + 2µ

r− α2

θ

r2dr

We have

βi = qi +∂ω′

∂α

θ − θ0 = − ∂ω′

∂αθ

146

Page 147: MSc Maths Optional Paper V

= −r∫

r0

(

− 1

r2

)

2αθ

2

2αt + 2µ

r− α2

θ

r2

dr

= −r∫

r0

αθ

r2

2αt + 2µ

r− α2

θ

r2

dr

= −r∫

r0

dr

r2αθ

2αt

αθ

+ 2µ

rαθ

− 1

r2

= −r∫

r0

dr

r2

2αt

αθ

+2µ

rαθ

− 1

r2

= −r∫

r0

dr

r2

2αt

α2θ

+µ2

α4θ

−(

1

r− µ

αθ

)2

= −r∫

r0

d

(

1

r− µ

αθ

)

r2

2αt + α2θ + µα

α4θ

−(

1

r− µ

αθ

)2

θ − θ0 = cos−1

1

r− µ

αθ√

2αt

α2θ + µ2

α4θ

r

r0

Let θ = θ0 where r = r0.

θ − θ0 = cos−1

[

α2θ − µr

r√

2αt + α2θ + µ2

]

− cos−1

[

−µr0ro

µ2

]

= cos−1

[

α2θ − µr

r√

2αt + α2θ + µ2

]

− cos−1 (−1)

cos θ =−µr + α2

θ

r√

2αt + α2θ + µ2

147

Page 148: MSc Maths Optional Paper V

⇒ θ0 = 0 where r0 = 0√

2αtα2θ + 1 cos θ = −µr + αθ2

2αtα2θ + 1

µ2cos θ =

−µ+αθ2

µ

µ

αθ2

µ

r= 1 +

1 + 2αtα2θ

µ2cos θ

This is the forml

r= 1 + e cos θ.

This the cone whose eccentricity is

e =

µ2 + 2αtα2θ

µ2

This is the equation of a conic section having eccentricity.

r =αθ2

µ

1 +

µ2 + 2αtα2θ

µ2 cos θ

Hence the Kepler’s problem.

Seperability :

The index of seperability associated with the solution of P.D.E by

a solution to that is by expressing the solution interms of integrals

each involving only one variable.

Orthogonal system :

It is conservative holonomic system whose K.E function contains

only squared forms and no produed tems in this variable. If a system

is seperable that is possible to find a charatorstic function ω such that

ω =n∑

i=1

ωi (qi) where each contain only one of the q’s.

Note :

A particularly simple example of a seperable system occure in the

ω all but of the co-ordinates all ignorable.

148

Page 149: MSc Maths Optional Paper V

Liouville’s System :

It is an orthogonal system whose K.E. and P.E. are of the form

T =1

2

[

r∑

i=1

pi (qi)

]

n∑

i=r

[

(qi)2

Ri (qi)

]

(1)

=R1P

21 +R2P

22 + ...+RnP

2n

2 (f1 + f2 + ...+ fn)

=v1 (q1) + v2 (q2) + ...+ vn (qn)

f1 (q1) + f2 (q2) + ...+ fn (qn)

where fi, qi and vi are each function of qi.We assume thatn∑

i=1

fi (qi) > 0

and Ri (qi) > 0.

1

2a = φ2 − ψ

a = 2φ2 − 2ψ

This is identical with (a).

Hence the system is seperable.

Using the pfaffian equation, derive the Hamilton canonical equation

from Hamilton funciton

ds =n∑

i=1

pi1dqi1 −n∑

i=1

pi0dqi0 −H1dt1 +H0dt0

Solution :

Book Work 5: pff equation.

Discuss the Kepler’s probelm using seperability.

Solution :

Let us consider a particle of unit mass p attached towards ‘O’ by

an inverse square gravitational force. Let T be the K,E, of the system.

Let V be the P.E. of the system.

T =1

2

[

r2 + r2θ2+ r2φ

2sin2 θ

]

dr = −µr

149

Page 150: MSc Maths Optional Paper V

L = T − V =1

2

[

r2 + r2θ2+ r2φ

2sin2 θ

]

r

Now

pr =∂L

∂r=

1

22r = r

pθ =∂L

∂θ=

1

2r22θ = r2θ

pφ =∂L

∂φ=

1

2r2 sin2 θ2φ = r2 sin2 θφ

T =1

2

[

p2r +

p2θ

r2+

p2φ

r2 sin2 θ

]

where pr, pθ and pφ are the generalized moments. This system is an

orthogonal system.

∴ we have H = T + V = αt.

The modified Hamilton Jacobi equation is

1

2

(

∂ωr

∂r

)2

+1

2r2

(

∂ωθ

∂θ

)2

+1

2r2 sin2 θ

(

∂ωφ

∂φ

)1

− µ

r= αt

where

ω = ωr (r) + ωθ (θ) + ωφ (φ) (1)

∵ φ is missing from the above equation since φ is ignorable co-

ordinates.

∵ [rφ (φ) = αφ (φ)] ∴ pφ = αφ =∂ωφ

∂φ

∴ (1) × 2r2

⇒ r2

(

∂ωr

∂r

)2

− 2r2

r+ αt

)

+

(

∂ωφ

∂φ

)2

+1

sin2 θ(αφ)

2 = 0 (2)

The first two terms are functions of r only and the last terms are

fuctions of θ.

Hence they are each to seperable constant.

Stackel’s thoorem

150

Page 151: MSc Maths Optional Paper V

Statement : Consider an orthogonal system whose K.E is given by

T =1

2

n∑

i=1

miq2i =

1

2

n∑

i=1

cip2i

where ci (q1, q2, ..., qn) > 0. Stackel’s theorem asserts that this is a

seperable system iff there exists a non-singular matrix n × n [Φij (qi)]

and a column matrix [Ψ (qi)] exists such that

i) CT Φ = (1, 0, ..., 0)

ii) CT Ψ = V, where V (q1, q2, ..., qn) is the P.E. and C is a column

matrix composed of the nc’s.

Proof :

Necesary part :

Let us assume that the general orthogonal system be seperable.

Hence it has a characterstic function ω(q, α) which consist of the

sum of terms of the form ωi (qi, α1, α2, ..., αn) .

This characteristic function is the complete integral of the modified

H.J equation

1

2

n∑

i=1

ci

(

∂ωi

∂qi

)2

+ V = α1 (1)

Here α1 is chosen as the total energy because of the system is assumed

to be separability.

(

∂ωi

∂qi

)2

is a function of (qi, α1, α2, ..., αn) .

We can choose the seperation constants such that the d’s appear

linearly.

The most general form involving the single co-ordinate qi is

(

∂ωi

∂qi

)2

= −2Ψi (qi) + 2n∑

j=1

Φij (qj)αj (I)

where the mumerical co-efficients are chosen for our convenient for our

convenient

151

Page 152: MSc Maths Optional Paper V

Then subustitute (I) in (1)

1

2

n∑

i=1

ci

[

−2Ψi (qi) + 2n∑

j=1

Φij (qj)αj

]

+ V = α1 (2)

−n∑

i=1

ciΨi (qi) +n∑

i=1

n∑

j=1

ciΦij (qj)αj + V = α1

⇒ CT Φ + CT Φα + V = α1.

Comparing the terms containing α’s we find that CT Φ = (1, 0, 0, ..., 0) ,which

we recognise as the first Stackel’s conditions. Similary the term in (3)

which do not involve α’s must sum to zero reading to CT Ψ = V,which

is Stackel’s condition.

Sufficient part :

Define a column matrix a =

(

∂ωi

∂qi

)2

.

The modified Hamilton Jacobi equation (1) can be written as

CT (a) + V = α.

Using second Stacicel’s condition

1

2CTa+ CT Ψ = (1, 0, 0, ..., 0)α (4)

⇒ CT Φ = (1, 0, 0, ..., 0)(

CT Ψ = V)

CT = (1, 0, 0, ..., 0) Φ−1 (5)

Sustituting (5) in (4)

1

2(1, 0, 0, ..., 0) + CT Φ = α1

1

2(1, 0, 0, ..., 0) Φ−1a+ (1, 0, 0, ..., 0) Φ−1Ψ = (1, 0, 0, ..., 0)α

152

Page 153: MSc Maths Optional Paper V

(1, 0, 0, ..., 0)

[

1

2Φ−1a+ Φ−1Ψ

]

= (1, 0, 0, ..., 0)α

1

2Φ−1a+ Φ−1Ψ = α

1

2Φ−1a = α− Φ−1Ψ

Φ (α− Ψ) =1

2a

a = 2 (Φα− Ψ)

This is identical with equation (I).

∴ The system is separable.

Note:

(

∂ωθ

∂θ

)2

+dφ2

sin2 θ= α2

θ

⇒ r2

(

∂ωθ

∂θ

)2

+∂r2

(µαr

r+) = α2

θ

We all now assume that the system is separable.

∂ωr

∂r=

2(µ

r+ dt

)

− α2θ

r2

∂ωθ

∂θ=

α2θ −

dφ2

sin2 θ

which is immediatery integrable and find ωr, ωθ are already found out.

Hence the system is separable and hence reduces of quadratare

ωr + ωθ + ωφ = 0.

Discuss the kepler’s problem using separability.

Let us consider a particle of unit mass p attached towards O by an

iverse square gravitational force.

Let T be the K.E. of the system.

Let V be the P.E. of the system.

153

Page 154: MSc Maths Optional Paper V

T =1

2

[

r2 + r2θ2+ r2φ

2sin2 θ

]

dr = −µr

L = T − V =1

2

[

r2 + r2θ2+ r2φ

2sin2 θ

]

r

Now, we can find generalized moments

pr =∂L

∂r=

1

22r = r

pθ =∂L

∂θ=

1

2r22θ = r2θ

pφ =∂L

∂φ=

1

2r2 sin2 θ2φ = r2 sin2 θφ⇒ φ =

r2 sin2 θ

T =1

2

[

p2r +

p2θ

r2+

p2φ

r2 sin2 θ

]

where pr, pθ and pφ are the generalized moments.

This system is an orthogonal system, ∴ we have H = T + V = dt.

The modified Hamilton Jacobi equation is

1

2

(

∂ωr

∂r

)2

+1

2r2

(

∂ωθ

∂θ

)2

+1

2r2 sin2 θ

(

∂ωφ

∂φ

)1

− µ

r= dt (1)

where ω = ωr (r) + ωθ (θ) + ωφ (φ) .

∵ φ is missing from the above equation since φ is ignorable co-

ordinates..

∵ [rφ (φ) = dφ (φ)] ∴ pφ = αφ =∂ωφ

∂φ

∴ (1) × 2r2

⇒ r2

(

∂ωr

∂r

)2

− 2r2

r+ αt

)

+

(

∂ωθ

∂θ

)2

+1

sin2 θ

(

∂ωφ

∂φ

)2

= 0

The first two terms are functions of r only and the last terms are

fuctions of θ.

154

Page 155: MSc Maths Optional Paper V

Hence they are each to seperable constant.

r2

(

∂ωr

∂r

)2

− 2r2

r+ αt

)

= −αθ2 (2)

(

∂ωθ

∂θ

)2

+1

sin2 θ

(

∂ωφ

∂φ

)2

= αθ2 (3)

(2) ⇒ r2

(

∂ωr

∂r

)2

= −α2θ + 2r2

r+ αt

)

= 2r2

r+ αt

)

− α2θ

∂ωr

∂r=

2(µ

r+ αt

)

− α2θ

r2

(3) ⇒(

∂ωθ

∂θ

)2

= α2θ −

(

∂ωφ

∂φ

)21

sin2 θ

∂ωθ

∂θ=

α2θ −

α2φ

sin2 θ

which is immeadiately integrable and find ωr, ωθ are already found out.

Hence the system is separable and hence reduces to quadrative

ωr + ωθ + ωφ = 0.

Jacobi Theorem :

If s (q, α, t) is any complete solution of the Hamilton Jacobi equa-

tion

∂s

∂t+H

(

q,∂s

∂q, t

)

= 0, βi =∂s

∂αi

, pi =∂s

∂qi, i = 1, 2, 3, ..., n

where β’s are arbitrary constants are used to solve qi (α, β, t) and pi

(α, β, t) . Then these expression provide the general solution of the

canonical equation associated with Hamilton H(q, p, t).

155

Page 156: MSc Maths Optional Paper V

Proof :∂s

∂t+H

(

q,∂s

∂q, t

)

= 0 (1)

Differentiating (1) w.r.to αi, by considering pj as a function of (q, α, t)

∂2s

∂αi∂t+

n∑

j=1

∂H

∂pj

∂pj

∂αi

= 0 (2)

Given∂s

∂αi

= −βi

Differentiating both sides w.r.to t

∂2s

∂t∂αi

+n∑

j=1

∂2s

∂qj∂αi

qj = 0 (3)

(3) − (2) ⇒

n∑

j=1

qj∂2s

∂qj∂αi

− ∂H

∂pi

∂pi

∂αi

= 0

pi =∂s

∂qj

Differentiating w.r.to αi,

∂pi

∂αi

=∂2s

∂αi∂qj

Using this into second term (4) becomes

n∑

j=1

qj∂2s

∂qj∂αi

− ∂H

∂pi

∂2s

∂αi∂qj= 0

n∑

j=1

(

qj −∂H

∂pi

)

∂2s

∂αi∂qj= 0.

Now∣

∂2s

∂αi∂qj

6= 0

156

Page 157: MSc Maths Optional Paper V

∴ Each co-efficeient must be in this

∴ q =∂H

∂pj

.

This is Hamilton first equation.

In order to derive the Hamilton second equation we again differen-

tiate (1) partially w.r.to qj and associated pi = f (q, α, t)

∂2s

∂qj∂t+

n∑

i=1

∂H

∂pi

∂pi

∂qj+∂H

∂qj= 0 (5)

pj =∂s

∂qj⇒ pj =

∂2s

∂qj∂t

pj −∂2s

∂qj∂t−

n∑

i=1

∂2s

∂qj∂qiqi = 0 (6)

(5) + (6) ⇒

n∑

i=1

∂H

∂pi

∂pi

∂qj+∂H

∂qj+ pj −

n∑

i=1

∂2s

∂qj∂qiqi = 0 (7)

But∂H

∂qj= qj and

∂pi

∂qj=

∂2s

∂qi∂qj

∴ First and last terms get called in (8) reduces to

∂H

∂qj+ pj = 0

pj = −∂H∂qj

This is second kind equation.

157

Page 158: MSc Maths Optional Paper V

Unit-V

Canonical transformation

Let us consider only a holonomic system which can be described by

the standard form of either Hamilton’s (or) Lagrange’s equation.

Hamilton’s principle applies to these system the generalized coordi-

nates are q1, q2, . . . , qn . We know that the Hamilton principle is

δI = 0 where I =

t1∫

t0

Ldt

∴ δ

t1∫

t0

L (q, q, t) dt = 0

where the endpoints of the varied paths are fixed in both configuration

space and time.

Let L = T − V.

Now if a new set of coordinates Q1, Q2, . . . , Qn is Q related to the

formal set by a point transformation. The Larangian function is given

by,

L∗

(

Q, Q, t)

= L (q, q, t) = T − V

i.e., L and L∗are equal. Now the Hamilton’s principle function

δI = 0 .

δ

t∫

t0

L∗

(

Q, Q, t)

dt = 0.

Now we consider the Lagrangian equation is

L∗

(

Q, Q, t)

= L (q, q, t) − d

dtφ (q,Q, t)

158

Page 159: MSc Maths Optional Paper V

where φ (q,Q, t) is twice differential. Then,

δI = δ

t∫

t0

L∗

(

Q, Q, t)

dt

= δ

t∫

t0

[

L (q, q, t) − d

dtφ (q,Q, t)

]

dt

= δ

t∫

t0

L (q, q, t) dt− δ

t∫

t0

dϕ (q,Q, t)

= δ

t∫

t0

L (q, q, t) dt− δ [φ (q,Q, t)]tt0

δI = δ

t∫

t0

L (q, q, t) dt

i.e., δq ’s and δQ ’s are zero at the fixed time t0 and t .

⇒ δ

t∫

t0

L∗

(

Q, Q, t)

= 0.

Now Hamilton are described by the equation

H (q, p, t) =n∑

i=1

piqi − L (q, q, t)

K (Q,P, t) =n∑

i=1

piQi − L∗

(

Q, Q, t)

where the generalized moment are given by,

pi =∂L

∂qi, Pi =

∂L∗

∂Qi

qi =∂H

∂pi

, pi = −∂H∂qi

, i = 1, 2, . . . , n.

159

Page 160: MSc Maths Optional Paper V

Here the function L∗

(

Q, Q, t)

.

∴ The Hamilton canonical function

Qi =∂K

∂Pi

, Pi = − ∂H

∂Qi

, i = 1, 2, . . . , n

A transformation form (q, p) to (Q,P ) which preserves the canon-

ical form of the equation of motion is known as canonical transforma-

tion.

Differential forms of Pfaffian differential equation:

Let us consider a Hamilton function H (q, p, t) suppose the trans-

formation equation is of the form,

Qi = Qi (q, p, t) , Pi = Pi (q, p, t) i = 1, 2, . . . , n

where each function is atleast twice differentiable. Now the Lagrangian

equation is given by,

L∗

(

Q, Q, t)

= L (q, q, t) − d

dt[φ (q,Q, t)] (1)

Hamilton function is defied by,

H (q, p, t) =∑

piqi − L (q, q, t) (2)

K (Q,P, t) =∑

PiQi − L∗

(

Q, Q, t)

. (3)

Using (2) and (3) in (1) we get

PiQi −K (Q,P, t) =∑

piqi −H (q, p, t) − d

dt[φ (q,Q, t)]

⇒∑

Pi

d

dt(Qi) −K =

pi

d

dt(qi) −H − d

dt(φ)

⇒ dφ =∑

pidqi −Hdt−∑

PidQi +Kdt (4)

where φ is the generalized function. Equation (4) is called the differ-

ence of two pfaffian differential equation and (4) is an exact differential

equation

160

Page 161: MSc Maths Optional Paper V

Note: The function φ (q,Q, t) is called the generating function for

the transformation.

Book work: Consider Q =√

2qet cos p, P =√

2qe−t sin p .

Show that the above transformation is canonical.

Solution: Given that

Q =√

2qet cos p,

P =√

2qe−t sin p.

We obtained

pδq − PδQ = δψ (1)

δQ = et

[

2δq cos p

2√

2qδq +

2q (− sin p) δp

]

= et

[

cos p√2qδq − sin p

(

2q)

δp

]

(1)⇒

pδq − PδQ = pδq −√

2qe−t sin p

[

et

(

cos p√2qδq − sin p

2qδp

)]

= pδq − sin p cos pδq + 2q sin2 pδp

δψ = [p− sin p cos p] δq + 2q sin2 pδp (2)

which is the differential form,

pdx+ qdy = f (x)

and

∂P

∂y=

∂Q

∂x⇒ ∂P

∂p=∂Q

∂q∂

∂p(p− sin p cos p) = 1 −

[

sin p (− sin p) + cos2 p]

= 1 −[

− sin2 p+ cos2 p]

= 1 − cos 2p

161

Page 162: MSc Maths Optional Paper V

∂p[p− sin p cos p] = 2 sin2 p

∂p[p− sin p cos p] = 2 sin2 p

Similarly,

∂q

[

2q sin2 p]

= 2 sin2 p

∂p[p− sin p cos p] =

∂q

[

2q sin2 p]

∴ The transformation is caonical.

Example: Let us consider the transformation

Q =1

2

(

q2 + p2)

, P = − tan−1

(

q

p

)

Show that the transformation is canonical. Find the new Hamilton

canonical equation.

Solution: Given

P = − tan−1

(

q

p

)

;

Q =1

2

(

q2 + p2)

.

We obtained

pδq − PδQ = dψ (1)

δQ =1

22qδq +

1

22pδp

δQ = qδq + pδp

(1) ⇒

pδq − pδQ = pδq + tan−1

(

q

p

)

(qδq + pδp)

162

Page 163: MSc Maths Optional Paper V

= pδq + tan−1

(

q

p

)

qδq + tan−1

(

q

p

)

pδp

δψ =

[

p+ q tan−1

(

q

p

)]

δq + p tan−1

(

q

p

)

δp

∂p

[

p+ q tan−1

(

q

p

)]

= 1 + q

1

1 +q2

p2

(

− q

p2

)

= 1 + q

(

p2

p2 + q2

)(

− q

p2

)

= 1 − q2

p2 + q2

=p2 + q2 − q2

p2 + q2

=p2

p2 + q2

∂p

[

p+ tan−1

(

q

p

)]

=p2

p2 + q2

∂q

[

p+ tan−1

(

q

p

)]

= p

p

1

1 +q2

p2

(

1

p

)

= p

(

p2

p2 + q2

)(

1

p

)

=p2

p2 + q2

∂p

[

p+ tan−1

(

q

p

)]

=∂

∂Q

[

p+ tan−1

(

q

p

)]

∴The transformation is canonical.

Principle forms of generating functions:

Let us consider a generating function φ (q,Q, t) . The other types

of generating functions namely F2 (q, P, t) , F3 (P,Q, t) and F4 (p, P, t)

163

Page 164: MSc Maths Optional Paper V

. Now we show the relationship of F2 and F1 and again F3 and F4 and

F1 and F4.

The relationship of the generating function F1 , F2 , F3 and F4 are

called the principle of generating function.

Proof: Given that the generating function φ (q,Q, t) . We know

that the differential form

piδqi −∑

PiδQi −Hdt+Kdt = dF1 (q,Q, t) . (1)

Let us consider

QidPi −∑

PidQi = d(

QiPi

)

. (2)

Adding (1) and (2) we get

Pidqi +∑

QidPi −Hdt+Kdt = dF2 (q, P, t) (3)

where dF2 (q, P, t) = dF1 (q,Q, t) + d

(

n∑

i=1

QiPi

)

.

F2 (q, P, t) = F1 (q,Q, t) +∑

QiPi. (4)

Equation (4) gives the relationship in between F1 and F2 . Now the

total differential form of F2 is the form

dF2 =∑ ∂F2

∂qidqi +

∑ ∂F2

∂Pi

dPi +∂F2

∂tdt. (5)

Equating the coefficient in (3) and (5)

Pi =∂F2

∂qi, Qi =

∂F2

∂pi

, K =∂F2

∂t+H. (6)

Equation (6) is called the canonical transformation of the function F2

.

Similarly we can derive the following equation associated with

F3 (p,Q, t) .

164

Page 165: MSc Maths Optional Paper V

Consider

F3 (p,Q, t) = F1 (q,Q, t) −∑

qipi (7)

(7) ⇒F3 = dF1 (q,Q, t) −

qidpi −∑

piqi

Using (1),

dF3 =∑

pidqi −∑

PidQi −Hdt+Kdt−∑

qidpi −∑

pidqi

dF3 ( p,Q, t) = −∑

PidQi −Hdt+Kdt−∑

qidpi (8)

Then the total differentiating dF3 (p,Q, t)

dF3 (p,Q, t) =∑ ∂F3

∂Pi

dpi +∑ ∂F3

∂Qi

dQi +∂F3

∂tdt. (9)

Equating the coefficients in (8) and (9),

−qi =∂F3

∂Pi

(i = 1, 2, . . . , n)

−Pi =∂F3

∂Qi

(i = 1, 2, . . . , n)

K = H +∂F3

∂t

(10)

which is the canonical transformation of equation (8).

Similarly for next generating function F4 (p, P, t) we derived as fol-

lows.

F4 (p, P, t) = F2 (q, P, t) −∑

qiPi (11)

dF4 = dF2 −∑

pidqi −∑

qidpi

Using (3),

dF4 =∑

pidqi +∑

QidPi −Hdt+Kdt−∑

pidqi −∑

qidpi (12)

dF4 =∑

QidPi −Hdt+Kdt−∑

qidpi.

165

Page 166: MSc Maths Optional Paper V

Taking the total differential of F4 ,

dF4 =∑ ∂F4

∂pi

dpi +∑ ∂F4

∂Pi

dPi +∂F4

∂tdt (13)

Equating the coefficient in (12) and (13),

Qi =∂F4

∂Pi

(i = 1, 2, . . . , n)

qi = −∂F4

∂pi

(i = 1, 2, . . . , n)

K = H +∂F4

∂t

(14)

which is the canonical transformation of F4 (p, P, t) .

Equations (4), (7) and (11) gives the relation between the generat-

ing function which is the principle function.

Book Work: Consider the transformation

Q = logsin p

q, P = q cot p

Obtain the four major types of generating function associated with

the transformation.

Proof: Given that,

Q = logsin p

q, P = q cot p

We know that,

pδq − PδQ = δψ (1)

δQ =1

sin p

q

[

1

qcos pδP + sin p

(

− 1

q2

)

δq

]

=q

sin p

[

cos p

qδP − sin p

q2δq

]

= cot pδp− 1

qδq

166

Page 167: MSc Maths Optional Paper V

pδq − q cot p

[

cot pδp− 1

qδq

]

= δψ

pδq − q cot2 pδp+ cot pδq = δψ

(p+ cot p) δq − q cot2 pδp = δψ

∂p(p+ cot p) = 1 +

(

− cosec2 p)

= 1 − cosec c2p = − cot2 p

∂p(p+ cot p) = − cot2 p

∂q

(

−q cot2p)

= − cot2 p

∂p(p+ cot p) =

∂q

(

−q cot2p)

= δψ

∴ The transformation is canonical.

Let ψ = pq + q cot p (from (1)).

The geometrical representation of the transformation

sin p =q

e−Q= qeQ

cos p =p

e−Q= peQ

cot p =cos p

sin p=peQ

qeQ=p

qq cot p = p

Now,

p2 + q2 =(

e−Q)2

= e−2Q

p2 = e−2Q − q2

p =√

e−2Q − q2

167

Page 168: MSc Maths Optional Paper V

=1

eQ

[

1 − q2e2Q

]

We know that

cos p = peQ

=1

eQ

[

1 − q2e2Q

]

eQ

cos p =√

1 − q2e2Q

p = cos−1

(

1 − q2e2Q

)

From the principle of generating function

ψ (p, q) = F1 (q,Q) (2)

F1 (q,Q) = q cos−1

(

1 − q2e2Q

)

+√

e−2Q − q2 (3)

∂F1

∂q= cos−1

(

1 − q2e2Q

)

= p

∂F1

∂Q=√

e−2Q − q2 = −p

Comparing (2) and (3) equating the coefficient we get

p = cos−1

(

1 − q2e2Q

)

,

p =√

e−2Q − q2.

The above canonical can be written as

∂F1

∂q= cos−1

(

1 − q2e2Q

)

= p,

∂F1

∂q= −

(

e−2Q − q2

)

= −p.

168

Page 169: MSc Maths Optional Paper V

To find the generating function F2 (q, P ) . From the principle of gen-

erating function

F2 (q, P ) = F1 (q,Q, t) +∑

QiPi

F2 = pq + q cotP +QP (4)

F2 is the generations function of the variable (q, P ) .

We know that

q cot p = P

cot p =P

q

tan p =q

P⇒ p = tan−1

( q

P

)

From the triangle

P 2 = e−2Q − q2

e−Q =√

p2 + q2

eQ =1

p2 + q2

Q = log

(

1√

p2 + q2

)

= log 1 − log(

p2 + q2

)

= − log(

p2 + q2

)

Substitute p,Q in equation (4) we get

F2 (q, P ) = q tan−1

( q

P

)

+ p+(

− log(

p2 + q2

))

p

= q tan−1

( q

P

)

+ p(

1 − log(

p2 + q2

))

(5)

From (4) and (5) we get

169

Page 170: MSc Maths Optional Paper V

∂F2

∂q= tan−1

( q

P

)

= p

∂F2

∂p= 0 − log

q2 + p2 = Q

which are the canonical equation of the function F2 .

To find the function F3 ,

F3 = F3 (p,Q)

We know that,

F3 (p,Q) = F1 (q,Q) −∑

qipi

= qp+ q cotP − qp

F3 = q cot p (6)

q cot p = qcos p

sin p= q · Pe

Q

qeQ= p

Next to find the function F4 (p, P ) .

Similarly

F4 (p, P ) = F2 − qp

= qp+ q cot p+QP − qp

= q cot p+QP

F4 (p, P ) = P +QP

cos p = p = PeQ

eQ =cosP

p

Q = log

(

cosP

p

)

F4 = p+ log

(

cosP

p

)

P

170

Page 171: MSc Maths Optional Paper V

F4 = p+ P log

(

cos p

p

)

∂F4

∂p= q (− tan p cot p) = −p tan p = −q

∂F4

∂p= log

(

cosP

p

)

= Q

Hence the four generating function.

Special transformation

1.Identity transformation:

Let us consider the identity transformation which is an obvious

example of a canonical transformation. It is generated by a function

is of the form,

Tn =n∑

i=1

qipi (1)

Then

Pi =∂F2

∂qi= pi (i = 1, 2, . . . , n)

Qi =∂F2

∂Pi

= qi (i = 1, 2, . . . , n) . (2)

The identity transformation is generated by

F3 =n∑

i=1

PiQi.

2. Orthogonal transformation:

This is the simple canonical transformation. The orthogonal trans-

formation of q ’s and p ’s generated by

F2 =n∑

i=1

n∑

j=1

aijpiqj,

where ai ’s are constant meeting the orthogonality conditions

n∑

i=1

aijajk = δjk.

171

Page 172: MSc Maths Optional Paper V

δjk is the kroneckal delta the orthogonality can be written in the matrix

form

a · a−1 = a−1 · a = 1.

The transformation equation are

Pj =∂F2

∂qi=∑

aij, Qi =∂F2

∂pi

=∑

aijqj.

These equation can be solved for pi ’s in terms of Pi ’s and Qi ’s in

terms of qi’s then the transformation equations are

Qi =n∑

j=1

aijqi

Pi =n∑

j=1

aijPj (i = 1, 2, . . . , n)

These equations represents the equal rotation is q -space and p -space.

3. Translation:

We know that F2 =∑

qiPi, replace qi by qi + ci and replace Pi by

Pi − di

F2 =∑

qiPi

=∑

(qi + ci) (Pi−di)

=∑

qipi + ciPi − di

We have to omit cidi . Now,

pi =∂F2

∂qi= Pi − di

Qi =∂F2

∂Pi

= qi + ci

Qi = qi + ci,

Pi = Pi + di (i = 1, 2, . . . , n)

172

Page 173: MSc Maths Optional Paper V

which represents the required translation.

Homogeneous Canonical transformation:

We know that the differential form is

n∑

i=1

piδqi −n∑

i=1

PiQi = δψ (1)

is an exact differential. Consider the function φ and ψ are identically

zero. Thenn∑

i=1

(Piδqi − PiδQi) = 0

and the corresponding transformation is called a homogeneous canon-

ical transformation. This transformation is also known as Mathieu

transformation (or) contact transformation.

Point transformation:

We consider the class of homogeneous canonical transformation for

which full set of n independent function Ωj (q,Q, t) exist and equal to

zero.

This implies that the two n× n coefficient matrices of

n∑

i=1

∂Ωj

∂qjδqi +

n∑

i=1

∂Ωj

∂Qi

δQi = 0 (j = 1, 2, . . . ,m)

are non singular and the corresponding determinants are non zero,

that is,∣

∂Ωj

∂qi

6= 0,

∂Ωj

∂Qi

6= 0.

Hence we can solve for the Q’s in terms of (q, t) on for the q ’s as

functions of (Q, t) .

We have n equation is of the form

Qi = fi (q, t) i = 1, 2, . . . , n

where the fi ’s are twice differentiable. These equations represent

a point transformation. i.e., they represent a mapping of points in

173

Page 174: MSc Maths Optional Paper V

configuration space. If one also includes the equations for the P ’s,

the entire set represents a transformation in phase space and is known

as extended point transformation.

Momentum transformation:

The momentum transformation equation are obtained from

pi =n∑

j=1

Pj

∂Qi

∂qi(i = 1, 2, . . . , n)

which in this case takes the form

pi =n∑

j=1

∂fi

∂qi(i = 1, 2, . . . , n) .

Thus the p ’s are homogeneous linear functions of the P ’s and vice

versa. If we define nΩ ’s of the form

Ωj = Qj − fj (q, t) (j = 1, 2, . . . , n)

If the represents the transformation is called the momentum transfor-

mation.

Non homogeneous point transformation:

Let the basic differential form be

n∑

i=1

piδqi −n∑

i=1

PiδQi = δψ. (1)

Consider

Qi = fi (q, t) ,

Qj = fj (q, t)

⇒ δQj =n∑

j=1

∂fi

∂qiδqi. (2)

174

Page 175: MSc Maths Optional Paper V

Substitute the equation (2) in (1) implies

n∑

i=1

piδqi −n∑

i=1

Pi

[

n∑

j=1

∂fi

∂qiδqi

]

= δψ. (3)

In general,

δψ =n∑

i=1

∂ψ

∂qiδqi +

n∑

i=1

∂ψ

∂Pi

δpi (4)

Here the δq ’s and δp ’s are independent.Equation (3) and (4) we get

∂ψ

∂qi=

n∑

i=1

pi −n∑

i=1

Pi

n∑

j=1

∂fj

∂qi

=n∑

i=1

[

pi − Pi

n∑

j=1

∂fj

∂qi

]

pi =∂ψ

∂qi+ Pi

n∑

j=1

∂fi

∂qi(i = 1, 2, . . . , n)

⇒ ∂ψ

∂pi

= 0

Equation (4) implies ψ (q, t) is not an explicit function of the p ’s.

Furthermore it has the proper form for a generating function of the

type of fi . Hence we can choose F1 to be identically equal to ψ for a

point transformation.

Example: Illustrate the theory for the nonhomogeneous case, sup-

pose we are given sceleronomic extended point transformation

Q = tan q, P = (p−mν0) cos2 q,

where m and ν0 are constants.

Solution: Given that Q = tan q, P = (p−mν0) cos2 q , we wish

to show that the transformation is canonical.

δQ = sec2 qδq

= Pδq =[

(p−mν0) sec2 q]

sec2 qδq

175

Page 176: MSc Maths Optional Paper V

= δψ

∴ pδq = pδq +mν0δq = δψ

⇒ mν0δq = δψ (1)

which is an exact differential integrating

F1 = ψ = mν0q. (2)

Hence this is a nonhomogeneous canonical transformation. Now the

point transformation is,

Qi (q, t) = fi (q, t)

Q = tan q − f (q) .

From the non homogeneous point transformation

Ωj = Qi − fi (q, t)

Ω = Q− tan q

which yields K = H . Thus the two Hamilton functions are equal

F2 (q, P ) = F1 +QP

= mv0q +QP

= mv0q + tan qP

p =∂F2

∂q= mv0 + P sec2 q

Q =∂F2

∂P= tan q.

Here the canonical equations for the generating functionF2 .

To find F3 :

F3 (P,Q) = F1 − qP

176

Page 177: MSc Maths Optional Paper V

= mv0q − qP∂F3

∂P= −q, ∂F3

∂Q=

mv0

1 +Q2.

To find F4 :

F4 (p, P ) = F2 − qp = F1 +QP − qp

= mv0q + tan qP − qp∂F4

∂p= −q, ∂F4

∂P= tan q.

Now let us apply this transformation to a specific physical situation.

Consider a mass spring system which is attached to a plane that is

translating with a constant velocity v0 as shown in the following figure.

0v

km

0q+l

The unstressed length of the spring. Hence we have a conservative

holonomic system (or) Hamilton’s equations.

L = T − V

L =1

2m (v0 + q)2 − 1

2Kq2

p =∂L

∂q=

1

2m2 (v0 + q) = m (v0 + q)

The Hamilton function is H = T2 − T0 + V from the mass spring

system.

H =p2

2m− v0p+

1

2Kq2

177

Page 178: MSc Maths Optional Paper V

Hence the canonical equation of the motion for (q, p) variables are,

q =∂H

∂p=

p

m− v0,

p = −∂H∂q

= −1

2K (2q) = −Kq

Now let us obtain L∗

(

Q, Q)

relative to the moving frame. Here,

T =1

2mq2

V =1

2Kq2

Given that Q = tan q ⇒ q = tan−1Q.

q =1

1 +Q2Q

L∗ = T − V

=1

2m

(

Q2

(1 +Q2)2

)

− 1

2K(

tan−1Q)2

=1

2

mQ2

(1 +Q2)2− 1

2K(

tan−1Q)2

P =∂Lα

∂Q=

m2Q

2 (1 +Q2)2=

mQ

(1 +Q2)2

Given that

P = (p−mv0) cos2 q

cos2 q =P

p−mv0

=mQ

(1 +Q2)2mq=

Q

q (1 +Q2)2

cos2 q =Qq

(1 +Q2)2

=1 +Q2

(1 +Q2)2=

1

1 +Q2

[

SinceQ

q= 1 +Q2

]

178

Page 179: MSc Maths Optional Paper V

cos q =1

1 +Q2

In this case in K.E consist of T2 terms only Hamiltonian function.

H = T2 + V

=p2

2m+

1

2Kq2

K = T + V

=1

2mq2 +

1

2Kq2

=1

2m

[

Q2

(1 +Q2)2

]

+1

2K(

tan−1Q)2

We know that

P =mQ

(1 +Q2)2

mQ = P(

1 +Q2)2

Q =P (1 +Q2)

2

m

K =1

2mQ

Q

(1 +Q2)2+

1

2K(

tan−1Q)2

=1

2p(1 +Q2)

2

(1 +Q2)2p(1 +Q2)

2

m+

1

2K(

tan−1Q)2

=p2

2m

(

1 +Q2)2

+1

2K(

tan−1Q)2

Here the canonical equations are

Q =∂K

∂p=

1

2m2p(

1 +Q2)2

=p

m

(

1 +Q2)2

P = −∂K∂Q

= −[

p2

2m

[(

1 +Q2)

2θ]

+1

2K2 tan−1Q

(

1

1 +Q2

)]

179

Page 180: MSc Maths Optional Paper V

= −2p2Q

m

(

1 +Q2)

− K tan−1Q

1 +Q2.

Hence the canonical equation.

Lagrangian and Poisson brackets:

Lagrangian brackets:

Suppose the given transformation equation

Qi = Qi(q, P, t)

Pi = Pi(q, p, t)(i = 1, 2, · · · , n)

Let us consider the canonical differential form

n∑

i=1

piδqi −n∑

i=1

PiδQi = δψ (1)

δQi =n∑

i=1

∂Qi

∂qiδqi +

n∑

i=1

∂Qi

∂Pj

δPj (2)

Using (2) in (1)

n∑

i=1

piδqi −n∑

i=1

Pi

[

n∑

j=1

∂Qi

∂qjδqj +

n∑

j=1

∂Qi

∂Pj

δPj

]

= δψ

n∑

j=1

[

pj −n∑

i=1

Pi

∂Qi

∂qj

]

δqj −n∑

i=1

n∑

j=1

Pi

∂Qi

∂Pj

δPj = δψ

Now,

∂qk

(

pj −n∑

i=1

Pi

∂Qi

∂qi

)

=∂

∂qj

(

pk −n∑

i=1

Pi

∂Qi

∂qk

)

∂pk

(

n∑

i=1

Pi

∂Qi

∂pj

)

=∂

∂pj

(

n∑

i=1

Pi

∂Qi

∂Pk

)

∂pk

(

pj −n∑

i=1

Pi

∂Qi

∂qj

)

=∂

∂qj

(

−n∑

i=1

Pi

∂Qi

∂pk

)

180

Page 181: MSc Maths Optional Paper V

where j, k = 1, 2, . . . .n.These equation can be written as

n∑

i=1

(

∂Qi

∂qi

∂Pi

∂qk− ∂Pi

∂qi

∂Qi

∂qk

)

= 0

n∑

i=1

(

∂Qi

∂pj

∂Pi

∂pK

− ∂Pi

∂pj

∂Qi

∂pK

)

= 0

n∑

i=1

(

∂Qi

∂qj

∂Pi

∂pK

− ∂Pi

∂qj

∂Qi

∂pK

)

= δjk

where δjkis the kronecker delta.

Definition: Lagrangian brackets expression of two variable (u, v)

by using the notation

[u, v] =n∑

i=1

(

∂Qi

∂u

∂Pi

∂v− ∂Pi

∂u

∂Qi

∂v

)

where u andv are any two variables q1, q2, . . . , qn, p1, p2, . . . , pn .

Skew symmetry: As a consequence of a two skew symmetry of

the Lagrangian bracket is

[u, v] = − [v, u]

and

[u, u] = [v, v] = 0.

Invariant: A general characteristic of the Lagrangian bracket is

invariant under a canonical transformation.

Example: If the 2n variables (q, p) and in variable (Q,P ) are

related by a canonical transformation. The either set may be used in

equating a given bracket evaluation. Thus we have

[u, v] =n∑

i=1

(

∂qi∂u

∂pi

∂v− ∂pi

∂u

∂qi∂v

)

=n∑

i=1

(

∂Qi

∂u

∂Pi

∂v− ∂Pi

∂u

∂Qi

∂v

)

181

Page 182: MSc Maths Optional Paper V

where each set of dynamical variable in assume to be a function of

(u, v) as well as other variable.

Poisson brackets: Suppose that the function of the dynamical

variable and time namely u = u (q, p, t) and v = v (q, p, t). The poisson

bracket expression for function is

(u, v) =n∑

i=1

(

∂u

∂qi

∂v

∂pi

− ∂u

∂pi

∂v

∂qi

)

.

In the case of Lagrangian brackets, we have

(u, v) = − (v, u)

(u, u) = (v, v) = 0.

The poisson brackets is useful testing whether the transformation is

canonical (or) not.

Poisson theorem:

Statement: If u (q, p) and v (q, p) are integrals of a Hamiltonian

system, then the Poisson bracket (u, v) is also an integral, that is (u, v)

is constant of the motion.

Proof: Let u and v be integrals of the motion. We know that,

df

dt= (f,H) +

∂f

∂t,

(u,H) +∂u

∂t= 0, (1)

(v,H) +∂v

∂t= 0. (2)

Also,

d

dt(u, v) = [(u, v) , H] +

∂t(u, v)

d

dt(u, v) = [(u, v) , H] +

(

∂u

∂t+ v

)

+

(

u,∂v

∂t

)

(3)

182

Page 183: MSc Maths Optional Paper V

From (1) and (3)

d

dt(u, v) = [(u, v) , H] − [(u,H) , v] − [u, (v,H)]

= [(u, 0) , H] + [(H, u) , v] + [(v,H) , u]

By Jacobi identity,

[u, (v, w)] + [v, (w, u)] + [w, (u, v)] = 0

d

dt(u, v) = 0.

Hence the proof.

Example: Consider the transformation

Q =√

e−2q − p2, P = cos−1 (peq) .

Use the poisson bracket to show that it is canonical.

Solution: We know that the poisson bracket

(u, v) =n∑

i=1

(

∂u

∂qi

∂v

∂pi

− ∂u

∂pi

∂v

∂qi

)

. (1)

Given that

Q =√

e−2q − p2,

P = cos−1 (peq) .

Here u = Q, v = P.

(Q,P ) =∂Q

∂q

∂P

∂p− ∂Q

∂q

∂P

∂p(2)

Q =√

e−2q − p2

∂Q

∂q=

1

2

(

e−2q (−2)√

e−2q − p2

)

183

Page 184: MSc Maths Optional Paper V

= − e−2q

e−2q − p2

=−p

e−2q − p2

P = cos−1 (peq)

∂P

∂q=

(−1) peq

1 − p2e2q

=−p

e−2q − p2

∂P

∂p=

−e2q

1 − p2e2q

=−1

e−2q − p2.

Using these values in (2) we get

(Q,P ) =e−2q

(

e−2q − p2

)2− p2

(

e−2q − p2

)2=e−2q − p2

e−2q − p2= 1

We know that (u, u) = (v, v) = 0

⇒ (Q,Q) = 0, (P, P ) = 0

Hence the given transformation is canonical.

Bilinear co-variant:

This is the another method which may be used in testing whether

a given transformation is canonical involves the bilinear co-variant.

Suppose we consider the Pfaffian differential

Ω =n∑

i=1

Xi (x) dxi (1)

where the δx ’s are an independent set of infinitesimal displacements

from the same reference point. Now we can write,

δΩ =n∑

i=1

(δXidxi +Xiδdxi)

184

Page 185: MSc Maths Optional Paper V

dQ =n∑

j=1

(dXjδxj +Xjdδxj)

where

δXi =n∑

j=1

∂Xj

∂xj

δxj,

dXj =n∑

i=1

∂Xj

∂xi

dxi.

Then we obtain

δΩ − dQ =n∑

i=1

n∑

j=1

(

∂Xi

∂xj

− ∂Xj

∂xi

)

dxiδxj

=n∑

i=1

n∑

j=1

[(δXjdxi − dxjδxj) + (Xjδdxi −Xδxj)] (2)

note that δdxi = dδxi

Now, considering contemporaneous variants where

τ ij =∂Xi

∂xj

− ∂Xj

∂xi

.

We find that

δΩdθ =n∑

i=1

n∑

j=1

τ ijdXiδxj

This is the bilinear co-variant associated with the differential equation

(1).

185